Вы находитесь на странице: 1из 231

a2 b2

ab
ab
2
1
1
1
9

2
2
2 4(ab bc ca)
(a b) (b c) (c a)

a
b
c
3

bc ca ab 2

vismPaBeRCIserIs
CMBUkTI1

lMhat;eRCIserIs
a2 b2
ab
2

!> cUrRsaybBaak;fa a b
RKb;cMnYnBitviCman a nig b .
@> cUrRsaybBaak;facMeBaHRKb; a , b , c 0 eKmanvismPaB
8
(a b )(b c)(a c) (a b c)(ab bc ca)
9
#> cUrbgajfa (a b)(b c)(c3 a) 278
(a b c )

cMeBaHRKb;cMnYnBitviCman a , b , c .
$> eK[cMnYnBitviCman a , b , c , x , y , z .
cUrRsayfa 3 (x a)(y b)(z c) 3 xyz 3 abc
%> cMeBaHRKb;cMnYnBitviCman a , b nig c cUrRsaybBaak;fa
a
b
c
3

.
bc ca ab 2
^> eK[ x , y , z CabIcMnYnBitviCman .
cUrbgajfa 1 2 1 2 1 2 (x 2y()(xyyz)(zx) z )
(x y )

eroberogeday lwm pln

(y z )

(z x)

- Page 1 -

vismPaBeRCIserIs
&> cUrRsaybBaak;vismPaB
a 2 (1 b )2 b 2 (1 c)2 c2 (1 a )2

cMeBaHRKb;cMnYnBit a , b , c .
*> ebI a , b , c (0 , 1 ) cUrbgajfa

3 2
2

abc (1 a )(1 b )(1 c) 1

(>eK[ a , b , c CabIcMnYnBitviCmanEdl abc 1 .


cUrRsayfa a cb b ac c ba a b c 3
!0> eK[ a , b , c , d CabYncMnYnBitEdl a2 b2 c2 d2 4 .
cUrRsaybBaak;fa a3 b3 c3 d3 8 .
!!> eK[ a , b , c CabIcMnYnBitminGviCman .
cUrRsayfa a3 b3 c3 3abc 2 ( b 2 c a)3 .
!@> eK[ a , b , c CabIcMnYnBitviCmanEdl abc 1 .
a b c 5 a2 b 2 c2
cUrbgajfa 3
3
!#> eK[ a , b , c CabIcMnYnBitxusKa .

cUrRsayfa

a2
b2
c2

2
2
2
2
(b c )
(c a )
(a b )

eroberogeday lwm pln

- Page 2 -

vismPaBeRCIserIs
!$> eK[ a , b , c CabIcMnYnBitminGviCmanEdl a2 b2 c2 a b c
cUrRsayfa a2b2 b2c2 c2a2 ab bc ca .
!%> eK[ a , b , c CabIcMnYnBitminGviCman . cUrRsayfa
(a 2 ab b 2 )(b 2 bc c2 )(c2 ca a 2 ) (ab bc ca)3

!^> eK[ a , b , c CabIcMnYnBitminGviCmanEdl abc 1 . cUrRsayfa


c a3 b 3 a b 3 c3 b c3 a3
3

.
2
2
2
2
2
2
2
a b
b c
c a
!&> eK[ a , b , c CabIcMnYnBitviCman . cUrRsaybBaak;vismPaB
a3
b3
c3

3
1
3
3
3
3
3
a (b c )
b (c a )
c (a b )

!*> cMeBaHRKb;cMnYnBitviCman a , b , c cUrRsayfa


a
b
c
3

ab
bc
ca
2
!(> cMeBaHRKb;cMnYnBitminGviCman x , y , z cUrRsayfa
2(1 x 2 )(1 y 2 )(1 z 2 ) (1 x )(1 y )(1 z )(1 xyz )

@0> cMeBaHRKb;cMnYnBitminGviCman a, b, c, d cUrRsayfa


4(1 a a 2 )(1 b b 2 )(1 c c2 )(1 d d 2 ) (1 abcd )2

@!> ebIsmIkar x4 ax3 2x2 bx 1 0 manbsy:agticmYy


CacMnYnBitenaHbgajfa a4 b4 8 .
eroberogeday lwm pln

- Page 3 -

vismPaBeRCIserIs
@@> cUrRsaybBaak;fa
a
b
c
9

2
2
2
4(a b c)
(b c )
(c a )
(a b )

cMeBaHRKb; a , b , c CacMnYnBitviCman .
a3 b 3 c3 a2 b 2 c2
@#> cUrRsaybBaak;fa 2 2 2 b c a
b
c
a
cMeBaHRKb; a , b , c CacMnYnBitviCman .
@$> eK[ x , y , z CabIcMnYnBitviCman . cUrRsaybBaak;fa
x
y
z

1
x ( x y )( x z ) y ( y z )( y x ) z ( z x )( z y )

@%> RKb;cMnYnBitviCman a , b , c cUrRsaybBaak;fa


bc ca ab

a
b
c

b
c
a

b
c
c
a
a
b

@^> eK[ a , b , c CabIcMnYnBitviCmanEdl abc 1 .

cUrRsayfa (a b)(b c)(c a) 4(a b c 1)


@&> cUrbgajfa (a2 2)(b2 2)(c2 2) 9(ab bc ca)
cMeBaHRKb;cMnYnBitviCman a , b , c .
@*> eKyk a , b , c CabIcMnYnBitviCman . cUrbgajfa
(b c a )2
(c a b )2
(a b c )2 3

2
2
2
2
2
2
5
(b c ) a
(c a ) b
(a b ) c

eroberogeday lwm pln

- Page 4 -

vismPaBeRCIserIs
@(> eK[ x , y , z , t CabYncMnYnBitviCman . cUrRsayfa
2

x y z t




1 .
x y y z z t t x
#0> eK[ a , b , c nig x , y , z CacMnYnBit . cUrRsaybBaak;fa
4(a 2 x 2 )(b 2 y 2 )(c 2 z 2 ) 3(bcx cay abz )2

#!> eK[bIcMnYnBit a 1 , b 1 , c 1 . cUrRsaybBaak;fa


1 a2
1 b2
1 c2

2 .
2
2
2
1 b c 1 c a
1 a b
#@> eK[bIcMnYnBitviCman a , b , c Edl abc 1 . cUrRsaybBaak;fa
1
1
1
3

.
3
3
3
8
(1 a )

(1 b )

(1 c)

##> eK[bIcMnYnBitviCman a , b , c Edl a2 b2 c2 (a b c)2 4


cUrRsaybBaak;fa ab 12 bc 12 ca 12 3 .
(a b )

(b c )

(c a )

#$> eK[bIcMnYnBitviCman a , b nig c . cUrRsayfa


(a5 a 2 3)(b 5 b 2 3)(c5 c 2 3) (a b c)3

#%> cUrRsayfacMeBaHRKb;cMnYnBitviCman a , b , c eKmanvismPaB


1
1
1
1

a 3 b 3 abc b 3 c 3 abc c 3 a 3 abc abc

eroberogeday lwm pln

- Page 5 -

vismPaBeRCIserIs
mm 1 nn 1
#^> eKyk a m n Edl m nig n
m n
cUrRsayfa am an mm nn .

CacMnYnKt;viCman .

#&> eK[ x ; y ; z CabIcMnYnBitviCmanEdl xyz 1 .


cUrRsaybBaak;fa
x9 y 9
x x y y
6

3 3

y9 z9
y y z z
6

3 3

z 9 x9
z z x x
6

3 3

#*> eK[ a , b , c CacMnYnviCmanEdl ab bc ca 3 .


1
cUrbgajfa 1 a 1(b c) 1 b 1(c a) 1 c (1a b) abc
2

#(> eK[ a nig b CaBIrcMnYnBitminGviCmanEdl a

cUrRsayfa

b2 4

ab
2 1
ab2
x3 y 3 z 3
3
xyz | ( x y )( y z )( z x ) |
3
4
x;y;z0

$0> cUrRsayfa
.
cMeBaHRKb;
$!> eK[ a , b , c CaRbEvgRCugnRtIekaNmYy .
cUrRsayfa b aa c a bb c a bc c 3 .
$@> eK[ a , b , c CabIcMnYnBitEdlepgpat; a b c 3 .
cUrbgajfa | a | | b | | c | abc 4 .
2

eroberogeday lwm pln

- Page 6 -

vismPaBeRCIserIs
$#> eK[ a , b , c CabIcMnYnBitviCmanEdl abc 1 .
cUrbgajfa 1 a b3 c ab bc6 ca .
$$> eK[ a 1 nig b 1 .
cUrbgajfa log a log b 2 log ( a b ) .
2
$%> eK[ x , y , z CabIcMnYnBitviCmanEdlepgpat; xyz 1 .
cUrbgajvismPaB
2

( x y 1) 2 ( y z 1) 2 ( z x 1) 2

xyz
z
x
y

$^>eK[ a , b , c CacMnYnBitviCman .
a 2b(b c) b 2c(c a ) c 2a(a b )
cUrbgajfa a b b c c a 0 .
$&> eK[ a , b , c CabIcMnYnBitviCman . cUrbgajfa
a5 b 5 c5 (a b c)5 10
(a b c ) 2
3
3
3
3
9
a b c (a b c )
$*> cMeBaHRKb;cMnYnBit x (0 , 4 )
cUrbgajfa (cos x)cos x (sin x)sin x .

eroberogeday lwm pln

- Page 7 -

vismPaBeRCIserIs
$(> eK[ a , b , c , d CabIcMnYnBitEdlepgpat;vismPaB
(a 2 b 2 1)(c 2 d 2 1) (ac bd 1)2

cUrbgajfa a b 1 nig c d 1 .
%0> eK[ a , b , c CabIcMnYnBitviCmanEdl ab bc ca 1 .
cUrkMnt;tmGb,brmankenSam E a a b bb c c c a .
%!> eK[ x1 , x2 , ..., xn Edl n 2 CacMnYnBitviCmanEdlepgpat;
2

1
1
1
1

....

xn 1998 1998
x1 1998 x 2 1998

cUrbgajfa x1n.x2 1...xn 1998 .


%@> eK[ x , y , z CabIcMnYnBitviCmanEdl xyz x y z .
cUrRsayfa 1xzy 1yxz 1zyx 227
.
xyz
%#> cUrbgajfaebI abc 8 nig a , b , c 0 enaHeK)an
n

a2
(1 a 3 )(1 b 3 )

b2
(1 b 3 )(1 c 3 )

c2
(1 c 3 )(1 a 3 )

%$> eK[RtIekaN ABC mYymanRCug BC a , AC b , AB c


ehIymanmMukgCamMuRsYc .
ab bc ca

4(a b c)
cUrRsayfa cos
C cos A cos B
eroberogeday lwm pln

- Page 8 -

4
3

vismPaBeRCIserIs
%%> eK[ x , y , z 0 Edl x y z 1 .
cUrRsayfa (1 x x) (1 y y ) (1 z z ) 14 .
%^> eK[ a , b , c , d CacMnYnBitviCmanEdl abcd 1 .
ebIeKdwgfa a b c d ba bc dc da enaHcUrbgajfa
3

b c d a
.
a b c d
bgajfa a b c d ba bc dc da
abcd

%&> eKyk a , b , c CacMnYnviCmanEdlepgpat; abc 1 .


cUrbgajfa (a 1)(a b 1) (b 1b)(c 1) (c 1)(c a 1) 43 .
%*> eKyk a , b , c CacMnYnviCman.
bc
cUrbgajfa (1 ba )(1 bc )(1 ac ) 2(1 a 3abc
).
%(> eKyk a , b , c CacMnYnBitncenaH (0 , 1 ) .
cUrbgajfa abc (1 a)(1 b)(1 c) 1 .
^0> eKyk a , b , c CacMnYnBitviCmanEdl ab bc ca abc
a4 b4
b 4 c4
c4 a4
cUrbgajfa ab(a3 b3 ) bc(b3 c3 ) ca(c3 a3 ) 1 .

eroberogeday lwm pln

- Page 9 -

vismPaBeRCIserIs
^!> eKyk x , y , z CacMnYnBitviCmanEdl xyz 1
cUrbgajfa yz1 z zx1 x xy1 y 32 .
^@> cMnYnBit a, , b , c , x , y , z epgpat; a b c 0
nig x y z 0 . cUrbgajfa
a2x2
b2y 2
c2z 2
3

(by cz )(bz cy ) (cz ax )(cx az ) (ax by )(ay bx ) 4


3 3 xyz
x
y
z

1 x 1 y 1 z 1 3 xyz

^#> cUrbgajfa
cMeBaHRKb; 0 x , y , z 1 .
^$> cUrbgajfacMeBaHRKb; a , b , c CacMnYnBitviCmaneK)an
a
b
c

1 .
2
2
2
a 8bc

b 8ca

c 8ab

bgajfa 2 a 2 b 2 c 1
a 8bc
b 8ca
c 8ab
^%> cUrbgajfacMeBaHRKb; a , b , c CacMnYnBitviCmaneK)an
(a 1)(b 1)2
3 c a 1
3

2 2

(b 1)(c 1)2
3 a b 1
3

2 2

(c 1)(a 1)2

3 b c 1
3

^^> eK[ x , y , z CacMnYnBitviCmanEdl


cUrRsayfa
x 2 yz
2x ( y z )
2

y 2 zx
2y ( z x )

eroberogeday lwm pln

2 2

abc3

x y z 1

z 2 xy
2z ( x y )
2

.
- Page 10 -

vismPaBeRCIserIs
^&> ebIcMnYnBitviCman a , b , c epgpat; a2 b2 c2 3
enaHcUrbgajfa
a2
b2
c2
(a b c ) 2

2
2
2
12
2bc
2ca
2ab

.
^*> eKmancMnYnBitviCman a , b , c epgpat; a b c 1 .
b ca c ab 3
cUrbgajfa aa bc
.

bc b ca c ab 2
^(> eK[ a , b , c Ca cMnYnBitviCman.
cUrbgajfa a 1 bc b 1 ca c 1 ab 12 ( ab1 bc1 ca1 ) .
&0> eK[ a , b , c , d CacMnYnBitviCman .
2

a4
b4
c4
d4
S

(a b )(a 2 b 2 ) (b c)(b 2 c 2 ) (c d )(c 2 d 2 ) (d a)(d 2 a 2 )

cUrbgajfa S a b 4 c d .
&!> eK[cMnYnBitviCman x1 , x2 , ...., xn ehIyeKtag
Sn x1 x 2 ... xn . cUrRsayfa
S2
Sn
(1 x1 )(1 x 2 )....(1 xn ) 1 S ...
2!
n!

eroberogeday lwm pln

- Page 11 -

vismPaBeRCIserIs
&@> cMeBaHRKb;cMnYnKt;viCman n nig cMeBaHRKb;cMnYnBitviCman
a1 , a 2 , ...., a n epgpat; a1a 2a 3 ....a n 1 .
n
ai
1 n 1
cUrbgajfa 4 2 a .
i 1

ai 3

i 1 i

&#> eK[ a , b , c CacMnYnBitviCman edaydwgfa


1 1 1
a b c . cUrbgajfa
a b c
1
( 2a b c)

1
(a 2b c)

1
(a b 2c)

3
16

&$> smIkar x3 ax2 bx c 0 manbbICacMnYnBitviCman

mincaM)ac;xusKa .
cUrkMNt;tmGb,brmaEdlGacn 13a2abbc bc .
&%> cUrRsayfa
2
2 2
2
(
a
b
)(
a
ab
b
) 2 2 2 2 1
1
1

4
(
a
b
c
)(
)

2
3
ab bc ca
Cyc

cMeBaHRKb;cMnYnBitviCman a , b , c .
&^> ebI a , b , c CargVas;RCugnRtIekaNmYy ehIy r CakaMrgVg;
carwkkgnRtIekaNenaHcUrRsayfa a12 b12 c12 41r 2 .
eroberogeday lwm pln

- Page 12 -

vismPaBeRCIserIs
&&> eK[ a , b , c CacMnYnBitviCmanehIyepgpat;lkxN
16(a b c)

1 1 1

a b c

1
(a b 2a 2c)3

. cUrRsaybBaak;fa
1

1
8

(b c 2a 2b)3 (c a 2b 2c)3 9

&*> eK[ a , b , c CacMnYnBitviCmanEdl a b c 1 .

cUrbgajfa ab 2 bc 2 ca 2 83 .
1 c
1 a
1b
&(> eK[ a , b , c CabIcMnYnBitminGviCman nig x , y , z
CabIcMnYnBitviCmanedaydwgfa a b c x y z .
a3 b 3 c3
cUrRsaybBaak;fa 2 2 2 a b c .
x

*0> eK[ a , b , c 0 . cUrRsaybBaak;fa


a 3 4(b 3 c 3 ) b 3 4(c 3 a 3 ) c 3 4( a 3 b 3 ) 9

bc
ca
ab
2
*!> eKeGaybIcMnYnBitviCman a , b , c . cUrRsayfa
a3
b3
c3
3

(b c )3 (c a )3 (a b )3 8

eroberogeday lwm pln

- Page 13 -

vismPaBeRCIserIs
*@> cMeBaHRKb;cMnYnBitviCman a , b , c eKkMNt;tag A a b3 c
3
G abc nig
.
1 1 1

a b c
3
A
cUrRsayfa G 14 43 . HA ?

H

*#> eK[cMnYnBit x1 , x2 , y1 , y 2 , z1 , z 2 epgpat; x1 0 , x2 0


x1y1 z12 0 nig x 2 y 2 z 2 2 0 .
cUrRsaybBaak;fa
8
( x1 x 2 )( y1 y 2 ) ( z1 z 2 )

1
x1y1 z1

1
x2y 2 z 2

(*)

*$> eK[RtIekaN ABC mYymanmMukgCamMuRsYc . cUrRsayfa


3 2
cos A cos B cos C
.
2
*%> eK[ x ; y ; z CacMnYnBitviCmanEdl xyz 1 . cUrRsayfa

1
1
1
1

( x 1)2 y 2 1 ( y 1)2 z 2 1 ( z 1)2 x 2 1 2

*^> eK[ a ; b ; c CacMnYnBitviCmanEdl abc 1 .


cUrRsayfa
a(b 2 b ) b(c 2 c ) c(a 2 a ) 0

eroberogeday lwm pln

- Page 14 -

vismPaBeRCIserIs
*&> eK[ a1 , a2 , ..., an 0 nig x1 , x2 , ..., xn 0 .
cUrRsaytamkMeNInfa
a
a
a
(a a ... a )
.

...

x
x
x
x x .... x
**> eK[ a ; b ; c CabIcMnYnBitviCmanEdl abc 1 .
cUrbgajfa
1
1
1
3

?
3
3
3
2
a ( b c ) b (c a ) c (a b )
*(> eK[ x ; y ; z 0 . cUrRsaybBaak;fa
2

x
y
z
1

x 2y 3z y 2z 3x z 2x 3y 2

(0> eK[sIVt a1 ; a2 ; ....; an epgpat;lkxN


a1 0 ; | a 2 || a1 1 |; ... nig | an || an 1 1 | .
cUrbgajfa a a n ... a 12 .
(!>eK[ a ; b ; c CabIcMnYnBitviCman .
cUrbgajfa
abc
c
b
a

) .
1 1 1 2 (1 3
a
c
b
abc

eroberogeday lwm pln

- Page 15 -

vismPaBeRCIserIs
(@>eK[ a ; b ; c CaRbEvgRCugrbs;RtIekaNmYy .
cUrbgajfa
abc bca cab a b c

(#>eK[RtIekaN ABC mYymanmMukgCamMuRsYc . cUrbgajfa


sin 2 A sin 2 B sin 2 C 2 3 sin A sin B sin C .
($>eK[ A ; B ; C CamMukgrbs;RtIekaN ABC mYy .
cUrbgajfa cot A2 cot B2 cot C2 3 3
(%>eK[ A ; B ; C CamMuRsYckgrbs;RtIekaN ABC mYy .
cUrbgajfa (1 tan A)(1 tan B)(1 tan C) (1 3 ) .
(^>eK[ a ; b ; c CabIcMnYnBitviCman . cUrRsayfa
(a 2)(b 2)(c 2) 9(ab bc ca) .
(&>RbsinebI xyx (1 x)(1 y )(1 z ) Edl 0 x ; y ; z 1
cUrbgajfa x(1 z ) y(1 x) z(1 y ) 43 .
(*>eK[ a ; b ; c CaRbEvgRCugrbs;RtIekaNmYyEdlman
brimaRtesI 2 .
cUrRsayfa 32 a b c 2abc 2 .
3

eroberogeday lwm pln

- Page 16 -

vismPaBeRCIserIs
((>eK[ x ; y ; z CabIcMnYnBitviCmanEdl x y z 1 .
cUrRsaybBaak;fa 1x 1 y1 1 1z 1 8 .

!00> eK[ a ; b ; c CabIcMnYnBitviCmanEdl a b c 1


1
1
1
2(a 3 b 3 c 3 )
bgajfa a2 b2 c2 3
abc
!0!> eK[ a , b , c CabIcMnYnBitviCman . cUrRsaybBaak;fa
2

a2
2a 2 ab b 2

b2
2b 2 bc c2

c2
2c2 ca a 2

abc
2

!0@> eK[bIcMnYnBitviCman a , b , c . cUrRsaybBaak;fa

29a 3 b 3 29b 3 c 3 29c 3 a 3


4(a b c)

2
2
2
ab 6a
bc 6b
ca 6c
!0#> eK[ a , b , c CabIcMnYnBitviCman . cUrRsaybBaak;vismPaB
a3
b3
c3
abc

3
a 2 ab b 2 b 2 bc c 2 c 2 ca a 2
!0$> eK[bIcMnYnBitviCman a , b , c Edl a b c 1 .

cUrRsaybBaak;fa
1
1
1
1

ab 2c 2 2c bc 2a 2 2a ca 2b 2 2b ab bc ca

eroberogeday lwm pln

- Page 17 -

vismPaBeRCIserIs
!0%> eK[RtIekaN ABC mYymanRCug BC a , CA b , AB c
ehIymMukg A , B , C CamMuRsYcbmMuEkg .
tag S CapRkLan ABC
cUrRsaybBaak;fa 14 14 14 9 2 .
a

!0^> cUrbgajfa

16 S

1
1
1
4

2
2
2
xy yz zx
(x y )
(y z )
(z x)

cMeBaHRKb;cMnYnBitminGviCmanxusKa x , y , z .
!0&> cUrbgajfa 1 1 1 1 1 1 1 1 1

d
ac
cMeBaHRKb;cMnYnBitviCman a , b , c , d .
a

bd

!0*> eK[ a , b , c CacMnYnBitviCman . cUrRsaybBaak;fa


( 2a b c)2
( 2b c a )2
( 2c a b )2
2
2
8
2
2
2
2
2a (b c)
2b (c a )
2c (a b )
!0(>eKyk x CacMnYnBit edaydwgfa 0 x 2 .
cUrbgajfa cos2 (x) cot(x) sin 2 (x) tan( x) 1 .

eroberogeday lwm pln

- Page 18 -

vismPaBeRCIserIs
21
!!0> sIVt {an } kMNt;eday a1 16
nigcMeBaH n 2 : 2an 3an1 n31
2
eKyk m CacMnYnKt;mYyEdl m 2 .
cUrbgajfacMeBaH n m eyIg)an
n ( m 1)

3
m2 1

2 m

an n 3 m

2
3


mn1

!!!> eK[ a , b , c CacMnYnBitviCmanEdl a b c 3 .


1
m

a2
b2
c2
3

a b 2 b c2 c a2 2

cUrRsaybBaak;vismPaB
!!@> eK[ a , b , c CaRCugrbs;RtIekaNmYy . cUrRsayfa

( c a b ) 4 ( a b c )4 ( b c a ) 4

ab bc ca
a(a b c) b(b c a ) c(c a b )

!!#> eKkMNt;cMnYn a0 , a1 , a2 , ...., an dUcxageRkam


1
ak 2
a 0 ; ak 1 ak
( n 1 , k 0 , 1 , 2 , ...., n 1 )
2
n
cUrbgajfa 1 n1 an 1 .
!!$> cMeBaHRKb; a , b , c FMCagmYy cUrbgajfa
loga bc logb ca logc ab 4 (logab c logbc a logca b)

eroberogeday lwm pln

- Page 19 -

vismPaBeRCIserIs
!!%> eKtag , , CargVas;mMukgRtIekaN ABC mYyEdlmanbrimaRt 2p
nigkaMrgVg;carwkeRkA R .
a / cUrRsaybBaak;fa
R2

cot cot cot 3 9 . 2 1


p

etIeBlNaeTIbeK)ansmPaB ?
!!^> eKyk a , b , c CabIcMnYnBitviCman . cUrbgajfa
b/

(b c a )2
(c a b )2
(a b c )2 3

2
2
2
2
2
2
5
(b c ) a
(c a ) b
(a b ) c

!!&> eK[bIcMnYnBitminGviCman a , b , c nigminsUnRBmKaBIr .


cUrRsaybBaak;fa

1
1
1 9
(ab bc ca)

2
2
2
(b c )
(c a ) 4
(a b )
www.mathtoday.wordpress.com

eroberogeday lwm pln

- Page 20 -

vismPaBeRCIserIs
CMBUkTI2

EpkdMeNaHRsay
lMhat;TI1
cUrRsayfa

ab

a2 b2
ab RKb;cMnYnBitviCman a
2
a2 b2
ab Bit
2

nig b

]bmafa a b
elIkGgTaMgBIrnvismPaBCakaereK)an

2
(a b )

a b

ab

a2 b2
a2 b2
2
2
a 2ab b
2 ab .
ab
2
2
2

a2 b 2
a2 b2
a 2ab b
2 ab .
ab 0
2
2
2

a2 b2

a b
a b

2 ab .
ab
ab 0

2
2
2

a2 b2
dUcenH a b
ab .
2
2

eroberogeday lwm pln

- Page 21 -

vismPaBeRCIserIs
lMhat;TI2
cUrRsaybBaak;facMeBaHRKb; a , b , c 0 eKmanvismPaB
8
(a b )(b c)(a c) (a b c)(ab bc ca)
9

eKmansmPaB

(a b)(b c)(c a) a2b a2c ab2 b2a ac2 bc2 2abc


(a b c)(ab bc ca) a2b a2c ab2 ab2 ac2 bc2 3abc

eFIVpldksmPaBBIrenHeK)an
(a b c)(ab bc ca) (a b )(b c)(c a ) abc (1)
a b 2 ab

tamvismPaB AM GM eKman b c 2 bc

c a 2 ac
eK)an (a b)(b c)(c a) 8abc (2)

yk (1) CMnYskg (2) eK)an


9(a b )(b c)(c a ) 8(a b c)(ab bc ca)
dUcenH (a b)(b c)(a c) 89 (a b c)(ab bc ca)

eroberogeday lwm pln

- Page 22 -

vismPaBeRCIserIs
lMhat;TI3
cUrbgajfa

(a b )(b c)(c a ) 8

3
27
(a b c )

cMeBaHRKb;cMnYnBitviCman a , b , c .
tamvismPaB AM GM eKman
(a b ) (b c) (c a ) 3 3 (a b )(b c)(a c)
2(a b c) 3 3 (a b )(b c)(c a )

elIkGgTaMgBIrCasVyKuN 3 eK)an
8 (a b c)3 27(a b )(b c)(c a )
dUcenH (a b)(b c)(c3 a) 278 .
(a b c )

eroberogeday lwm pln

- Page 23 -

vismPaBeRCIserIs
lMhat;TI4
eK[cMnYnBitviCman a , b , c , x , y , z .
cUrRsayfa 3 (x a)(y b)(z c) 3 xyz 3 abc
eKman
( x a)(y b)(z c) xyz ayz bxz cxy abz acy bcx abc

tamvismPaB AM GM eKman
ayz bxz cxy 3 3 abcx 2y 2z 2
abz acy bcx 3 3 a 2b 2c 2 xyz

eK)an
( x a)( y b )( z x) xyz 33 abcx2y 2 z 2 3 3 a 2b 2c2xyz abc

( x a )( y b )( z c) ( 3 xyz 3 abc )3

dUcenH

( x a )( y b )( z c) 3 xyz 3 abc

eroberogeday lwm pln

- Page 24 -

vismPaBeRCIserIs
lMhat;TI5
cMeBaHRKb;cMnYnBitviCman a , b nig c cUrRsaybBaak;fa
a
b
c
3
.

bc ca ab 2
rebobTI1 tamvismPaB Cauchy Schwarz
(a1b1 a 2b 2 a 3b 3 )2 (a12 a 2 2 a 3 2 )(b12 b 2 2 b 3 2 )
yk a1 b a c , a2 c b a , a3 a c b
nig b1 a(b c) , b2 b(c a) , b3 c(a b)
a
b
c

)( 2ab 2bc 2ca)


(a b c ) 2 (
bc ca ab
a
b
c
(a b c )2
eKTaj b c c a a b 2(ab bc ca) (1)

tamvismPaB AM GM eKman
a2 b 2 b 2 c2 c2 a2

ab bc ac
2
2
2
b a2 b2 c2 ab bc ca

b (a b c)2 3(ab bc ca) (2)


tamvismPaB (1) nig (2) eKTaj)an
a
b
c
3
.

bc ca ab 2
eroberogeday lwm pln

- Page 25 -

vismPaBeRCIserIs
rebobTI2
a
b
c
3

eKman b c c a a b 2
a
b
c
3
1) (
1) (
1) 3
bc
ca
ab
2
b a b b c c a c b a c a a b b c 92
b (a b c) ( a 1 b b 1 c c 1 a ) 92
tamvismPaB AM GM eKman

eK)an

(a b ) (b c) (c a ) 3 3 (a b )(b c)(a c)
2(a b c) 3 3 (a b )(b c)(c a ) (1)
1
1
1
3
( 2)

3
ab bc ca
(a b )(b c)(c a )

KuNvismPaB (1) nig (2) Gg nig GgeK)an


1
1
1

)9
ab bc ca
eKTaj (a b c) ( a 1 b b 1 c c 1 a ) 92
dUcenH b a c c b a a c b 32 .
2(a b c)(

eroberogeday lwm pln

Bit

- Page 26 -

vismPaBeRCIserIs
lMhat;TI6
eK[ x , y , z CabIcMnYnBitviCman .
cUrbgajfa 1 2 1 2
(x y )

(y z )

1
2( x y z )

( z x )2 ( x y )( y z )( x z )

tamvismPaB AM GM eKman
1
1
2

(1)
2
2
( x y )( y z )
(x y )
(y z )
1
1
2

( 2)
2
2
( y z )( z x )
(y z)
(z x)
1
1
2

( 3)
2
2
( z x )( x y )
( z x)
(x y )

bUkvismPaB (1) , (2) nig (3) eK)an


1
1
1
4( x y z )
2

2
2
2 ( x y )( y z )( x z )
(
x

y
)
(
y

z
)
(
z

x
)

dUcenH

1
1
1
2( x y z )

( x y )2 ( y z )2 ( z x )2 ( x y )( y z )( z x)

eroberogeday lwm pln

- Page 27 -

vismPaBeRCIserIs
lMhat;TI7
cUrRsaybBaak;vismPaB
a 2 (1 b )2 b 2 (1 c)2 c2 (1 a )2

cMeBaHRKb;cMnYnBit a , b , c .
tamvismPaB Minkowsky eKman

3 2
2

x12 y12 x 22 y 2 2 x 3 2 y 32 ( x1 x 2 x 3 )2 ( y1 y 2 y 3 )2

yk x1 a , x2 b , x3 c
nig y1 1 b , y 2 1 c , y 3 1 a nig s a b c
eK)an (x1 x2 x3 )2 (y1 y 2 y 3 )2 s2 (3 s)2 92
eRBaH s2 (3 s)2 2s2 6s 9 12 [(2s 3)2 9] 92
dUcenH
a 2 (1 b )2 b 2 (1 c)2 c2 (1 a )2

eroberogeday lwm pln

3 2
2

- Page 28 -

vismPaBeRCIserIs
lMhat;TI8 Junior TST 2002, Romania
ebI a , b , c (0 , 1 ) cUrbgajfa
abc (1 a )(1 b )(1 c) 1

eyIgdwgfacMeBaH x (0 , 1) eKman x 3 x
ehtuenH abc 3 abc (1)
nig (1 a)(1 b)(1 c) 3 (1 a)(1 b)(1 c)
bUkvismPaB (1) & (2) eK)an

( 2)

abc (1 a)(1 b)(1 c) 3 abc 3 (1 a)(1 b)(1 c) ( 3)

tamvismPaB AM GM eKman
abc
( 4)
3

abc

(1 a )(1 b )(1 c)

1 a 1 b 1 c 3 (a b c )

( 5)
3
3

bUkvismPaB (4) & (5) eK)an

(a b c ) 3 (a b c )
3
3
abc 3 (1 a )(1 b )(1 c) 1 (6)
3

abc 3 (1 a )(1 b )(1 c)

tamvismPaB (3) & (6) eK)an


abc (1 a )(1 b )(1 c) 1

eroberogeday lwm pln

Bit .

- Page 29 -

vismPaBeRCIserIs
lMhat;TI9
eK[ a , b , c CabIcMnYnBitviCmanEdl abc 1 .
cUrRsayfa a cb b ac c ba a b
tamvismPaB AM GM eKman a b 2 ab
eK)an a cb 2 abc (1)
RsaydUcKaEdreK)an
bc
ca
bc
ca
( 2) nig
( 3)
2
2
a
b
a
b
bUkvismPaB (1), (2) & (3) eK)an

c3

ab
bc
ca
ab bc ca
) ( 4)

2(

c
a
b
c
a
b
ma:geTot abc cab 2 a (5)
bc
ab
bc
ca

2 b (6) nig

2 c (7)
a
c
a
b
bUkvismPaB (5), (6) & (7) eK)an
2(

ab
bc
ca

) 2( a b c ) a b c 3 (8)
c
a
b

eRBaH a b c 33 abc 3 abc 1


tam (4) & (8) eK)an a cb b ac c ba a
eroberogeday lwm pln

b c 3.
- Page 30 -

vismPaBeRCIserIs
lMhat;TI10
eK[ a , b , c , d CabYncMnYnBitEdl a2 b2 c2 d2 4 .
cUrRsaybBaak;fa a3 b3 c3 d3 8 .
eKman a2 b2 c2 d2 4
eKTaj b2 c2 d2 4 a2 0 b a2 4 b a 2
eday a2 0 enaH a3 2a2 (1)
RsaydUcKaEdr b3 2b2 (2) , c3 2c2 (3) , d3 2d2
bUkvismPaB (1) , (2), (3) & (4) Gg nig GgeK)an
a 3 b 3 c 3 d 3 2(a 2 b 2 c2 d 2 ) 8 Bit
dUcenH a3 b3 c3 d3 8 .

eroberogeday lwm pln

( 4)

- Page 31 -

vismPaBeRCIserIs
lMhat;TI11
eK[ a , b , c CabIcMnYnBitminGviCman .
cUrRsayfa a3 b3 c3 3abc 2 ( b 2 c a)3 .
tamvismPaB AM GM eKman a3 b3 c3 3abc
enaH a3 b3 c3 3abc 0 .
ebI b 2 c a 0 enaHvismPaBxagelIBitCanic .
eyIg]bmafa b 2 c a 0 .

bc
a )3
2
c a 2y enaH b c 2a 2x 2y

tag
yk b a 2x nig
Edl x 0 , y 0 .
eK)an

T a 3 b 3 c3 3abc 2(

T a 3 (a 2x )3 (a 2y )3 3a(a 2x )(a 2y ) 2( x y )3

bnab;BIbRgYmrYceK)an
T 12a( x 2 xy y 2 ) 6( x y )( x y )2 6( x y )( x y )2
bc
bc 2
T 6(
a )(
) 0 Bit
2
2
dUcenH a3 b3 c3 3abc 2 ( b 2 c a)3 .

eroberogeday lwm pln

- Page 32 -

vismPaBeRCIserIs
lMhat;TI12
eK[ a , b , c CabIcMnYnBitviCmanEdl abc 1 .
a b c 5 a2 b 2 c2
cUrbgajfa 3
.
3
a b c 5 a2 b 2 c2
]bmafa 3
Bi
t
3
5
a2 b 2 c2
a b c
smmUl 3
3

smmUl (a b c)5 81(a2 b2 c2 ) 81abc(a2 b2 c2 )

tamvismPaB AM GM eKman
x2 y 2 y 2 z 2 z 2 x2

xy yz zx
2
2
2
x 2 y 2 z 2 xy yz zx
( x y z )2 2( xy yz zx ) xy yz zx
( x y z )2 3( xy yz zx )

yk x ab , y bc , z ca eK)an
(ab bc ca)2 3abc(a b c)

enaH (a2 b2 c2 )(ab bc ca)2 3abc(a b c)(a2 b2 c2 )


edIm,IRsayfa (a b c)5 81abc(a2 b2 c2 ) Bit
eyIgRtUvRsayfa (a b c)6 27(a2 b2 c2 )(ab bc ca)2
eroberogeday lwm pln

- Page 33 -

vismPaBeRCIserIs
tag T (a b c)6 27(a2 b2 c2 )(ab bc ca)2
yk S a b c nig P ab bc ca
enaH a2 b2 c2 (a b c)2 2(ab bc ca) S2 2P
eK)an T S6 27(S2 2P)P 2
S6 27S 2P 2 54P 4
(S 2 3 P )2 (S 2 6 P 2 ) 0

dUcenH

abc

eroberogeday lwm pln

a2 b 2 c2
3

- Page 34 -

vismPaBeRCIserIs
lMhat;TI13
eK[ a , b , c CabIcMnYnBitxusKa .
a2
b2
cUrRsayfa

2
2
(b c )

(c a )

c2
2
2
(a b )

tamsmPaB (x y z )2 x2 y 2 z 2 2xy 2yz 2xz


eKTaj x2 y 2 z 2 (x y z )2 2(xy yz zx)
eday (x y z )2 0 RKb;cMnYnBit x , y , z enaHeK)an
x 2 y 2 z 2 2( xy yz zx ) (*)
yk x b a c , y c b a , z a c b enaHeK)an
ab
bc
ac
xy yz zx

(b c)(c a ) (a b )(c a ) (a b )(b c)


ab(a b ) bc(b c) ac(c a )

(a b )(b c)(c a )

b(c a )(b c a ) ac(c a )


(a b )(b c)(c a )
(c a )(b c)(b a )
1

(a b )(b c)(c a )

tamTMnak;TMng (*) eKTaj)an x2 y 2 z 2 2(1) 2


a2
b2
c2

2 .
dUcenH
2
2
2
(b c )

(c a )

eroberogeday lwm pln

(a b )

- Page 35 -

vismPaBeRCIserIs
lMhat;TI14
eK[ a , b , c CabIcMnYnBitminGviCmanEdl a2 b2 c2 a b c .
cUrRsayfa a2b2 b2c2 c2a2 ab bc ca .
eKman a2 b2 c2 a b c
eK)an (a2 b2 c2 )2 (a b c)2
a4 b4 c4 2(a 2b 2 b 2c2 c2a 2 ) a 2 b 2 c2 2(ab bc ca)

2(ab bc ca a 2b 2 b 2c2 c2a 2 ) a4 b4 c4 a 2 b 2 c2

tamsmPaBenH edIm,Ibgajfa a2b2 b2c2 c2a2 ab bc ca


eyIgRtUvbgajfa a4 b4 c4 a2 b2 c2 .
tamvismPaB Holder eKman
(a13 a23 a33)(b13 b23 b33)(c13 c23 c33) (a1b1c1 a2b2c2 a3b3c3 )3

yk
nig

a1 b1

c1

4
a3

2
a3

, c2

, a2 b2
4
b3

, c3

2
b3

4
c3

, a3 b3

2
c3

eK)an

(a 2 b 2 c 2 )2 (a 4 b 4 c 4 ) (a 2 b 2 c 2 ) 3

naM[ a4 b4 c4 a2 b2 c2 Bit
dUcenH a2b2 b2c2 c2a2 ab bc ca .

eroberogeday lwm pln

- Page 36 -

vismPaBeRCIserIs
lMhat;TI15
eK[ a , b , c CabIcMnYnBitminGviCman . cUrRsayfa
(a 2 ab b 2 )(b 2 bc c2 )(c2 ca a 2 ) (ab bc ca)3

tamvismPaB AM GM eKman x2 y 2 2xy


naM[ x2 y 2 3x2 3y 2 4xy 2xy 3x2 3y 2 4xy
b 4(x2 xy y 2 ) 3(x y )2 RKb;cMnYnBitviCman x nig y .
ehtuenHcMeBaH a , b , c CabIcMnYnBitminGviCman eKTaj)an
3
(a2 ab b2 )(b2 bc c2 )(c2 ca a2 ) ( )3 (a b)2 (b c)2 (c a)2
4

eyIgnwgRsayfa (a b)2 (b c)2 (c a)2 ( 43 )3 (ab bc ca)3


eKmansmPaB
(a b)(b c)(c a) a2b a2c ab2 b2a ac2 bc2 2abc
(a b c)(ab bc ca) a2b a2c ab2 ab2 ac2 bc2 3abc

eFIVpldksmPaBBIrenHeK)an
(a b c)(ab bc ca) (a b )(b c)(c a ) abc (1)
a b 2 ab

tamvismPaB AM GM eKman b c 2 bc

c a 2 ac
eK)an (a b)(b c)(c a) 8abc (2)

eroberogeday lwm pln

- Page 37 -

vismPaBeRCIserIs
yk (1) CMnYskg (2) eK)an
9(a b )(b c)(c a ) 8(a b c)(ab bc ca )
b (a b)2 (b c)2 (a c)2 64
(a b c)2 (ab bc ca)2 ( 3)
81
tamvismPaB AM GM eKman

a2 b 2 b 2 c2 c2 a2

ab bc ac
2
2
2
b a2 b2 c2 ab bc ca

b (a b c)2 3(ab bc ca)


tam (3) nig (4) eKTaj)an

( 4)

4
(a b )2 (b c)2 (c a )2 ( )3 (ab bc ca)3
3

Bit

dUcenH

(a 2 ab b 2 )(b 2 bc c2 )(c2 ca a 2 ) (ab bc ca)3

eroberogeday lwm pln

- Page 38 -

vismPaBeRCIserIs
lMhat;TI16
eK[ a , b , c CabIcMnYnBitminGviCmanEdl abc 1 . cUrRsayfa
c a3 b 3 a b 3 c3 b c3 a3
3

2
2
2
2
2
2
2
a b
b c
c a
c a3 b 3 a b 3 c3 b c3 a3
tag T 2 2 2 2 2 2
a b
b c
c a
tamvismPaB Cauchy Schwarz eKman

(a 2 b 2 ) 2

1 3
(a 2 .a 2

eKTaj a2 b2

1 3
b 2 .b 2 )2

(a b )(a 3 b 3 )

a b . a3 b3

c a3 b3
b 2 2 ac b (1) . RsaydUcKaEdreK)an
a b
a b 3 c3
a
b c3 a3
b

(
2
)
ni
g
( 3)
2
2
2
2
bc
ca
b c
c a
bUkvismPaB (1), (2) & (3) eK)an
a
b
c

T
bc
ca
ab
tamvismPaB Cauchy Schwarz eK)an
2

b
c
a2
b2
c2
a

) ( 4)

3(
ca
ab
bc ca ab
bc

eday a b c

b
c
a
. bc
. ca
ab
bc
ca
ab

eroberogeday lwm pln

- Page 39 -

vismPaBeRCIserIs
eK)an
a2
b2
c2
(a b c) [(b c) (c a ) (a b )](

)
bc ca ab
a2
b2
c2
2
(a b c) 2(a b c)(

)
bc ca ab
a2
b2
c2
a b c 33 abc 3
eKTaj b c c a a b 2 2 2 (5)
2
b
c
a
tam (4) & (5) eK)an b c c a a b 92

naM[ ba c cb a ac b 32 enaH T 32
c a3 b 3 a b 3 c3 b c3 a3
dUcenH 2 2 2 2 2 2 32 .
a b
b c
c a
2

eroberogeday lwm pln

- Page 40 -

vismPaBeRCIserIs
lMhat;TI17
eK[ a , b , c CabIcMnYnBitviCman . cUrRsaybBaak;vismPaB
a3
b3
c3

3
1
3
3
3
3
3
a (b c )
b (c a )
c (a b )

a3
b3
c3

3
tag S 3
3
3
3
a (b c )
b (c a )
c (a b )3
yk x b a c , y c b a , z a c b
eK)an S 1 3 1 3 1 3
1 x
1 y
1 z

tamvismPaB AM GM RKb; u 0 eKman


2

1 x 1 x x2
3
2
(1 1 x 2 )2
1 x (1 x )(1 x x )

2
2

1
1
1

(b c )2
1 x3 1 1 x2
1
2
2a 2

eKTaj)an

(b c ) 2
2(b 2 c2 ) a 2 b 2 c2
eday 1 2 1

2
2a
2a
a2
1
a2
eKTaj)an
2
3
a b 2 c2
1 x
a2
b2
c2
dUcenH S 2 2 2 2 2 2 2 2 2 1
a b c
a b c
a b c

eroberogeday lwm pln

- Page 41 -

vismPaBeRCIserIs
lMhat;TI18
cMeBaHRKb;cMnYnBitviCman a , b , c cUrRsayfa
a
b
c
3

ab
bc
ca
2
yk S a a b b b c c c a
tag x ba , y bc , z ac ehIy xyz 1
eK)an S 1 2 1 2 1 2
1 x
1 y
1 z

]bmafa x y z eK)an x3 xyz 1 b x 1 ehIy yz 1x 1


tamvismPaB Cauchy Schwarz eKman
(

1
1 y2

1
1 z2

)2 2(

1
1
)

2
2
1 y
1 z

eday

1
1
2 y2 z2
1 y 2z 2

1
2
2
2
2
2 2
1 y
1 z
1 y z z y
1 y 2 z 2 y 2z 2

ehIy

1 z 2y 2
(1 yz )(1 yz ) 1 yz

2
2
2 2
2 2
1 yz
1 y z y z
1 2yz y z
2

1
1
1 yz
4

eK)an

2(1
)
2
2
1 yz 1 yz

1
y
1
z

eroberogeday lwm pln

- Page 42 -

vismPaBeRCIserIs
eKTaj)an

1
1 y2
1

2
1 yz

1 z2
2
ehtuenH S

1 yz
1 x2
eyIgnwgRsayfa 1 2 1 2 yz 32
1 x

eKman (1 x)2 2(1 x2 ) b 1 x 2 .


enaH 1 2 1 2x 21 1 2yz
yz
1 x

naM[
ehIy

eKTaj
dUcenH

1 x2

yz
1
2
2yz
2

1 yz 1 yz
1 yz
1 x2
3(1 yz ) 2yz 2 2 1 yz
3
2yz
2

2 1 yz
1 yz
(1 yz ) 2
3 yz 2 2 1 yz

(1 yz ) 2
( 2 1 yz )2

0
(1 yz ) 2
1
2
2yz
2
3

1 yz 1 yz
1 yz
2
1 x2
a
b
c
3
.

ab
bc
ca
2

eroberogeday lwm pln

Bit

- Page 43 -

vismPaBeRCIserIs
lMhat;TI19
cMeBaHRKb;cMnYnBitminGviCman x , y , z cUrRsayfa
2(1 x 2 )(1 y 2 )(1 z 2 ) (1 x )(1 y )(1 z )(1 xyz )

ebI y z x enaHvismPaBeTACa 2(1 x2 )3 (1 x)3 (1 x3 )


smmUl 2(1 x2 )3 (1 x)3 (1 x3 ) 0
smmUl (1 x)4 (1 x x2 ) 0 Bit
ehtuenHeK)an

2(1 x 2 )3 (1 x )3 (1 x 3 )

2 3
3
3
2(1 y ) (1 y ) (1 y )

2 3
3
3
2(1 z ) (1 z ) (1 z )

KuNvismPaBenHGgnigGgeK)an

[2(1 x 2 )(1 y 2 )(1 z 2 )]3 [(1 x )(1 y )(1 z )]3 P

b 2(1 x2 )(1 y 2 )(1 z 2 ) (1 x)(1 y )(1 z ) 3 P (*)


Edl P (1 x3 )(1 y 3 )(1 z 3 )
1 ( x3 y 3 z 3 ) ( x3y 3 y 3z 3 z 3x3 ) x3y 3z 3

tamvismPaB AM GM eK)an
P 1 33 x 3y 3z 3 33 x6y 6z 6 x 3y 3z 3 (1 xyz )3

tamTMnak;TMng (*) eK)an


2(1 x 2 )(1 y 2 )(1 z 2 ) (1 x )(1 y )(1 z )(1 xyz )

eroberogeday lwm pln

Bit.

- Page 44 -

vismPaBeRCIserIs
lMhat;TI20
cMeBaHRKb;cMnYnBitminGviCman a, b, c, d cUrRsayfa
4(1 a a 2 )(1 b b 2 )(1 c c 2 )(1 d d 2 ) (1 abcd )2

ebI d c b a vismPaBkayCa 4(1 a a2 )4 (1 a4 )2


smmUl 2(1 a a2 )2 1 a4
smmUl 2(1 a a2 )2 (1 a4 ) (1 a)4 0 Bit
2(1 a a 2 )2 1 a4
ehtuenH
2(1 b b 2 )2 1 b 4
eK)an 4(1 a a2 )2 (1 b b2 )2 (1 a4 )(1 b4 )

eday (1 a4 )(1 b4 ) 1 (a4 b4 ) a4b4 1 2a2b2 a4b4


b (1 a4 )(1 b4 ) (1 a2b2 )2
eKTaj 4(1 a a2 )2 (1 b b2 )2 (1 a2b2 )2
b 2(1 a a2 )(1 b b2 ) 1 a2b2 (1)
dUcKaEdr 2(1 c c2 )(1 d d2 ) 1 c2d2 (2)
ehIy (1 a2b2 )(1 c2d2 ) (1 abcd)2
eFIVplKuNvismPaB (1) nig (2) eK)an
4(1 a a 2 )(1 b b 2 )(1 c c2 )(1 d d 2 ) (1 abcd )2

eroberogeday lwm pln

- Page 45 -

vismPaBeRCIserIs
lMhat;TI21 ( Tournament of the Towns,1993)
ebIsmIkar x4 ax3 2x2 bx 1 0 manbsy:agticmYyCacMnYnBit
enaHbgajfa a4 b4 8 .
tag x0 IR Edl x0 0 Cabsrbs;smIkarenaHeK)an
x04 ax0 3 2x0 2 bx0 1 0 b ax0 3 bx0 ( x0 2 1)2
tamvismPaB Cauchy Schwarz eK)an
( x0 1)4 (ax0 3 bx 0 )2 ( x06 x0 2 )(a 2 b 2 )

eKTaj

a b
2

eyIgnwgRsayfa

( x0 2 1)4
x06 x 0 2

( x0 2 1)4
x0 x0
6

eKman (x02 1)4 8(x06 x02 ) x04 4x03 6x02 4x0 1


b (x02 1)4 8(x06 x02 ) (x0 1)4 0
( x0 2 1)4
eKTaj)an 6 2 8 Bit
x0 x0

dUcenH a4 b4 8 .

eroberogeday lwm pln

- Page 46 -

vismPaBeRCIserIs
lMhat;TI22
cUrRsaybBaak;fa
a
b
c
9

(b c)2 (c a )2 (a b )2 4(a b c)

cMeBaHRKb; a , b , c CacMnYnBitviCman .
tamvismPaB Cauchy Schwarz
(a12 a 2 2 a 3 2 )(b12 b 2 2 b 3 2 ) (a1b1 a 2b 2 a 3b 3 )2
yk a1 b ac , a2 c ba , a3 a cb
nig b1 a , b2 b , b3 c eK)an
2
a
b
c
b
c
a
(a b c )

2
2
2
b
c
c
a
a
b

(
b
c
)
(
c
a
)
(
a
b
)

eday b a c c b a a c b 32 emIllMhat;TI5
eKTaj

a
b
c
9

(
a
b
c
)

2
2
2
4
(b c ) (c a ) (a b )

dUcenH

a
b
c
9

(b c)2 (c a )2 (a b )2 4(a b c)

eroberogeday lwm pln

- Page 47 -

vismPaBeRCIserIs
lMhat;TI23 ( JBMO 2002 Shortlist)
a3 b 3 c3 a2 b 2 c2
cUrRsaybBaak;fa 2 2 2 b c a
b
c
a
cMeBaHRKb; a , b , c CacMnYnBitviCman .
tamvismPaB Cauchy Schwarz
(a12 a 2 2 a 3 2 )(b12 b 2 2 b 3 2 ) (a1b1 a 2b 2 a 3b 3 )2

yk a1 a , a2 b , a3 c
nig b1 a ba , b2 b c b , b3 c a c eK)an

a3 b 3 c3 a2 b 2 c2
(a b c) 2 2 2

c
a
c
a b
b
ebIeKyk a1 ab , a2 bc , a3 ca
nig b1 b , b2 c , b3 a eK)an

(1)

a2 b 2 c2

(a b c ) (a b c ) 2 ( 2 )

b
c
a

KuNvismPaB (1) nig (2) Gg nig Gg rYcsRmYlkta (a b c)2


a2 b 2 c2
a3 b 3 c3 a2 b 2 c2
nig b c a eK)an 2 2 2 b c a Bit
b
c
a
a3 b 3 c3 a2 b 2 c2
dUcenH 2 2 2 b c a .
b
c
a

eroberogeday lwm pln

- Page 48 -

vismPaBeRCIserIs
lMhat;TI24 ( Crux Mathematicorum)
eK[ x , y , z CabIcMnYnBitviCman . cUrRsaybBaak;fa
x
y
z

1
x ( x y )( x z ) y ( y z )( y x ) z ( z x )( z y )

eKman (x y )(x z ) x2 xy yz xz xy 2x

yz zx

( x y )( x z ) ( xy xz )2
( x y )( x z ) xy xz
x ( x y )( x z ) x xy xz x ( x y z )

eKTaj x x xy )(x z ) x
RsaydUcKaEdr y (y yz )(y x)
nig z

x
(1)
y z
y
( 2)
x y z

z
z

( 3)
( z x )( z y )
x y z

eFIVplbUkvismPaB (1), (2) & (3) eK)an


x
y
z

1
x (x y)(x z) y (y z)(y x) z (z x)(z y)

eroberogeday lwm pln

- Page 49 -

vismPaBeRCIserIs
lMhat;TI25
RKb;cMnYnBitviCman a , b , c cUrRsaybBaak;fa

bc ca ab
b
c
a

a
b
c
b c c a a b
tamvismPaB AM GM RKb; x 0 , y 0 eKman
xy
xy
2
b (x y )2 4xy
y 1 1

b x 4 y xxy
x y
yk x ba nig y ac eK)an b4ac ba ac (1)
dUcKaEdr c4ba bc ba (2) nig a 4cb ac bc (3)
eFIVplbUkvismPaB (1), (2) & (3) eK)an
4a
4b
4c
ab bc ca

bc ca ab
c
a
b
dUcenH b a c c b a a c b 4 b a c c b a a c b

eroberogeday lwm pln

- Page 50 -

vismPaBeRCIserIs
lMhat;TI26 ( MOSP,2001)
eK[ a , b , c CabIcMnYnBitviCmanEdl abc 1 .
cUrRsayfa (a b)(b c)(c a) 4(a b c 1)
man (a b c)(ab bc ca) (a b)(b c)(c a) abc (1)
tamvismPaB AM GM eKman
(a b )(b c)(a c) 2 ab .2 bc .2 ac 8abc ( 2)

yk (1) CMnYskg (2) eK)an


8
(a b )(b c)(c a ) (a b c)(ab bc ca)
9
edIm,Ibgajfa (a b)(b c)(c a) 4(a b c 1)
eyIgRtUvRsayfa 89 (a b c)(ab bc ca) 4(a b c 1) Bit
b 92 .(ab bc ca) a b1 c 1 Bit
tamvismPaB AM GM eK)an
ab bc ca ab bc c
1
(ab bc ca)2
3

3
9
9
abc
81(a b c)

2
2
1
(
ab

bc

ca
)
(ab bc ca)
3
1 Bit
9
abc
3(a b c)

eRBaH (ab bc ca)2 3abc(a b c) 3(a b c) .


dUcenH (a b)(b c)(c a) 4(a b c 1) .
eroberogeday lwm pln

- Page 51 -

vismPaBeRCIserIs
lMhat;TI27 ( APMO,2004)
cUrbgajfa (a2 2)(b2 2)(c2 2) 9(ab bc ca)
cMeBaHRKb;cMnYnBitviCman a , b , c .
eKman (a b c)2 3(ab bc ca)
edIm,Ibgajfa (a2 2)(b2 2)(c2 2) 9(ab bc ca)
eyIgRtUvbgajfa (a2 2)(b2 2)(c2 2) 3(a b c)2
u2 2
u2 1
eKman 3 1 3 enaHvismPaBGacsresr
a2 1
b2 1
c 2 1 (a b c )2
(1
)(1
)(1
)
3
3
3
9
tamsmPaB (1 x)(1 y ) 1 x y xy

ebI x nig y mansBaadUcKaenaH xy 0


ehtuenH (1 x)(1 y ) 1 x y RKb;cMnYnBit x , y .
-ebI x 0 , y 0 , z 0 b x 0 , y 0 , z 0 eK)an
(1 x )(1 y )(1 z ) (1 x y )(1 z ) 1 x y z (1)

-ebI 1 x 0 , 1 y 0 , 1 z 0 enaH x y nig z


mansBaadUcKa ehIy (1 x y )(1 z ) 1 x y z
eK)an (1 x)(1 y )(1 z ) 1 x y z
eyIgsikSavismPaBtambIkrNIdUcxageRkam
eroberogeday lwm pln

- Page 52 -

vismPaBeRCIserIs
a2 1
b2 1
c2 1
0,
0,
0
3
3
3

!-ebI a , b , c 1 enaH
tamlkN (1) eK)an

b2 1
c2 1
a2 b 2 c2 3
a2 1
(1
)(1
)(1
) 1
3
3
3
3
b2 1
c2 1 a2 b 2 c2
a2 1
(1
)(1
)(1
)
3
3
3
3
a 2 b 2 c 2 (a b c )2

eday

Cauchy Schwarz
3
9
a2 1
b2 1
c 2 1 (a b c )2
eK)an (1 3 )(1 3 )(1 3 ) 9
Bit
a2 1
b2 1
c2 1
@-ebI 0 a , b , c 1 enaH 3 0, 3 0 , 3 0
tamlkNH (2) eK)an
a2 1
b2 1
c2 1
a2 b 2 c2 3
(1
)(1
)(1
) 1
3
3
3
3
c 2 1 a 2 b 2 c 2 (a b c )2
b2 1
a2 1

)
)(1
)(1
(1
3
9
3
3
3

Bit

#-ebI a , b , c manBIrEdlFMCag 1 enaHeyIgsntfa a 1 , b 1 , c 1


a2 1
b2 1
a2 b2 2 1 a2 b2
eK)an (1 3 )(1 3 ) 1 3 3
a2 1
b2 1
c2 1 (1 a 2 b 2 )(c2 2)
enaH (1 3 )(1 3 )(1 3 )
9

eroberogeday lwm pln

- Page 53 -

vismPaBeRCIserIs
eday (1 a2 b2 )(c2 1 1) (a b c)2
b2 1
c 2 1 (a b c )2
a2 1
eK)an (1 3 )(1 3 )(1 3 ) 9
Bit .
$-cMeBaHkrNI EdlRtIFatu a , b , c manEtmYyKt;EdlFMCag 1 sikSadUc
krNITI! .
srubcMeBaHRKb;cMnYnBitviCman a , b , c eK)an
(a 2 2)(b 2 2)(c 2 2) 9(ab bc ca ) .

eroberogeday lwm pln

- Page 54 -

vismPaBeRCIserIs
lMhat;TI28 ( Japan,1997)
eKyk a , b , c CabIcMnYnBitviCman . cUrbgajfa
(b c a )2
(c a b )2
(a b c )2 3

2
2
2
2
2
2
5
(b c ) a
(c a ) b
(a b ) c
(b c a )2
(c a b ) 2
(a b c ) 2

tag T
2
2
2
2
(b c ) a
(c a ) b
(a b )2 c 2
bc
ca
ab
yk x a , y b , z c
eK)an 1 1 x 1 1 y 1 1 z aa bb cc 1 ehIykenSam T eTACa
( x 1)2 ( y 1)2 ( z 1)2
2
2
.
T 2
x 1
y 1
z 1
ebI x y z 2 eK)an T 53 CakrNIEdlvismPaBkayCasmPaB .
( x 1)2
eyIgsntfamanBIrcMnYnBit a nig b EdlnaM[ 2 x a 1 b Bit
x 1
RKb; x 0 ehIyeday x 1 1 y 1 1 z 1 1 1 enaHedIm,I[ T 53
luHRtaEt ( x a 1 b) ( y a 1 b) ( z a 1 b) 53
naM[ a 3b 53 (*)

ma:geTotedaysmPaBekIteLIgcMeBaH x 2 enaH x 2 CabsDb


eroberogeday lwm pln

- Page 55 -

vismPaBeRCIserIs
( x 1)2
rbs;smIkar 2 x a 1 b
x 1
b (x 1)(x 1)2 (x2 1)(a b bx) 0

tag f (x) (x 1)(x 1)2 (x2 1)(a b bx)


x 2 CabsDbn f ( x ) 0 enaH f ' ( 2) 0
eday f ' (x) (x 1)2 2(x2 1) 2x(a b bx) b(x2 1)
enaH f ' (2) 1 6 4(a 3b) 5b 7 4 53 5b 0
3
54
eKTaj)an b 23
ehI
y
tam
(*) eK)an a 3b
25
5
25

eK)an

( x 1)2 23
54

x 2 1 25 25( x 1)

smmUl

2( x 2)2 ( x 7 )
0 Bit
2
25( x 1)( x 1)

( x 1)2 23
ehtuenH 2 25 25(54
(1)
x
1
)

x 1
( y 1)2 23
54
( z 1)2 23
54

(
2
)
ni
g
( 3)
2
2
y 1 25 25( y 1)
z 1 25 25( z 1)

bUkvismPaB (1) , (2) & (3) eK)an


T

23
54 1
1
1
69 54 15 3
)
3 (

25
25 x 1 y 1 z 1
25
25 5

dUcenH

(b c a )2
(c a b )2
(a b c )2 3

2
2
2
2
2
2
5
(b c ) a
(c a ) b
(a b ) c

eroberogeday lwm pln

Bit

- Page 56 -

vismPaBeRCIserIs
lMhat;TI29
eK[ x , y , z , t CabYncMnYnBitviCman . cUrRsayfa
2

x y z t



1 .

x y y z z t t x
tag a yx , b yz , c zt nig d xt Edl abcd 1
vismPaBsmmUl 1 2 1 2 1 2 1 2 1
(1 a )
(1 b )
(1 c)
(1 d )

tamvismPaB Cauchy Schwarz eKman


1
1
1
ab(a b )2 (1 ab )2

0
2
2 1 ab
2
2
(1 a )
(1 b )
(1 a ) (1 b ) (1 ab )
eKTaj 1 2 1 2 1 1ab (1)
(1 a )
(1 b )
RsaydUcKaEdr 1 2 1 2 1 1cd 1 abab (2)
(1 c)
(1 d )

bUkvismPaB (1) & (2) eK)an


1
1
1
1
1
ab

1
2
2
2
2
1 ab 1 ab
(1 a )
(1 b )
(1 c)
(1 d )
2

dUcenH x x y y y z z z t t t x 1

eroberogeday lwm pln

- Page 57 -

vismPaBeRCIserIs
lMhat;TI30
eK[ a , b , c nig x , y , z CacMnYnBit . cUrRsaybBaak;fa
4(a 2 x 2 )(b 2 y 2 )(c 2 z 2 ) 3(bcx cay abz )2

tamvismPaB Cauchy Schwarz eKman


[x(bc ) a(cy bz )]2 ( x 2 a 2 )[b 2c 2 (cy bz )2 ]

b (a2 x2 )[b2c2 (cy bz )2 ] (bcx acy abz )2


edIm,IRsayfa 4(a2 x2 )(b2 y 2 )(c2 z 2 ) 3(bcx cay abz)2
eyIgRKan;EtRsayfa 4(b2 x2 )(c2 z 2 ) 3[b2c2 (cy bz )2 ]
b 4(b2 x2 )(c2 z 2 ) 3[b2c2 (cy bz )2 ] 0
b b2c2 b2z 2 c2y 2 4y 2z 2 2bcyz 4bcyz 0
b (cy bz )2 (bc 2yz )2 0 Bit
dUcenH 4(a2 x2 )(b2 y 2 )(c2 z 2 ) 3(bcx cay abz )2

eroberogeday lwm pln

- Page 58 -

vismPaBeRCIserIs
lMhat;TI31
eK[bIcMnYnBit a 1 , b 1 , c 1 . cUrRsaybBaak;fa
1 a2
1 b2
1 c2
2 .

2
2
2
1 c a
1 a b
1 a2
1 b2
1 c2
yk T

2
2
1 b c 1 c a
1 a b2
eKman 1 b 0 nig c2 0 enaH 1 b c2 1 b 0
1 b c

1 b2
ehIy (1 b) 0 b b 2
2
1 b 2 2(1 c 2 )
2 1 b
2
naM[ 1 b c 2 1 c
2
1 a2
2(1 a 2 )

eKTaj)an
(1)
2
2
2
1 b c
1 b 2(1 c )
2

RsaydUcKa

1 b2
2(1 b 2 )

( 2)
2
2
2
1 c a
1 c 2(1 a )

nig

1 c2
2(1 c2 )

( 3)
2
2
2
1 a b
1 a 2(1 b )

bUkvismPaB (1) , (2) nig (3) Gg nig GgeK)an


T

2x
2y
2z

y 2z z 2x x 2y

Edl x 1 a2 , y 1 b2 , z 1 c2 .
eroberogeday lwm pln

- Page 59 -

vismPaBeRCIserIs
tamvismPaB Cauchy Schwarz eK)an
2x 2
2y 2
2z 2
2( x y z )2
T

xy 2xz yz 2xy xz 2yz 3( xy yz zx )

eday (x y z )2 3(xy yz zx) enaH T 2


1 a2
1 b2
1 c2
dUcenH
2 .

2
2
2
1 b c

1 c a

eroberogeday lwm pln

1 a b

- Page 60 -

vismPaBeRCIserIs
lMhat;TI32
eK[bIcMnYnBitviCman a , b , c Edl abc 1 . cUrRsaybBaak;fa
1
1
1
3

.
3
3
3
8
(1 a )

tag

(1 b )
(1 c)
1
1
1

T
(1 a )3 (1 b )3 (1 c)3

tamvismPaB Holder
(a13 a23 a33)(b13 b23 b33)(c13 c23 c33) (a1b1c1 a2b2c2 a3b3c3)3
yk a1 b1 1 1 a , a2 b2 1 1 b , a3 b3 1 1 c
ehIy c1 c2 c3 1 eK)an

1
1
1
3 T2

2
(1 b )2 (1 c)2
(1 a )

1
1
1
b

2
2
2
(
1
a
)
(
1
b
)
(
1
c
)

edIm,IRsayfa T 83 eyIgRtUvRsayfa
3
2
1 1
1
1
9
3

3 (1 a )2 (1 b )2 (1 c)2
64
8
b 1 2 1 2 1 2 43
(1 a )
(1 b )
(1 c)
1
T2
3

eroberogeday lwm pln

- Page 61 -

vismPaBeRCIserIs
eKman
1
1
1
ab(a b )2 (1 ab )2

0
2
2 1 ab
2
2
(1 a )
(1 b )
(1 a ) (1 b ) (1 ab )
eKTaj)an 1 2 1 2 1 1ab
(1 a )
(1 b )
ehIy 1 2 1 2 1 2 1 1ab 1 2
(1 a )
(1 b )
(1 c)
(1 c)
eyIgnwgRsayfa 1 1ab 1 2 43 eday abc 1
(1 c)
1
1
3
c
1
3 (c 1)2
eK)an 1 ab


0
2
2
4 1 c (1 c)
4 4(1 c)
(1 c)
eKTaj 1 1ab 1 2 43 Bit
(1 c)
dUcenH 1 3 1 3 1 3 83 .
(1 a )
(1 b )
(1 c)

eroberogeday lwm pln

- Page 62 -

vismPaBeRCIserIs
lMhat;TI33 USAMO 2011
eK[bIcMnYnBitviCman a , b , c Edl a2 b2 c2 (a b c)2 4 .
cUrRsaybBaak;fa ab 12 bc 12 ca 12 3 .
(a b )

(b c )

(c a )

eKman a2 b2 c2 (a b c)2 4
2(a 2 b 2 c 2 ab bc ca) 4
a 2 b 2 c 2 ab bc ca 2

eK)an 2ab a2 b2 (c2 bc) (ab ca) 2 2ab


(a b )2 (b c)(c a ) 2(ab 1)
eKTaj ab 12 12 (b c)(c 2a) (1)
(a b )
2(a b )
RsaydUcKaEdreK)an bc 12 12 (a b)(a 2 c) (2)
(b c )
2(b c)
nig ca 12 12 (a b)(b 2 c) (3)
(c a )
2(c a )
bUk (1), (2) & (3) eK)an ab 12 bc 12 ca 12 32 T
(a b )
(b c )
(c a )
Edl T (b c)(c 2a) (a b)(a 2 c) (a b)(b 2 c) 32
2(a b )
2(b c)
2(c a )
dUcenH ab 12 bc 12 ca 12 3 .
(a b )
(b c )
(c a )

eroberogeday lwm pln

- Page 63 -

vismPaBeRCIserIs
lMhat;TI34 USAMO 2004
eK[bIcMnYnBitviCman a , b nig c .
cUrRsayfa (a5 a2 3)(b5 b2 3)(c5 c2 3) (a b c)3
cMeBaHRKb; x 0 eyIgnwgRsayfa x5 x2 3 x3 2 (*)
eK)an x5 x3 x2 1 (x2 1)(x3 1) 0 Bit
tag P (a5 a2 3)(b5 b2 3)(c5 c2 3)
tamvismPaB (*) eK)an P (a3 2)(b3 2)(c3 2)
edIm,IRsayfa P (a b c)3 eyIgRKan;EtRsay[eXIjfa
(a 3 2)(b 3 2)(c 3 2) (a b c)3
t3 2
t3 1
eday 3 1 3 enaHvismPaBGacsresrCa
a3 1
b3 1
c 3 1 (a b c )3
(1
)(1
)(1
)
3
3
3
27
eKmansmPaB (1 u)(1 v ) 1 u v uv

ebI u nig v CaBIrcMnYnBitmansBadUcKaenaHplKuN uv 0


ehtuenH (1 u)(1 v ) 1 u v
cMeBaH w 1 eK)an (1 u)(1 v)(1 w ) (1 u v)(1 w )
-ebI u , v , w 0 enaH (1 u v)(1 w ) 1 u v w
eKTaj (1 u)(1 v )(1 w ) 1 u v w (**)
eroberogeday lwm pln

- Page 64 -

vismPaBeRCIserIs
-ebI u , v , w 0 enaH (1 u v )(1 w ) 1 u v w
eKTaj (1 u)(1 v )(1 w ) 1 u v w (* * *)
a3 1
b3 1
c 3 1 (a b c )3
edIm,IRsayfa (1 3 )(1 3 )(1 3 ) 27 Bit
eyIgRtUvsikSakrNIdUcxageRkam
-krNITI1 ebI a , b , c 1 b a , b , c 1 enaHtam (**) nig (* * *)
eKGacsresr
a3 1
b3 1
c3 1
a3 b 3 c3 3
(1
)(1
)(1
) 1
3
3
3
3
a3 1
b3 1
c3 1 a3 b 3 c3
(1
)(1
)(1
)
3
3
3
3
tamvismPaB Hoder eKman 9(a3 b3 c3 ) (a b c)3
a3 1
b3 1
c 3 1 (a b c ) 3
(1
)(1
)(1
)
3
3
3
27
a,b ,c
1
1

eKTaj)an
Bit
manmYytUcCag nig BIrcMnYneTotFMCag
-krNITI2 ebIbIcMnYn
sntfa c 1 nig a , b 1 enaHeK)an
a3 1
b3 1
a3 b 3 2 a3 b3 1

(1
)(1
) 1
3
3
3
3
c3 1 c3 2 1 1 c3
ehIy 1 3 3 3 enaHtamvismPaB Hoder
a 3 b 3 1 1 1 c 3 1 1 1 (a b c )3

.
.
3
3
3
27

eroberogeday lwm pln

- Page 65 -

vismPaBeRCIserIs
a3 1
b3 1
c 3 1 (a b c ) 3
(1
)(1
)(1
)
3
3
3
27
(a5 a 2 3)(b 5 b 2 3)(c5 c 2 3) (a b c)3

ehtuenH
srubmk
cMeBaHRKb;cMnYnBitviCman a , b , c .

eroberogeday lwm pln

Bit

- Page 66 -

vismPaBeRCIserIs
lMhat;TI35 USAMO 1997
cUrRsayfacMeBaHRKb;cMnYnBitviCman a , b , c eKmanvismPaB
1
1
1
1

a 3 b 3 abc b 3 c 3 abc c 3 a 3 abc abc


eKman (a b)(a2 b2 ) 0 RKb;cMnYnBitviCman a nig b

eK)an a3 b3 a2b ab2


a 3 b 3 abc a 2b ab 2 abc
a 3 b 3 abc ab(a b c)
1
1
c
eKTaj 3 13
.
(1)

a b abc ab(a b c) abc a b c


1
a
.

( 2)
RsaydUcKaEdr 3 13
b c abc abc a b c
1
b
.

nig 3 31
( 3)
c a abc abc a b c

bUkvismPaB (1), (2) & (3) GgnigGgeK)an

1
1
1
1

a 3 b 3 abc b 3 c 3 abc c 3 a 3 abc abc

eroberogeday lwm pln

- Page 67 -

vismPaBeRCIserIs
lMhat;TI36 USAMO 1991
mm 1 nn 1
eKyk a m n Edl m nig n CacMnYnKt;viCman .
m n
cUrRsayfa am an mm nn .
a
a
m m 1 nn 1
eyIgeRCIserIs x m 1 nig y n 1 eday a m n
m n

n
m
1
1
n
m
m
n
enaH x n . m n nig y m . m n
m n
m n
tamvismPaB Bernoullie eKman (1 x)m 1 mx
eK)an (1 ma 1)m 1 nn . nm m n
m n
naM[ am mm mmnn . nm m n (1)
m n
ehIy (1 y )n 1 ny
eK)an (1 na 1)n 1 mm . mm n n
m n
naM[ an nn mmnn . mm n n (2)
m n
bUkvismPaB (1) & (2) eK)an am an mm nn Bit

dUcenH

am an m m n n

eroberogeday lwm pln

- Page 68 -

vismPaBeRCIserIs
lMhat;TI37
eK[ x ; y ; z CabIcMnYnBitviCmanEdl xyz 1 .
cUrRsaybBaak;fa
x9 y 9
x x y y
6

3 3

y9 z9
y y z z
6

3 3

z 9 x9
z z x x
6

3 3

RsaybBaak;fa
x9 y 9
y9 z9
z 9 x9
6
6
2
6
3 3
6
3 3
6
3 3
6
x x y y
y y z z
z z x x

eyIgman

x 9 y 9 ( x 3 y 3 )( x 6 x 3 y 3 y 6 )

x 9 y 9 ( x 3 y 3 )( x 6 x 3 y 3 y 6 ) 2 x 3 y 3 ( x 3 y 3 )
2 x3 y3 ( x3 y3 )
x9 y9
3
3
x y 6
x6 x3 y3 y6
x x3 y3 y6

eKTaj
tamvismPaB AM GM
eKman x x y y 3 x y
eK)an 2xx y x( xy yy ) 23 ( x
6

eKTaj
b

y3 )

2 3
x9 y9
3
3

( x y3 )
6
3 3
6
3
x x y y
1 3
x9 y9

( x y 3 ) (1)
6
3 3
6
3
x x y y
1 3
y9 z9

( y z 3 ) ( 2)
6
3 3
6
3
y y z z

RsaydUcKaEdreK)an

eroberogeday lwm pln

- Page 69 -

vismPaBeRCIserIs
z9 x9
1 3

( z x 3 ) ( 3)
6
3 3
6
z z x x
3

eFIVplbUk (1) ; (2) nig (3) Gg nig Gg eK)an


x9 y9
y9 z9
z9 x9
2 3

( x y3 z3 )
6
3 3
6
6
3 3
6
6
3 3
6
x x y y
y y z z
z z x x
3

tamvismPaB AM GM
eK)an x y z 3 xyz 3 eRBaH xyz 1
dUcenH x xx y y y y y y zz z z zz x x x
3

eroberogeday lwm pln

3 3

- Page 70 -

vismPaBeRCIserIs
lMhat;TI38
eK[ a , b , c CacMnYnviCmanEdl ab bc ca 3 .
1
cUrbgajfa 1 a 1(b c) 1 b 1(c a) 1 c 1(a b) abc
2

tamvismPaB AM GM eKman
ab bc ca
1
(abc ) b abc 1 .
3
eKman
3

1
1
1
1

1 a 2 (b c) 1 a(ab ac) 1 a( 3 bc ) 3a (1 abc)


1
1
abc 1 1 abc 0

(1)
2
1 a (b c) 3a
1
1
1
1

(
2
)
,

( 3)
2
2
1 b (c a ) 3b
1 c (a b ) 3c

eday
b
enaH
dUcKaEdr
bUkvismPaB (1) ; (2) nig (3) eK)an

1
1
1
1
1
1

1 a 2 (b c) 1 b 2 (c a ) 1 c 2 (a b ) 3a 3b 3c

ab bc ca
3abc

eday ab bc ca 3
1
dUcenH 1 a 1(b c) 1 b 1(c a) 1 c 1(a b) abc
.
2

eroberogeday lwm pln

- Page 71 -

vismPaBeRCIserIs
lMhat;TI39 (Austrian Olympiad 1989)

eK[ a nig b CaBIrcMnYnBitminGviCmanEdl a b 4 .


cUrRsayfa a abb 2 2 1 .
eyIgman (a b) a b 2ab 4 2ab
eKTaj 2ab (a b) 4 (a b 2)(a b 2)
naM[ a abb 2 a b2 2
2

ab
ab

1
ab2
2

(1)

eKman (a b) (a b) 2(a b )
eKTaj (a b) 2(a b ) (a b) 2(a b ) 8
naM[ a b 2 2 b a 2 b 2 (2)
tam (1) nig (2) eKTaj)an a abb 2 2 1 .
2

eroberogeday lwm pln

- Page 72 -

vismPaBeRCIserIs
lMhat;TI40 (Selection test for JBMO 2007)
cUrRsayfa x y3 z xyz 43 | (x y )(y z )(z x) |
cMeBaHRKb; x ; y ; z 0 .
3

x3 y 3 z 3
3
xyz | ( x y )( y z )( z x ) |
3
4

tag p | (x y )(y z )(z x) |


eKmanklkNHPaB

x 3 y 3 z 3 3xyz ( x y z )( x 2 y 2 z 2 xy yz zx ) (i )

cMeBaHRKb; x ; y ; z 0 eKman
x y |xy| , y z |yz| , z x |zx|

eK)an 2(x y z ) | x y | | y z | | z x |
tamvismPaB AM GM eK)an 2(x y z ) 3
ma:geToteKmansmPaB
x 2 y 2 z 2 xy yz zx

p (1)

( x y )2 ( y z )2 ( z x )2
2

tamvismPaB AM GM eK)an
x 2 y 2 z 2 xy yz zx

eroberogeday lwm pln

33 2
p ( 2)
2

- Page 73 -

vismPaBeRCIserIs

eFIVviFIKuNvismPaB (1) nig (2) eKTTYl)an


9
2( x y z )( x 2 y 2 z 2 xy yz zx ) p
2
9
( x y z )( x 2 y 2 z 2 xy yz zx ) p (ii )
4
9
(i )
(ii )
x 3 y 3 z 3 3xyz p
4
x3 y 3 z 3
3
xyz p
p | ( x y )( y z )( z x ) |
3
4
x3 y 3 z 3
3
xyz | ( x y )( y z )( z x ) |
3
4

tam nig
eKTaj
dUcenH

eK)an

eroberogeday lwm pln

eday

- Page 74 -

vismPaBeRCIserIs
lMhat;TI41 RMC 2000
eK[ a , b , c CaRbEvgRCugnRtIekaNmYy .
cUrRsayfa b aa c a bb c a bc c 3 .
tag
yk

b a c x
yz
zx
xy

a
;
b
;
c

2
2
2
a b c z

a
b
c
T

bac abc abc


yz zx xy

2x
2y
2z

enaH

1 y z z x x y

2 x x y y z z

eRBaH
dUcenH

1 y x
y z
z x 1
( ) ( ) ( ) ( 2 2 2) 3
2 x y
z y
x z 2
y x
y z
z x
2; 2; 2
x y
z y
x z
a
b
c

3
bac abc abc

eroberogeday lwm pln

- Page 75 -

vismPaBeRCIserIs
lMhat;TI42 RMC 2004
eK[ a , b , c CabIcMnYnBitEdlepgpat; a b
cUrbgajfa | a | | b | | c | abc 4 .
tamvismPaB Cauchy-Schwarz eK)an
2

| a | | b | | c | 2

c2 3

3(a 2 b 2 c 2 ) 9

eKTaj | a | | b | | c | 3 (1)
tamvismPaB AM GM eK)an
a 2 b 2 c 2 3 3 (abc )2
3

a2 b 2 c2
2
1
(abc )
3

eKTaj
naM[ 1 abc 1
eKTaj)an abc 1 (2)
eFIVplbUkvismPaB (1) nig (2) eK)an | a | | b | | c | abc 4 .

eroberogeday lwm pln

- Page 76 -

vismPaBeRCIserIs
lMhat;TI43 JBMO 2003
eK[ a , b , c CabIcMnYnBitviCmanEdl abc 1 .
cUrbgajfa 1 a b3 c ab bc6 ca .
eyIgtag a 1x ; b y1 ; c 1z enaHvismPaBGacsresr
1

3
6

xy yz zx x y z

1
eRBaH xyz abc
1

eyIgman (x y ) (y z ) (z x) 0
eKTaj x y z xy yz zx
EfmGgTaMgBIrnwg 2xy 2yz 2zx eK)an
2

( x y z )2 3( xy yz zx )
3
9
6
1
1

xy yz zx
( x y z )2 x y z

naM[

eRBaH
dUcenH

6
9
3
1

0
2
x y z (x y z )
xyz
3
6
1

a b c ab bc ca

eroberogeday lwm pln

- Page 77 -

vismPaBeRCIserIs
lMhat;TI44
eK[ a 1 nig b 1 .
cUrbgajfa log a log b 2 log ( a b ) .
2
cMeBaH a 1 nig b 1
eyIgman a b 2 a.b
naM[ log 2 ( a b ) 1 ( log 2 a log 2 b )
2

log 2 a log 2 b 2 log 2 (

ma:geToteKman

ab
)
2

(1)

log 2 a log 2 b 2 log 2 a . log 2 b


2(log 2 a log 2 b ) log 2 a 2 log 2 a . log 2 b log 2 b

2(log 2 a log 2 b ) ( log 2 a log 2 b ) 2


1
log 2 a log 2 b ( log 2 a log 2 b ) 2 ( 2)
2
tamTMnak;TMng (1) nig (2) eyIgTaj

eroberogeday lwm pln

- Page 78 -

vismPaBeRCIserIs
1
ab
( log 2 a log 2 b ) 2 2 log 2 (
)
2
2
ab
)
( log 2 a log 2 b ) 2 4 log 2 (
2
ab
log 2 a log 2 b 2 log 2 (
)
2

dUcenH

log 2 a log 2 b 2

eroberogeday lwm pln

log 2 (

ab
)
2

- Page 79 -

vismPaBeRCIserIs
lMhat;TI45
eK[ x , y , z CabIcMnYnBitviCmanEdlepgpat; xyz 1 .
cUrbgajvismPaB
( x y 1) 2 ( y z 1) 2 ( z x 1) 2

xyz
z
x
y

rebobTI1 tamvismPaB AM GM eK)an


( x y 1) 2
z 2 | x y 1 | 2( x y 1) (1)
z
( y z 1) 2
x 2 | y z 1 | 2( y z 1) ( 2)
x
( z x 1) 2
y 2 | z x 1 | 2( z x 1) ( 3)
y

bUkvismPaB (1) , (2) , (3) GgnwgGgeK)an


( x y 1) 2 ( y z 1) 2 ( z x 1) 2

3( x y z ) 6
z
x
y

eday x y z 3 3 xyz 3 eRBaH xyz 1


eK)an 2(x y z ) 6
b 2(x y z ) 6 0
b 3(x y z ) 6 x y z
( x y 1) 2 ( y z 1) 2 ( z x 1) 2

xyz
dUcenH
z
x
y
eroberogeday lwm pln

- Page 80 -

vismPaBeRCIserIs
rebobTI2
( x y 1) 2 ( y z 1) 2 ( z x 1) 2
T

(x y z )
z
x
y
x 2 y 2 z 2 (x y z )2
edayeRbIvismPaB a b c a b c eK)an
2

| x y 1 | | y z 1 | | z x 1 |
T
(x y z)
xyz

( x y 1 y z 1 z x 1) 2 ( x y z ) 2
T
xyz
( 2x 2y 2z 3) 2 ( x y z ) 2
T
xyz
3( x y z 3)( x y z 1)
T
xyz

tamvismPaB AM GM eKman
x y z 3 3 xyz 3 eRBaH xyz 1
eKTaj)an x y z 3 0 nig x y z 1 2
ehtuenH T 3(x y zx3y)(x z y z 1) 0 Bit
dUcenH

( x y 1) 2 ( y z 1) 2 ( z x 1) 2

xyz
z
x
y

eroberogeday lwm pln

- Page 81 -

vismPaBeRCIserIs
lMhat;TI46 BMO 2010
eK[ a , b , c CacMnYnBitviCman .
a 2b(b c) b 2c(c a ) c 2a(a b )
cUrbgajfa a b b c c a 0 .
bgajfa
a 2b(b c) b 2c(c a) c 2a(a b )

0
ab
bc
ca

eKman

a 2b(b c) a 2b 2 a 2bc (a 2b 2 ab 2c) (a 2bc ab 2c)

ab
ab
ab
ac
ab 2 (a c) abc(a b )
abc

ab 2 .
ab
ab
a 2 b( b c )
eK)an a b ab 2 . aa bc abc (1)
b 2c(c a )
RsaybMPWdUcKaEdreK)an b c bc 2 bb ac abc (2)
c 2a(a b )
nig c a ca2 cc ba (3)
bUksmPaB (1) , (2) nig (3) GgnigGgeK)an
cb
ba
cb
T ab 2
bc 2
ca 2
3abc
ab
bc
ca
a 2b(b c) b 2c(c a ) c 2a(a b )
Edl T a b b c c a

eroberogeday lwm pln

- Page 82 -

vismPaBeRCIserIs
tamvismPaB AM GM eKman

cb
ba
cb
bc 2
ca 2
3abc
ab
bc
ca
b ab 2 ac bb bc 2 bb ac ca2 cc ba 3abc 0
a 2b(b c) b 2c(c a ) c 2a(a b )
dUcenH a b b c c a 0
ab 2

eroberogeday lwm pln

- Page 83 -

vismPaBeRCIserIs
lMhat;TI47
eK[ a , b , c CabIcMnYnBitviCman . cUrbgajfa
a5 b 5 c5 (a b c)5 10
(a b c ) 2
3
3
3
3
9
a b c (a b c )

eKmansmPaB
a3 b3 c3 (a b c)3 3(a b)(b c)(c a)
a5 b5 c5 (a b c)5 5(a b)(b c)(c a)(a2 b2 c2 ab bc ca)

eK)an
a 5 b 5 c 5 ( a b c )5 5 2
2
2

(
a

c
ab bc ca)
3
3
3
3
3
a b c (a b c )
eyIgnwgRsayfa 53 (a2 b2 c2 ab bc ca) 109 (a b c)2
b 3(a2 b2 c2 ab bc ca) 2(a b c)2

b
a b c ab bc ca
tamvismPaB AM GM eKman
2

a2 b 2 b 2 c2 c2 a2
ab bc ca

a2 b 2 c2
2
2
2
a5 b 5 c5 (a b c)5 10
2

(
a

c
)
a 3 b 3 c 3 (a b c ) 3 9

dUcenH

eroberogeday lwm pln

Bit

- Page 84 -

vismPaBeRCIserIs
lMhat;TI48
cMeBaHRKb;cMnYnBit x (0 , 4 )
cUrbgajfa (cos x) (sin x) .
tamvismPaB Bernoulli cMeBaHRKb;cMnYn x nig a Edl x 1 nig a 1
eyIgman (1 x) 1 ax .
ehtuenHcMeBaH 0 x 4 eK)an
cos x

sin x

(cos x )

cos x
sin x

(1 sin x )

eday (1 sin x)
eK)an (cos x)
eKTaj (cos x)
2

cos x
sin x

cos x
sin x

cos x
sin x

.(1 sin x )

1 cos x

cos x
sin x

nig (1 sin x)

cos x
sin x

1 cos x

(1 cos x )(1 cos x ) sin 2 x

cos x
sin x

sin x

cos x
sin x

ln(cos x ) ln(sin x )
cos x
ln(cos x ) ln(sin x )
sin x
cos x ln(cos x ) sin x ln(sin x )
ln(cos x )cos x ln(sin x )sin x

dUcenH (cos x)

cos x

(sin x )sin x

eroberogeday lwm pln

- Page 85 -

vismPaBeRCIserIs
lMhat;TI49
eK[ a , b , c , d CabIcMnYnBitEdlepgpat;vismPaB
(a 2 b 2 1)(c 2 d 2 1) (ac bd 1)2

cUrbgajfa a b 1 nig c d 1 .
tag x 1 a b nig y 1 c d
eyIg]bmafa x 0 nig y 0
vismPaB (a b 1)(c d 1) (ac bd 1)
smmUl xy (ac bd 1)
b 4xy (2ac 2bd 2)
eday x y 2 a b c d
enaH 2ac 2bd 2 a b c d 2ac 2bd x y
2

[(a c)2 (b d )2 x y ]

eKTaj 4xy [(a c) (b d) (x y )] (x y )


b 4xy x 2xy y
b (x y ) 0 minBit . naM[kar]bmaxagelIpyBIkarBit .
dUcenHeKTaj x 0 nig y 0 naM[ a b 1 nig c d 1 .
2

eroberogeday lwm pln

- Page 86 -

vismPaBeRCIserIs
lMhat;TI50
eK[ a , b , c CabIcMnYnBitviCmanEdl ab bc ca 1 .
a
b
c
cUrkMnt;tmGb,brmankenSam E a b b c c a .
kMnt;tmGb,brmankenSam E
eKman a a b a aab b ab a aab
b
tamvismPaB AM GM eKman a b 2 ab
ab
eK)an a 1 b 2 1ab b aab

b
2
2

a2
ab
eKTaj a b a aab
a
(1)
b
2
2
b
bc
c2
ca
dUcKaEdr b c b 2 (2) ; c a c 2 (3)
bUkvismPaB (1) ; (2) & (3) eKTTYl)an
ab bc ca
1
abc
2
2
tamvismPaB Cauchy Schwarz eKman
Eabc

a b c ab bc ca 1
eKTaj E 1 12 12 . dUcenHtmGb,brman E

eroberogeday lwm pln

KW E

min

1
2

- Page 87 -

vismPaBeRCIserIs
lMhat;TI51
eK[ x1 , x2 , ..., xn Edl n 2 CacMnYnBitviCmanEdlepgpat;
1
1
1
1

....

xn 1998 1998
x1 1998 x 2 1998

cUrbgajfa x1n.x2 1...xn 1998 .


eKman 1 1 .... 1 1
x 1998 x 1998
x 1998 1998
b 1998 1998 .... 1998 1
x 1998 x 1998
x 1998
tag y 1998 eK)an y1 y 2 .... yn 1
n

xi 1

eKTaj 1 y i ji (y i ) Edl 1 i n nig 1 j n

tam AM GM eKman (y ) (n 1)
j i

n 1

(y i )
j i

eK)an 1 y i (n 1) n1 ji (y i )
eKTaj (1 y ) (n 1)
n

i 1

Et 1 y

yi

dUcenH

(y i )

i 1

x1x 2 ...xn
xi
1998n
1998
x1 .x 2 ...xn
1998
n1

enaH

eroberogeday lwm pln

b (1 y y ) (n 1)
n

i 1

(n 1)n

- Page 88 -

vismPaBeRCIserIs
lMhat;TI52
eK[ x , y , z CabIcMnYnBitviCmanEdl xyz x y z .
cUrRsayfa 1xzy 1yxz 1zyx 227
.
xyz
2

xy yz zx
27

1 z 2 1 x 2 1 y 2 2xyz
tag T 1xzy2 1yxz2 1zyx2
( x y )2
( y z )2
( z x )2
b T x y z 2 (x y ) y z x2 (y z ) z x y 2 (z x)

bgajfa

tamvismPaB Cauchy Schwarz eKman


an
(a1 a 2 a 3 ... an )2
a1 a 2 a 3

....

bn
b1 b 2 b 3 ... bn
b1 b 2 b 3
2

eK)an
T

( x y ) ( y z ) ( z x)2

2( x y z ) z 2 ( x y ) x 2 ( y z ) y 2 ( z x )

4( x y z )2
T
2xyz z 2 x z 2 y x 2 y x 2 z y 2 z y 2 x
4( x y z )2
T
( x y )( y z )( z x )

eroberogeday lwm pln

- Page 89 -

vismPaBeRCIserIs
tamvismPaB AM GM eKman
( x y ) ( y z ) ( z x ) 3 3 ( x y )( y z )( z x )
2( x y z ) 3 3 ( x y )( y z )( z x )
8 ( x y z )3 27( x y )( y z )( z x )
4( x y z )2
eKTaj (x y )(y z )(z x) 2(x 27y z ) 227
xyz
naM[ T 227
xyz
.
dUcenH 1xzy2 1yxz2 1zyx2 227
xyz

eroberogeday lwm pln

- Page 90 -

vismPaBeRCIserIs
lMhat;TI53
cUrbgajfaebI abc 8 nig a , b , c 0 enaHeK)an
a2
(1 a 3 )(1 b 3 )

tag T

b2

(1 b 3 )(1 c3 )

a2
(1 a )(1 b )
3

b2
(1 b )(1 c )
3

c2
(1 c 3 )(1 a 3 )

4
3

c2
(1 c3 )(1 a 3 )

tamvismPaB AM GM eKman
2

1 a 1 a a 2 a

1 a 3 (1 a )(1 a a 2 )
2
2

a2 2
3
eKTaj 1 a 2 .
a2
4a 2
2
ehtuenH
(1)
2
3
3
(1 a )(1 b ) (a 2)(b 2)
2

RsaydUcKaEdreK)an
b2

4b 2
( 2)
2
2
3
3
(
b
2
)(
c
2
)

(1 b )(1 c )
c2

4c2
( 3)
2
2
3
3
(1 c )(1 a ) (c 2)(a 2)

eroberogeday lwm pln

- Page 91 -

vismPaBeRCIserIs
bUkvismPaB (1) , (2) , (3) GgnwgGgeK)an
4a 2
4b 2
4c 2
T 2
2
2
2
2
(a 2)(b 2) (b 2)(c 2) (c 2)(a 2 2)
4[a 2 (c 2 2) b 2 (a 2 2) c 2 (b 2 2)]

(a 2 2)(b 2 2)(c 2 2)
4(a 2b 2 b 2c 2 c 2a 2 2a 2 2b 2 2c 2 )
2 2 2
a b c 2(a 2b 2 b 2c 2 c 2a 2 2a 2 2b 2 2c2 ) 8

eday abc 8 enaHeK)an


2(a 2b 2 b 2c2 c2a 2 2a 2 2b 2 2c 2 )
2t
T 2 2

(a b b 2c 2 c 2a 2 2a 2 2b 2 2c 2 ) 36 t 36

Edl t a b b c c a 2a
tamvismPaB AM GM eK)an
2 2

2 2

2 2

2b 2 2c 2

t 3 3 a 4b 4c4 63 a 2b 2c2 72 eRBaH abc 8 )


eK)an 1 36t 1 12 32 b t t36 32 naM[ t t36 23
ehtuenH T t 2t36 43 Bit .

dUcenH

a2
(1 a )(1 b )
3

eroberogeday lwm pln

b2
(1 b )(1 c )
3

c2
(1 c )(1 a )
3

4
3

- Page 92 -

vismPaBeRCIserIs
lMhat;TI54
eK[RtIekaN ABC mYymanRCug BC a , AC b , AB c ehIy
manmMukgCamMuRsYc .
ab bc ca
cUrRsayfa cos

4(a b c) .
C cos A cos B
ab bc ca
bgajfa cos

4(a b c)
C cos A cos B
ab bc ca

tag cos
C cos A cos B

(a b ) 2
(b c ) 2
(c a ) 2

(a b ) cos C (b c) cos A (c a ) cos B

x1 x 2
xn
( x1 x 2 ... xn )2
tamvismPaB y y .... y y y ... y
1
2
n
1
2
n
[(a b ) (b c) (c a )]2
eK)an (a b) cos C (b c) cos A (c a) cos B
2

4(a b c)2

(b cosC c cosB) (c cosA a cosC) (a cosB b cosA)

dUcenH

4(a b c)2

4(a b c)
abc
ab bc ca

4(a b c)
cos C cos A cos B

eroberogeday lwm pln

- Page 93 -

vismPaBeRCIserIs
lMhat;TI55
eK[ x , y , z 0 Edl x y z 1 .
cUrRsayfa (1 x x) (1 y y ) (1 z z )
3

1
4

x3
y3
z3
Rsayfa (1 x)2 (1 y )2 (1 z )2 14
x3
( x 2x 2 x 3 ) ( 2x 2 x )
eKBinit (1 x)2
(1 x )2
2x 2 x
x
(1 x )2
(9x 2 6x 1) (1 2x x 2 )
x
4(1 x )2
1 ( 3x 1)2
( 3x 1)2 (1 x )2
x
x
4 4(1 x )2
4(1 x )2
( 3x 1)2
x3
eday 4(1 x)2 0 enaH (1 x)2 x 14 (1)
y3
1
z3
dUcKaEdr (1 y )2 y 4 (2) nig (1 z )2 z 14 (3)

bUkvismPaB (1) , (2) , (3) Gg nig GgeK)an


3 1
x3
y3
z3

x
y
z
2
2
2
4 4
(1 x )
(1 y )
(1 z )

eroberogeday lwm pln

Bit .

- Page 94 -

vismPaBeRCIserIs
lMhat;TI56
eK[ a , b , c , d CacMnYnBitviCmanEdl abcd 1 .
ebIeKdwgfa a b c d ba bc dc da enaHcUrbgajfa
b c d a
.
a b c d
bgajfa a b c d ba bc dc da
a b c d a2 b 2 c2 d 2
eKman a b c d b c d a ab bc cd ad
(a b c d ) 2
abcd
ab bc cd da
naM[ ab bc cd da a b c (1)
abcd

b c d a (bc )2 (cd)2 (ad )2 (ab )2


ma:geTot a b c d abc2 bcd2 a2cd ab2d
(bc )2 (cd )2 (ad )2 (ab )2

c
d
a
b
d
a
b
c
(bc cd da ab )2
(*)

c d a b

d a b c
tam (1) eK)an (bc cd da ab)2 (a b c d)2 (2)
tamsmtikm a b c d ba bc dc da

eroberogeday lwm pln

- Page 95 -

vismPaBeRCIserIs
eKTaj

1
( 3)
abcd

a b c d

b c d a
KuNvismPaB (2) nig (3) GgnigGgeK)an

(bc cd da ab )2
a b c d (**)
a b c d

b c d a
tam (*) nig (**) eK)an ba bc dc da a b c d
dUcenH a b c d ba bc dc da .

eroberogeday lwm pln

- Page 96 -

vismPaBeRCIserIs
lMhat;TI57 Czech and Slovak Republics 2005
eKyk a , b , c CacMnYnviCmanEdlepgpat; abc 1 .
cUrbgajfa (a 1)(a b 1) (b 1b)(c 1) (c 1)(c a 1) 43 .
tag T (a 1)(a b 1) (b 1b)(c 1) (c 1)(c a 1)
a(c 1) b(a 1) c(b 1)
(a 1)(b 1)(c 1)
a b c ab bc ca

1 a b c ab bc ca abc
a b c ab bc ca

2 a b c ab bc ca
2
1
2 a b c ab bc ca

tamvismPaB AM GM eK)an

1 1 a b c ab bc ca 8 8 a 3 b 3c 3 8
1
1
2 3

T 1
2 a b c ab bc ca 8
8 4
a
b
c
3

(a 1)(b 1) (b 1)(c 1) (c 1)(a 1) 4

eKTaj
dUcenH

eroberogeday lwm pln

naM[

- Page 97 -

vismPaBeRCIserIs
lMhat;TI58 APMO 1998
eKyk a , b , c CacMnYnviCman.
bc
cUrbgajfa (1 ba )(1 bc )(1 ac ) 2(1 a 3abc
).
vismPaBenHsmmUl

abc
a c b abc
a b c
2(1 3
)
1 ( ) ( )
c b a abc
b c a
abc
a b c a c b 2(a b c)
3
b c a c b a
abc

tag a x

, b y3 , c z3

eK)an

x 3 y 3 z 3 x 3 z 3 y 3 2( x 3 y 3 z 3 )
3 3 3 3 3
3
xyz
y
z
x
z
y
x

tamvismPaB AM GM eK)an
x2
x3 x3
1 3
(1)
yz
y3 z3
y3 y3
y2
3 1 3
( 2)
3
zx
z
x
z2
z3 z3
3 1 3
( 3)
3
xy
x
y

x3 y 3 z 3 x3 z 3 y 3
x3 y 3 z 3 x3 z 3 y 3
3 3 3 3 3 66 3. 3. 3. 3. 3. 3 6
3
y z x z y x
y
z
x
z
y
x

1 x3 y 3 z 3 x3 z 3 y 3

) 3 ( 4)
(
2 y 3 z 3 x3 z 3 y 3 x3

eroberogeday lwm pln

- Page 98 -

vismPaBeRCIserIs
bUkvismPaB (1) , (2) , (3) nig (4) eK)an
3( x 3 y 3 z 3 )
x2 y 2 z 2
3 x3 y 3 z 3 x3 z 3 y 3

) 3(
)
(
xyz
yz zx xy
2 y 3 z 3 x3 z 3 y 3 x3

eKTaj
dUcenH

x 3 y 3 z 3 x 3 z 3 y 3 2( x 3 y 3 z 3 )
3 3 3 3 3
3
xyz
y
z
x
z
y
x
abc
c
b
a
)
(1 )(1 )(1 ) 2(1 3
a
c
b
abc

eroberogeday lwm pln

Bit

- Page 99 -

vismPaBeRCIserIs
lMhat;TI59 Romania 2002
eKyk a , b , c CacMnYnBitncenaH (0 ,1 ) .
cUrbgajfa abc (1 a)(1 b)(1 c) 1 .
cMeBaHRKb;cMnYnBit x , y , z ncenaH (0 , 2 )
eKyk a cos2 x , b cos2 y , c cos2 z
vismPaBsmmUl cos x cos y cos z sin x sin y sin z 1
eday z (0 , 2 ) eKman cos z 1 nig sin z 1
eKTaj cos x cos y cos z cos x cos y nig sin x sin y sin z sin x sin y
eRBaH cos x cos y 0 ; sin x sin y 0
naM[ cos x cos y cos z sin x sin y sin z cos x cos y sin x sin y
cos x cos y cos z sin x sin y sin z cos( x y )

eday cos(x y ) 1 enaH cos x cos y cos z sin x sin y sin z 1 Bit
dUcenH abc (1 a)(1 b)(1 c) 1 .

eroberogeday lwm pln

- Page 100 -

vismPaBeRCIserIs
lMhat;TI60 Poland 2006
eKyk a , b , c CacMnYnBitviCmanEdl ab bc ca abc
a4 b4
b 4 c4
c4 a4
cUrbgajfa ab(a3 b3 ) bc(b3 c3 ) ca(c3 a3 ) 1 .
eKman ab bc ca abc a1 b1 1c 1
tag x a1 , y b1 , z 1c enaH x y z 1 ehIyvismPaBsmmUl
x4 y 4 y 4 z 4 z 4 x4
1
3
3
3
3
3
3
x y
y z
z x

tamvismPaB Tchebyshev eKman


x4 y 4 x y
eKTaj x3 y 3 2 (1)

x4 y 4 x 3 y 3 x y

.
2
2
2

y4 z4 y z
( 2)

3
3
2
y z

RsaydUcKaEdreK)an
nig
bUkvismPaB (1) , (2) nig (3) eK)an

z 4 x4 z x
( 3)

3
3
2
z x

x4 y 4 y 4 z 4 z 4 x4
3
3
xyz 1
3
3
3
3
x y
y z
z x

Bit .

eroberogeday lwm pln

- Page 101 -

vismPaBeRCIserIs
lMhat;TI61 Kazakhstan 2008
eKyk x , y , z CacMnYnBitviCmanEdl xyz 1
cUrbgajfa yz1 z zx1 x xy1 y 32 .
tag x ba ; y bc ; z ac enaH xyz 1
vismPaBsmmUl b a c c b a a c b 32
tag

a
b
c
a2
b2
c2
T

b c c a a b ab ac bc ab ac bc
(a b c ) 2
T
Cauchy Schwarz
2(ab bc ca)

tamvismPaB
eK)an
tamvismPaB AM GM eKman

a2 b 2 b2 c2 c2 a2

ab bc ca
2
2
2
a 2 b 2 c 2 ab bc ca
(a b c)2 3(ab bc ca)
3
T
2
1
1
1
3

yz z zx x xy y 2

eKTaj)an
dUcenH

Bit .

eroberogeday lwm pln

- Page 102 -

vismPaBeRCIserIs
lMhat;TI62 Korea 2000
cMnYnBit a, , b , c , x , y , z epgpat; a b c 0 nig x y z 0
cUrbgajfa
a2x2
b2y 2
c2z 2
3

.
(by cz )(bz cy ) (cz ax )(cx az ) (ax by )(ay bx ) 4
eday a b c 0 nig x y z 0
eK)an (b c)((z y ) bz cy by cz 0 b bz cy by cz
eKTaj (by cz )(bz cy ) (by cz)2 2(b2y 2 c2z 2 )
a2x2
a2x2
ehtuenH (by cz)(bz cy ) 2(b2y 2 c2z 2 ) 12 . Y X Z (1)
Edl X a x , Y b y , Z c z .
RsaydUcKaEdreK)an
2 2

2 2

2 2

b2y 2
1 Y
c2z 2
1 Z
( 2) ,
( 3)
.
.
(cz ax )(cx az ) 2 Z X
(ax by )(ay bx ) 2 X Y

tag

a2x2
b2y 2
c2z 2
T

(by cz )(bz cy ) (cz ax )(cx az ) (ax by )(ay bx )


1 X
Y
Z
T (

)
2 YZ Z X XY

eK)an
tamvismPaB AM GM eKman

( X Y ) ( Y Z ) ( Z X ) 3 3 ( X Y )( Y Z )( Z X )
3
X Y Z 3 ( X Y )( Y Z )( Z X) (i )
2

eroberogeday lwm pln

- Page 103 -

vismPaBeRCIserIs
ehIy Y 1 Z Z 1 X X 1 Y ( X Y)(Y3 Z)(Z X)
KuNvismPaB (i) nig (ii ) Gg nig Gg eK)an
3

(ii )

XYZ XYZ XYZ 9

YZ
ZX
XY
2
X
Y
Z
9
1
1
1
YZ
ZX
XY
2
3
X
Y
Z
9

3
2
YZ Z X XY 2
3
T
4

eKTaj
dUcenH

Bit

a2x2
b2y 2
c2z 2
3

(by cz )(bz cy ) (cz ax )(cx az ) (ax by )(ay bx ) 4

eroberogeday lwm pln

- Page 104 -

vismPaBeRCIserIs
lMhat;TI63 Ireland 2002
3 3 xyz
x
y
z
cUrbgajfa 1 x 1 y 1 z 1 3 xyz
cMeBaHRKb; 0 x , y , z 1 .
tagGnuKmn_ f (x) 1 x x 1 1 x 1 Edl 0 x 1
eK)an f ' (x) (1 1x)2 ehIy f '' (x) (1 2x)3 0 cMeBaH 0 x 1
naM[ f (x) CaGnuKmn_e)a:g .
tamRTwsIbT Jensen cMeBaHRKb; 0 x , y , z 1 eK)an
f ( x) f ( y ) f (z ) 3 f (

xyz
)
3

eday f ' (x) (1 1x)2 0 naM[ f (x) CaGnuKmn_ekIn .


tamvismPaB AM GM eKman x y3 z xyz
eKTaj f ( x y3 z ) f ( xyz ) ehtuenH f (x) f (y ) f (z ) 3f (
3

dUcenH

3 3 xyz
x
y
z

1 x 1 y 1 z 1 3 xyz

eroberogeday lwm pln

xyz )

- Page 105 -

vismPaBeRCIserIs
lMhat;TI64 IMO 2001
cUrbgajfacMeBaHRKb; a , b , c CacMnYnBitviCmaneK)an
a
b
c

1 .
2
2
2
a 8bc

b 8ca
c 8ab
a
b
c

1
2
2
2
a 8bc
b 8ca
c 8ab
1
x0
f ( x)
x
1
3
f ' (x)
f '' ( x)
0
3
5
2 x
4 x

bgajfa

tagGnuKmn_
eK)an

Edl
nig

cMeBaHRKb; x 0

naM[ f (x) CaGnuKmn_e)a:g .


tamvismPaB Jensen RKb; a , b , c 0 eK)an
af ( x ) bf ( y ) cf ( z )
ax by cz
f(
)
abc
abc

eday

x a 2 8bc , y b 2 8ca , z c 2 8ab

a
a 2 8bc

b
b 2 8ca
abc

eK)an

c
c 2 8ab

abc
a 3 b 3 c 3 24abc

(a b c ) 3
(i )

3
3
3
2
2
2
a b c 24abc
a 8bc
b 8ca
c 8ab
a

eKman (a b c)

a 3 b 3 c 3 3(a b )(b c)(c a )

eroberogeday lwm pln

- Page 106 -

vismPaBeRCIserIs
tamvismPaB AM GM eKman
a b 2 ab

b c 2 bc

c a 2 ca

naM[ (a b)(b c)(c a) 2 ab .2 bc .2 ca 8abc


eKTaj)an (a b c) a b c 24abc
b a (ba bc c)24abc 1 (ii )
tamTMnak;TMng (i) nig (ii ) eKTaj)an
a
b
c

1 Bit .
2
2
2
3

a 8bc

b 8ca

eroberogeday lwm pln

c 8ab

- Page 107 -

vismPaBeRCIserIs
lMhat;TI65 Bulgaria 2007
cUrbgajfacMeBaHRKb; a , b , c CacMnYnBitviCmaneK)an
(a 1)(b 1)2
3 c a 1
3

2 2

(b 1)(c 1)2
3 a b 1
3

2 2

(c 1)(a 1)2
3 b c 1
3

2 2

abc3

tamvismPaB AM GM eK)an
ca c a 3 c 2a 2 ca c a 1 1 3 c 2a 2
3

(a 1)(c 1) 1 3 c 2a 2
(a 1)(b 1)2 (b 1)2
(1)

3 2 2
c
1

3 c a 1
3

eKTaj)an
RsaydUcKaEdreK)an
(b 1)(c 1)2

(c 1)2
(c 1)(a 1)2 (a 1)2
( 2) ,
( 3)

3
3 2 2
2
2
b1
a1
3 a b 1
3 b c 1

bUkvismPaB (1) , (2) , (3) eK)an


(a 1)(b 1)2

(b 1)(c 1)2

(c 1)(a 1)2

abc3

3 c a 1
3 a b 1
3 b c 1
(b 1)2 (c 1)2 (a 1)2 (a b c 3)2

abc3
c1
a1
b1
abc 3
3

2 2

eRBaH

eroberogeday lwm pln

2 2

2 2

- Page 108 -

vismPaBeRCIserIs
lMhat;TI66 APMO 2007
eK[ x , y , z CacMnYnBitviCmanEdl
cUrRsayfa
x 2 yz
2x ( y z )
2

eKman
eday

y 2 zx

2y ( z x )
2

x 2 yz
2x 2 ( y z )

x y z 1

z 2 xy

2z ( x y )
2

x 2 ( y z )x yz

( y z )x

2x 2 ( y z )
2x 2 ( y z )
yz
( x y )( x z )

2
2x 2 ( y z )
yz
y z

( y z )2 2( y z )
2
2
y z
x 2 yz
( x y )( x z )

(1)
2
2
2
2x ( y z )
2x ( y z )

eKTaj

RsaydUcKaEdreK)an
y 2 zx
2y 2 ( z x )
z 2 xy
2z 2 ( x y )

( y z )( y x )
2y 2 ( z x )
( z x )( z y )
2z 2 ( x y )

z x
( 2)
2

x y
( 3)
2

bUkvismPaB (1) , (2) , (3) eK)an


S

( x y )( x z )
2x ( y z )
2

( y z )( y x )

eroberogeday lwm pln

2y ( z x )
2

( z x )( z y )
2z ( x y )
2

x y z

- Page 109 -

vismPaBeRCIserIs
Edl S

x 2 yz
2x ( y z )
2

y 2 zx

2y ( z x )
2z ( x y )
( x y )( x z )
0
2
2x ( y z )
2

]bmafa x y z eK)an

z 2 xy
2

ehIy
( y z )(y x)
2y ( z x )
2

xz
xy
(y z )

2
2
2
2z ( x y )
2z ( x y )
2y ( z x)

( z x)(z y )

eday x z x y 0 eKTaj
( y z )(y x)
2y ( z x )
2

( y z )(y x)
2y ( z x )
2

xy
xy
(y z)

2
2
2
2z ( x y )
2y ( z x)
2z ( x y )

( z x)(z y )

1
1
(y z ) (x y )

2
2
2
2z ( x y )
2z ( x y )
2y ( z x)

( z x)(z y )

eday y 2 (z x) y 2z y 2z yz 2 z 2x z 2 (x y )
eKTaj 2 1 2 1 0 ehIy (y z )(x y ) 0
eKTaj
dUcenH

2z ( x y )
2y ( z x )
( y z )(y x) ( z x)(z y )

0
2
2
2y ( z x )
2z ( x y )
x 2 yz
2x ( y z )
2

eroberogeday lwm pln

y 2 zx
2y ( z x )
2

naM[ S
z 2 xy

2z ( x y )
2

x y z 1

- Page 110 -

vismPaBeRCIserIs
lMhat;TI67 Baltic 2008
ebIcMnYnBitviCman a , b , c epgpat; a2 b2 c2 3
enaHcUrbgajfa
a2
b2
c2
(a b c ) 2

12
2 b c2 2 c a2 2 a b 2

tamvismPaB Cauchy Schwarz eK)an

a2
b2
c2
(a b c ) 2

2
2
2
2bc
2ca
2ab
6 a b c a2 b2 c2
a2
b2
c2
(a b c ) 2

2 b c2 2 c a2 2 a b 2 9 a b c

eRBaH a2 b2 c2 3 .
eKman (a b c) 3(a b c ) 9 a b c 3
(a b c ) 2 (a b c ) 2
b 9 a b c 12 9 a b c 12
2

dUcenH

a2
b2
c2
(a b c ) 2

2
2
2
12
2bc
2ca
2ab

eroberogeday lwm pln

- Page 111 -

vismPaBeRCIserIs
lMhat;TI68 Canada 2008
eKmancMnYnBitviCman a , b , c epgpat; a b c 1 .
b ca c ab 3
cUrbgajfa aa bc
.

bc b ca c ab 2
b ca c ab 3
bgajfa aa bc

bc b ca c ab 2
2bc
2bc
2bc

eKman 1 aa bc
(1)
bc 1 b c bc (1 c)(1 b ) (a b )(a c)
2ca
dUcKaEdr 1 bb ca

( 2)
ca (b c)(b a )
2ab

nig 1 cc ab
( 3)
ab (c a )(c b )
bUkvismPaB (1) , (2) , (3) eK)an

a bc b ca c ab
2bc
2ca
2ab

a bc b ca c ab
(a b )(a c) (a b )(b c) (c a )(c b )

a bc b ca c ab
2bc
2ca
2ab

a bc b ca c ab
(a b)(a c) (a b)(b c) (c a)(c b)

3(

b ca c ab 3
Bi
t
]bmafa aa bc

bc b ca c ab 2
eKTaj 3 (a b2)(bca c) (a b2)(cab c) (c a2)(abc b) 32

3
2bc
2ca
2ab

(a b )(a c ) (a b )(b c) (c a )(c b ) 2

b
smmUl 4[ bc(b c) ca(c a) ab(a b)] 3(a b)(b c)(c a)
smmUl ab bc ca 9abc .
eroberogeday lwm pln

- Page 112 -

vismPaBeRCIserIs
tamvismPaB AM GM eKman
1 1 1
3
a b c 3 abc nig 3
a b c
abc
eKTaj (a b c)( a1 b1 1c ) 9
b (a b c)(ab bc ca) 9abc
eday a b c 1 enaH ab bc ca 9abc Bit
b ca c ab 3
dUcenH aa bc
.

bc b ca c ab 2
3

eroberogeday lwm pln

- Page 113 -

vismPaBeRCIserIs
lMhat;TI69 Lithuania 2006
eK[ a , b , c Ca cMnYnBitviCman.
cUrbgajfa a 1 bc b 1 ca c 1 ab 12 ( ab1 bc1 ca1 ) .
bgajfa a 1 bc b 1 ca c 1 ab 12 ( ab1 bc1 ca1 )
tamvismPaB AM GM eK)an
2

a 2 bc 2 a 2bc 2 ab . bc

RsaydUcKaEdreK)an

1
1 1
1
.
.

(1)
2
2
ab
bc
a bc

1
1 1
1
1
1 1
1
.
.
(
2
)
;
.
.

( 3)
2
2
b ca 2 bc . ca
c ab 2 ca ab

bUkvismPaB (1) , (2) , (3) eK)an

1
1
1
1 1
1
1
1
1
1
(
.
.
.

)
2
2
2
2
ab
bc
bc
ca
ca
ab
a bc b ca c ab

tamvismPaB Cauchy Schwarz eK)an


(

1
1
1 2
1
1 1 1 1
1
1
1
1

.
) ( )( )
.
ab bc ca bc ca ab
ab bc
bc ca
ca ab
1
1
1
1
1
1
1
1 1


.
.
.
ab bc
bc ca
ca ab ab bc ca
1
1
1
1 1
1
1

)
2
2
2
2
ab
bc
ca
a bc b ca c ab

b
dUcenH

eroberogeday lwm pln

- Page 114 -

vismPaBeRCIserIs
lMhat;TI70 USAMO 1991
eK[ a , b , c , d CacMnYnBitviCman .
a4
b4
c4
d4
S

(a b )(a 2 b 2 ) (b c)(b 2 c 2 ) (c d )(c 2 d 2 ) (d a)(d 2 a 2 )

cUrbgajfa S a b 4 c d .
bgajfa S a b 4 c d
tamvismPaB Cauchy Schwarz eKman (a b) 2(a b )
KuNvismPaBnwg a 2 b 2 eK)an (a b) (a b ) 2(a b )
eday (a b ) 2(a b ) enaH (a b) (a b ) 4(a b )
a4 b4
eKTaj (a b)(a 2 b 2 ) a 4 b
eKman a b 2a (a b ) 2a (a b)(a b)(a b )
2a 4
eK)an (a b)(a 2 b 2 ) (a b) a 4 b
2

2 2

2 2

a4
ab ab
(*)

2
2
8
2
(a b )(a b )
2(a b c d ) (a b ) (b c) (c a)
(*)

S
8
2
abcd
S
4

tam eKTaj)an
dUcenH

eroberogeday lwm pln

- Page 115 -

vismPaBeRCIserIs
lMhat;TI71
eK[cMnYnBitviCman x1 , x2 , ...., xn ehIyeKtag
Sn x1 x 2 ... xn . cUrRsayfa
S2
Sn
(1 x1 )(1 x 2 )....(1 xn ) 1 S ...
2!
n!
tamvismPaB AM GM eKman

S
n S
(1 x1 )(1 x 2 )...(1 xn )
1
n
n
n

tameTVFajtuneKman
n

2
3
n
k
n
S

2 S
3 S
n S
k S
1 1 S Cn . 2 Cn . 3 .... Cn n Cn k
n
n
n
n
n
k 0

eday

Ckn
n!
(n k 1)(n k 2)....(n 1)n 1
.

k
k
k
k!
n
k! (n k )! n
n
k 1
k2
1
1 1
)(1
).....(1 ).1.
(1
k! k!
n
n
n
n
n Sk
S

eK)an 1 n k! Bit
k 0

dUcenH

S2
Sn
(1 x1 )(1 x 2 )....(1 xn ) 1 S ...
2!
n!

eroberogeday lwm pln

- Page 116 -

vismPaBeRCIserIs
lMhat;TI72 Turkey National Olympiad 2010
cMeBaHRKb;cMnYnKt;viCman n nig cMeBaHRKb;cMnYnBitviCman
a1 , a 2 , ...., a n epgpat; a1a 2a 3 ....a n 1 cUrbgajfa
n
ai
1 n 1

.
4
2 a
i 1

ai 3

i 1 i

CadMbUgeyIgRtUvRsay[eXIjfacMeBaHRKb;cMnYnBit x 0 eKman
x

x1

x
x4 3

x4 3 ( x 1)2
( x 1)2 ( x 2 2x 2) 0

eday (x 1)2 0 nig x2 2x 2 (x 1)2 1 0


naM[ (x 1)2 (x2 2x 2) 0 BitRKb;cMnYnBit x .
n a
n
i
ehtuenH a 1 a4i (1)
i 1 i

i 1

CabneToteyIgnwgRsayfa
]bmafa
smmUl
smmUl

ai 3

n a
1 n 1
i
2 i 1ai i 1ai 1

n a
1
2 i
i 1ai
i 1a i 1
n
n a 1 1
n 1
1
i
)
2 (1
)
(

2
a
2
2
a
a
1
i
i
i 1
i 1
i 1 i
n
n 1
n a 1 n
2
i

2
n

2
a
2
2
a
i
i 1a i 1
i 1 i
i 1

eroberogeday lwm pln

Bit

- Page 117 -

vismPaBeRCIserIs
smmUl

n 1
ai 1
2
5n
)

(
ai 1 i 12ai
2
i 1 2ai
n

tamvismPaB AM GM cMeBaH a1 , a2 , ..., an 0 eKman


n a 1
( i

i 1 2ai
n 1

nig 2a
i 1

n a 1
2
2
2
n
) 2n
) 2n
( i .
ai 1
2
a
a

1
i
i
i 1
nn n 1 n

a1a 2a 3 ....a n 1

2 i 1 a i 2

eRBaH

n a 1

( i

eKTaj)an
eKTaj)an

n 1
2
n 5n
2n
)
ai 1 i 12ai
2 2

i 1 2a i
n a
1 n 1
i ( 2)
2 i 1ai i 1ai 1

tamTMnak;TMng (1) nig (2) eKTaj)an


dUcenH

1 n 1

4
i 1 a i 3 2 i 1 a i
ai

eroberogeday lwm pln

Bit

n
1 n 1
a
4i
2 i 1ai i 1 a 3
i

- Page 118 -

vismPaBeRCIserIs
lMhat;TI73 IMO Shortlist 2009
eK[ a , b , c CacMnYnBitviCman edaydwgfa
1 1 1
a b c . cUrbgajfa
a b c
1
( 2a b c)

1
(a 2b c)

1
(a b 2c)

3
16

eKman (2a b c)2 4a2 4a(b c) (b c)2


4a 2 4ab 4ac 4bc (b c)2
4(a b )(a c) (b c)2

eday (b c)2 0 enaH (2a b c)2 4(a b)(a c)


1
1
eKTaj

(1)
2
4(a b )(a c)
( 2a b c)

RsaydUcKaEdreK)an
nig

1
( 2)
4(a b )(b c)

(a 2b c)2
1
1

( 3)
2
4(b c)(a c)
(a b 2c)

bUkvismPaB (1) , (2) nig (3) eK)an


1
1
1

4(a b )(a c) 4(a b )(b c) 4(b c)(a c)


abc
S
2(a b )(b c)(c a )
S

eroberogeday lwm pln

- Page 119 -

vismPaBeRCIserIs
tamsmtikmeKman a1 b1 1c a b c
eKTaj ab bc ac abc(a b c)
naM[ (ab bc ca)(a b c) abc(a b c)2
eday (ab bc ca)(a b c) 9abc
eKTaj abc(a b c)2 9abc enaH a b c 3

eroberogeday lwm pln

- Page 120 -

vismPaBeRCIserIs
lMhat;TI74 Turkey Team Selection Tests 2008

smIkar x3 ax2 bx c 0 manbbICacMnYnBitviCman


mincaM)ac;xusKa .
cUrkMNt;tmGb,brmaEdlGacn 13a2abbc bc .
tag u , v , w Cabsrbs;smIkar x3 ax2 bx c 0 .
eK)an

u v w a

uv vw wu b
uvw c

eday u 0 , v 0 , w 0 enaH a 0 , b 0 , c 0
1 a b c c b 2 ab 2ac b 3c
eKman 3 2a b b b2 2ab 3a
3b 2 3ab 6ac 3b 9c

3(b 2 2ab 3b )
(b 2 2ab 3b ) ( 2b 2 ab 6ac 9c)

3(b 2 2ab 3b )
1 2b 2 ab 6ac 9c

3
3(b 2 2ab 3b )

eroberogeday lwm pln

- Page 121 -

vismPaBeRCIserIs

tamvismPaB AM GM eKman
a u v w 3 3 uvw

nig b uv vw wu 33 u2 v2w 2
eK)an ab 9uvw 9c b ab 9c 0 (*)
ma:geToteKman
u 2 v 2 v 2w 2
v 2w 2 u 2w 2
2
uv w (1) ,
uvw 2 ( 2)
2
2
u 2w 2 u 2 v 2
u 2 vw ( 3)
2

bUkvismPaB (1) , (2) , (3) eK)an


u 2 v 2 v 2w 2 w 2u 2 uvw (u v w )
(uv vw wu )2 3uvw (u v w )

b b2 3ac b 2b2 6ac 0 (**)


bUkvismPaB (*) & (**) eK)an 2b2 ab 6ac 9c 0
1 2b 2 ab 6ac 9c 1 1 a b c c 1
ehtuenH 3 3(b2 2ab 3b) 3 b 3 2a b b 3
dUcenHtmGb,brman 13a2abbc bc KW 13 .
eroberogeday lwm pln

- Page 122 -

vismPaBeRCIserIs
lMhat;TI75 Turkey Team Selection Tests 2010

cUrRsayfa
2
2 2
2
(
a
b
)(
a
ab
b
) 2 2 2 2 1
1
1

4
(
a
b
c
)(
)

2
3
ab bc ca
Cyc

cMeBaHRKb;cMnYnBitviCman a , b , c .
tamvismPaB Cauchy Schwarz eKman
1
1
1
9

a b b c c a 2(a b c)
2 2
(a b 2 c 2 )
3
2 2
1
1
1
3(a2 b2 c2 )
2
2
(a b c )(

)
(1)
3
ab bc ca
abc

KuNGgTaMgBIrnwg

eK)an

tamvismPaB AM GM eKman

a2 b2
(a 2 ab b 2 )
2
2
2
2
(
a
b
)(
a
ab
b
)

2
2
3a 2 3b 2 2ab
(a 2 b 2 )(a 2 ab b 2 )

4
2

eK)an
Cyc

(a 2 b 2 )(a 2 ab b 2 )
3a 2 3b 2 2ab

2
2
Cyc

eroberogeday lwm pln

- Page 123 -

vismPaBeRCIserIs

tamvismPaB Cauchy Schwarz eK)an

Cyc

3a2 3b2 2ab


3(6a2 6b2 6c2 2ab 2bc 2ca)

2
2

eKTaj)an

Cyc

6[3(a2 b2 c2 ) (ab bc ca)]


(a2 b2 )(a2 ab b2 )

(2)
2
2

eyIgnwgRsayfa
3(a 2 b 2 c2 )
6[3(a 2 b 2 c2 ) (ab bc ca)]

abc
2

smmUl

(a b c)2[3(a2 b2 c2 ) (ab bc ca)]


a b c
6
2

2(a b c)2[9(a2 b2 c2 ) 3(ab bc ca)


a b c
6
xy
xy
, x , y 0
AM GM
2
2

tamvismPaB

eKman

x 2(a b c)2 , y 9(a 2 b 2 c 2 ) 3(ab bc ca)

ehIy x y 11(a2 b2 c2 ) ab bc ca
enaHeK)an
eroberogeday lwm pln

- Page 124 -

vismPaBeRCIserIs
11(a 2 b 2 c2 ) ab bc ca
a b c
12
2

a 2 b 2 c2 ab bc ca
1
[ (a b )2 (b c ) 2 (c a )2 ] 0
2

b
ehtuenHeK)an

Bit

3(a2 b2 c2 )
6[3(a2 b2 c2 ) (ab bc ca)]
(3)

abc
2

tamTMnak;TMng (1) , (2) nig (3) eKTaj)an

Cyc

(a2 b2 )(a2 ab b2 ) 2 2 2 2 1
1
1
)
(a b c )(

2
3
ab bc ca

vismPaBenHkayCasmPaBluHRtaEt a b c .

eroberogeday lwm pln

- Page 125 -

vismPaBeRCIserIs
lMhat;TI76 Turkey National Olympiad 2005

ebI a , b , c CargVas;RCugnRtIekaNmYy ehIy r CakaMrgVg;


carwkkgnRtIekaNenaHcUrRsayfa a12 b12 c12 41r 2 .
tag

b c a x

c a b y
a b c z

yz

zx

xy

2
4
4
4

(1)
2
2
2
(y z)
( z x)
(x y )

enaH

eK)an a12 b12 c12


tamrUbmnehr:ugeK)an

S p r p(p a)(p b )(p c)

eKTaj r 2 (p a)(p p b)(p c)

Edl p a b2 c

b r 2 (b c a)(4(caabbc)() a b c) 4(x xyzy z )


yz 1
1
1
eKTaj 41r 2 x xyz


yz zx xy
tamvismPaB AM GM eKman

eroberogeday lwm pln

( 2)

- Page 126 -

vismPaBeRCIserIs
y z 2 yz

naM[ (y 4z )2 yz1

dUcKaEdr (z 4x)2 zx1 nig (x 4y )2 xy1


eK)an (y 4z )2 (z 4x)2 (x 4y )2 yz1 zx1 xy1

( 3)

tamTMnak;TMng (1) , (2) & (3) eKTaj)an a12 b12 c12 41r 2 Bit
dUcenH a12 b12 c12 41r 2 .

eroberogeday lwm pln

- Page 127 -

vismPaBeRCIserIs
lMhat;TI77 Vietnam Team Selection Tests 2010

eK[ a , b , c CacMnYnBitviCmanehIyepgpat;lkxN
1 1 1
16(a b c) . cUrRsaybBaak;fa
a b c
1

1
8

3
3
(a b 2a 2c)3 (b c 2a 2b) (c a 2b 2c) 9
1 1 1
16(a b c)
a b c

tamsmtikmeKman
eK)an ab bc ca 16abc(a b c) (1)
tamvismPaB AM GM eKman

a2b2 b2c2 b2c2 c2a2 c2a2 a2b2

ab2c abc2 a2bc


2
2
2

a 2b 2 b 2c2 c2a 2 ab 2c abc 2 a 2bc abc(a b c)

EfmGgTaMgBIrnwg 2ab2c 2abc2 2a2bc 2abc(a b c)


eK)an (ab bc ca)2 3abc(a b c) (2)
tam (1) nwg (2) eKTaj)an
3
(ab bc ca)
16
3
ab bc ca
( 3)
16

(ab bc ca)2

b
m:ageTottamvismPaB AM GM eKman
eroberogeday lwm pln

- Page 128 -

vismPaBeRCIserIs
1
1
(a b )(a c)
(a b ) ( 2a 2c ) ( 2a 2c ) 3 3
2
2
2
27
(a b 2a 2c )3 (a b )(a c)
2
1
2
1

.
( 4)
3
27 (a b )(a c)
(a b 2a 2c )

b
eKTaj)an

RsaybMPWdUcKaEdreK)an
1
(b c 2a 2b )3
1
(c a 2b 2c )3

1
2
( 5)
.
27 (a b )(b c)

2
1
.
( 6)
27 (c a )(b c)

tag
T

1
1

(a b 2a 2c)3 (b c 2a 2b)3 (c a 2b 2c)3

bUkvismPaB (4) , (5) , (6) eK)an


T

2
2(a b c)
.
27 (a b )(b c)(c a )

eKman
(a b c)(ab bc ca) (a b )(b c)(c a ) abc (7 )

tamvismPaB AM GM eKman
(a b )(b c)(c a ) 2 ab .2 bc .2 ca 8abc ( 8)

eroberogeday lwm pln

- Page 129 -

vismPaBeRCIserIs

tam (7) nig (8) eKTaj)an


8
(a b )(b c)(c a ) (a b c)(ab bc ca)
9
abc
9
1
9 16
.
. 6
(a b )(b c)(c a ) 8 ab bc ca 8 3
3
ab bc ca
16
8
2
T 2 6
9
27

naM[

eRBaH
eKTaj)an
dUcenH
1

(a b 2a 2c)3

Bit

1
8

(b c 2a 2b)3 (c a 2b 2c)3 9

eroberogeday lwm pln

- Page 130 -

vismPaBeRCIserIs
lMhat;TI78

eK[ a , b , c CacMnYnBitviCmanEdl a b c 1 .
cUrbgajfa ab 2 bc 2 ca 2 83 .
1 c
1 a
1b
eday a b c 1 enaH 1 c2 (a b c)2 c2

(a b )(a b 2c)
(a b )(a c) (a b )(b c)
ab
ab

1 c2 (a b )(a c) (a b )(b c)

eK)an
tamvismPaB Cauchy Schwarz eK)an

1
1
4

(a b)(a c) (a b)(b c) (a b)(a c) (a b)(b c)


a b 2c
4

(a b)(b c)(c a) (a b)(a c) (a b)(b c)


1 c
4

(a b)(b c)(c a) (a b)(a c) (a b)(b c)


ab
ab(1 c)

(1)
2 4(a b )(b c )(c a )
1 c
bc
bc(1 a )

( 2)
2
4(a b )(b c)(c a )
1 a
ac
ac(1 b )

( 3)
2 4(a b )(b c )(c a )
1 b

eKTaj

RsaybMPWdUcKaEdreK)an
nig

eroberogeday lwm pln

- Page 131 -

vismPaBeRCIserIs

bUkvismPaB (1) , (2) & (3) eK)an

ab(1 c) bc(1 a ) ac(1 b )


4(a b )(b c)(c a )
1 c2 1 a2 1 b 2
ab
bc
ac
ab bc ca 3abc

( 4)
2
2
2 4(a b )(b c )(c a )
1 c
1 a
1b
ab

bc

ac

eKman (a b)(b c)(c a) (1 a)(1 b)(1 c)

(1 a )(1 b )(1 c) 1 (a b c) (ab bc ca) abc


ab bc ca abc

eK)an (a b)(b c)(c a) ab bc ca abc


b (a b)(b c)(c a) 4abc ab bc ca 3abc
ab bc ca 3abc
b 1 (a b)(4babc

c)(c a ) (a b )(b c)(c a )


eday (a b)(b c)(c a) 8abc
eK)an (aba bbc)(bcac)(c3abca) 1 48 32 (5)
tam (4) & (5) eK)an ab 2 bc 2 ca 2 83 Bit .
1 c
1 a
1b
dUcenH ab 2 bc 2 ca 2 83 .
1 c

1 a

eroberogeday lwm pln

1b

- Page 132 -

vismPaBeRCIserIs
lMhat;TI79 Indonesia Team Selection test 2010

eK[ a , b , c CabIcMnYnBitminGviCman nig x , y , z


CabIcMnYnBitviCmanedaydwgfa a b c x y z .
a3 b 3 c3
cUrRsaybBaak;fa 2 2 2 a b c .
x

a3

RsaybBaak;fa

b3
y

c3
z

abc

tamvismPaB AM GM eKman
a3

eKTaj

a3
x

xx 3

a3
x

.x.x

3a 2x (1)

RsaydUcKaEdr

b3
y

3b 2y ( 2)

nig

c3
z

3c 2z ( 3)

bUkvismPaB (1) , (2) & (3) eK)an


a3
x

b3
y

dUcenH

a3
x

c3
z

3(a b c) 2( x y z ) a b c

b3
y

c3
z

eroberogeday lwm pln

abc

Bit

- Page 133 -

vismPaBeRCIserIs
lMhat;TI80

eK[ a , b , c 0 . cUrRsaybBaak;fa
a 3 4(b 3 c 3 ) b 3 4(c 3 a 3 ) c 3 4( a 3 b 3 ) 9

bc
ca
ab
2

eKman b c (b c) 3bc(b c)
tamvismPaB AM GM eKman b c 2 bc
eKTaj bc b 2 c naM[ 3bc(b c) 43 (b c)
eK)an b c (b c) 43 (b c) 14 (b c)
eKTaj b c 4(b c )
b a b c a 4(b c )
3

a 3 4(b 3 c 3 )
a
naM[
1
(1)
bc
bc
b 3 4(c 3 a 3 )
b
dUcKaEdr
1
( 2)
ca
ca
c 3 4(a 3 b 3 )
c
1
( 3)
nig
ab
ab

edaybUkTMnakTMng (1) , (2) , (3) eK)an


eroberogeday lwm pln

- Page 134 -

vismPaBeRCIserIs
T 3

a
b
c

bc ca ab

Edl

a 3 4(b 3 c 3 ) b 3 4(c 3 a 3 ) c 3 4(a 3 b 3 )

bc
ca
ab

eyIgnwgRsayfa
tag

3
a
b
c

bc ca ab 2

bcm

ca n
ab p

eK)an b c c a a b m n p
naM[ a b c m 2n p
eKTaj

npm

mnp

mnp

eK)an

a
b
c
npm mnp mnp

bc ca ab
2m
2n
2p

a
b
c
1 n m p m n p

3
b c c a a b 2 m n m p p n

eroberogeday lwm pln

- Page 135 -

vismPaBeRCIserIs

tamvismPaB AM GM eK)an
n p
p m
n m
2; 2; 2
p n
m p
m n

eK)an b a c c b a a c b
eKTaj
dUcenH

T 3

3 9

2 2

1
2 2 2 3 3
2
2

Bit .

a 3 4(b3 c 3 ) b 3 4(c 3 a 3 ) c 3 4(a 3 b3 ) 9

bc
ca
ab
2

eroberogeday lwm pln

- Page 136 -

vismPaBeRCIserIs
lMhat;TI1
eKeGaybIcMnYnBitviCman a , b , c . cUrRsayfa
a3
b3
c3
3

.
3
3
3
8
(b c )

(c a )

(a b )

tamvismPaB AM GM eK)an
1 1
a3
3a

8 8 (b c)3 4(b c)

a3
3a
1

(1)
3
4(b c) 4
(b c )

dUcKaEdr
nig

b3
3b
1

( 2)
3
4(c a ) 4
(c a )

c3
3c
1

( 3)
3
4
(
a

b
)
4
(a b )

bUkvismPaB (1) , (2) & (3) eK)an


a3

b3

c3

3 a
b
c 3

3
3
3
4 b c c a a b 4
(b c) (c a) (a b)
3 a
b
c
3 3
(
)

4 bc ca ab 4 8
a
b
c
3

bc ca ab 2

eyIgnwgRsayfa
b

eroberogeday lwm pln

- Page 137 -

vismPaBeRCIserIs

tag

bcm

ca n
ab p

eK)an b c c a a b m n p
naM[ a b c m 2n p
eKTaj

npm

mnp

mnp

eK)an

a
b
c
npm mnp mnp

bc ca ab
2m
2n
2p

a
b
c
1 n m p m n p

3
b c c a a b 2 m n m p p n

tamvismPaB AM GM eK)an

n m
p m
n p
2; 2; 2
m n
m p
p n
a
b
c
1
3

2 2 2 3
bc ca ab
2
2

eK)an

eroberogeday lwm pln

Bit

- Page 138 -

vismPaBeRCIserIs
lMhat;TI82 IMO Longlist 1992
abc
A
a
,
b
,
c
eKkM
N
t;
t
ag

cMeBaHRKb;cMnYnBitviCman
3
3
.
1 1

a b c
3
A
cUrRsayfa G 14 43 . HA ?

G 3 abc

nig H 1

]bmafa GA 14 43 . HA Bit

1 3 3 A 3
3
smmUl A 4 G 4 . H .G

abc
abc 3 1
3
3
(
) abc .
.abc
3
3
4
4
1 1 1

a b c
27
9abc
1 1 1
(a b c)3 abc
(a b c)( )
4
4
a b c
4(a b c)3 27abc 9(a b c)(ab bc ca )

b
tamvismPaB AM GM eyIg)an
a b c 33 abc naM[ (a b c)3 27abc

eroberogeday lwm pln

(1)

- Page 139 -

vismPaBeRCIserIs
a2 b 2 b 2 c2 c2 a2

ab bc ca
2
2
2
a 2 b 2 c2 ab bc ca

EfmGgTaMgBIrnwg 2ab 2bc 2ca eK)an


(a b c)2 3(ab bc ca)
3(a b c)3 9(ab bc ca) ( 2)

bUkvismPaB (1) & (2) eK)an


4(a b c)3 27abc 9(a b c)(ab bc ca )
3

dUcenH GA 14 43 . HA

eroberogeday lwm pln

Bit

- Page 140 -

vismPaBeRCIserIs
lMhat;TI83 IMO 1969
eK[cMnYnBit x1 , x2 , y1 , y 2 , z1 , z 2 epgpat; x1 0 , x2 0
x1y1 z12 0 nig x 2y 2 z 2 2 0 .
cUrRsaybBaak;fa
8
( x1 x 2 )( y1 y 2 ) ( z1 z 2 )

1
x1y1 z1

tag a x1y1 z12 0 ; b x2y 2 z 22 0


nig c (x1 x2 )(y1 y 2 ) (z1 z 2 )2
eday x1 0 nig x2 0 enaH

a z 12
y1
0
x1

1
x2y 2 z 2

(*)

nig y 2 0

c ( x1y1 z12 ) ( x 2y 2 z 2 2 ) x1y 2 x 2y1 2z1z 2


c a b x1y 2 x 2y1 2z1z 2

eday a x1y1 z1 enaH

a z 12
x1
y1

nig

b z 22
x2
y2

b x2y 2 z 2

enaH

a z 12
b z 22
eK)an c a b y 2 ( y ) y1 ( y ) 2z1z 2
1
2
y
y
y
y
a b 2 a 1 b 2 z 1 2 2 z 1z 2 1 z 2 2
y1
y2
y1
y2

eroberogeday lwm pln

- Page 141 -

vismPaBeRCIserIs
y

y
y
y1
a b 2 a 1 b 2 z1
z 2
y2
y1
y2
y1

eday yy2 z1 yy1 z 2 0 enaHeKTaj)an


2
1

y
y
y
y
c a b 2 a 1 b eday 2 a 1 b 2 ab
y1
y2
y1
y2
enaH c a b 2 ab ( a b )2 (1)
eyIg]bmafavismPaB (*) BiteBalKW 8c a1 b1 Bit

eK)an c a8abb eday c (


eyIgnwgRsayfa (
smmUl (a b)(
a b 2 ab

a b )2

tamvismPaB (1)

8ab
ab
a b )2 8ab . tamvismPaB AM GM
a b )2

ehIy (

a b )2 4 ab

a b ) 8ab Bit
8
1
1

( x1 x2 )(y1 y 2 ) ( z1 z 2 )2 x1y1 z12 x2y 2 z 22

enaH (a b)(
dUcenH

vismPaBenHkaysmPaBluHRtaEt
y
y2
y
y
z1
z 2 1 b z1y 2 z 2y1 nig 2 a 1 b .
y
y
y
y
1

eroberogeday lwm pln

- Page 142 -

vismPaBeRCIserIs
lMhat;TI82
cUrRsaybBaak;fa ba ba 2(a b)( a1 b1 )
cMeBaHRKb;cMnYnBitviCman a nig b .
edayKuNGgTaMgBIrnvismPaBnwg ab eK)an
3

a2 3 b2

2(a b ) 2

tag x a nig y 3 b
eK)an x y 2(x y ) (*)
tamvismPaB AM GM eKman
x x y x y 3x y nig y x y
bUkvismPaBTaMgBIrenHGgnigGgeK)an
3

3 3

3 3

4 2

3 3

x 3 y 3 3x 2 y 4

x 6 4x 3 y 3 y 6 3x 4 y 2 3x 2 y 4

EfmGgTaMgBIrnvismPaBnwg x

y6

eK)an

2( x 6 2x 3 y 3 y 6 ) x 6 3x 4 y 2 3x 2 y 4 y 6
2( x 3 y 3 ) 2 ( x 2 y 2 ) 3

eKTaj x y
dUcenH ba
2

naM[ (*) Bit .


1 1
2(a b )( ) RKb; a 0 ; b 0 .
a b

3 2( x 3 y 3 )

eroberogeday lwm pln

- Page 143 -

vismPaBeRCIserIs
lMhat;TI84
eK[RtIekaN ABC mYymanmMukgCamMuRsYc . cUrRsayfa
3 2
cos A cos B cos C
.
2
tamvismPaB Cauchy-Schwartz eyIg)an
( cos A cos B cos C ) 2 3(cos A cos B cos C) (1)

tag T cos A cos B cos C

A
BC
BC
cos
2 cos
2
2
2
A
A
BC
1 2 sin 2 2 sin cos
2
2
2
BC
A
A
cos sin
cos
2
2
2 2

B
C
0B
; 0C
2
2
BC
BC
cos
1

2
4
2
4
A
A 3 1
A
3
T 1 2 sin 2 2 sin (1 2 sin ) 2
2
2 2 2
2
2
3
cos A cos B cos C ( 2)
2
1 2 sin 2

eRBaH
.
eday nig CamMuRsYcenaH
eKTaj
naM[
ehtuenH
b
tamTMnak;TMng (1) nig (2) eKTaj)an
( cos A cos B cos C ) 2

dUcenH

9
2

cos A cos B cos C

eroberogeday lwm pln

3 2
2

.
- Page 144 -

vismPaBeRCIserIs
lMhat;TI85
eK[ x ; y ; z CacMnYnBitviCmanEdl xyz 1 . cUrRsayfa
1
1
1
1

( x 1)2 y 2 1 ( y 1)2 z 2 1 ( z 1)2 x 2 1 2

eyIgman (x 1) y 1 x y 2x 2 eday x
eKTaj (x 1) y 1 2(xy x 1)
naM[ (x 1)21 y 2 1 12 . xy 1x 1
2

y 2 2xy

eKman xyz 1 enaHeKGacyk x ba ; y bc , z ac


Edl a 0 ; b 0 ; c 0 .
eK)an xy x 1 ac ba 1 a ba c
ehtuenH (x 1)21 y 2 1 12 . a ab c 1
RsaydUcKaEdreK)an
1
1
b
2
.

2
2
( y 1) z 1 2 a b c
1
1
c
3
.

( z 1)2 x 2 1 2 a b c

eFIVplbUkvismPaB (1) ; (2) nig (3) eK)an


1
1
1
1

( x 1)2 y 2 1 ( y 1)2 z 2 1 ( z 1)2 x 2 1 2

eroberogeday lwm pln

- Page 145 -

vismPaBeRCIserIs
lMhat;TI86 USAMO 1991
eK[ a ; b ; c CacMnYnBitviCmanEdl abc 1 . cUrRsayfa
a(b b ) b(c c ) c(a a ) 0 .
x2
y2
z2
eday abc 1 enaHeKGactag a y 2 ; b z 2 ; c x2
Edl x 0 ; y 0 ; z 0 .
vismPaBxagelIsmmUl
2

x2 y 4 y y 2 z4 z
z 2 x4 x
( 4 ) 2 ( 4 ) 2( 4 ) 0
2
z
x x y
y
y z
z x
x2y 2 x2 y 2z 2 y 2 z 2x2 z 2

4
4
0
4
xy
yz
zx
z
x
y
2x 2 y 2 2x 2 2y 2 z 2 2y 2 2z 2 x 2 2z 2

0
4
4
4
yz
zx
xy
z
x
y
2

xy yz yz zx zx xy
2 2 2 2 2 2 0
x x
y y
z
z
2

eroberogeday lwm pln

Bit

- Page 146 -

vismPaBeRCIserIs
lMhat;TI87
eK[ a1 , a2 , ..., an 0 nig x1 , x2 , ..., xn 0 .
cUrRsaytamkMeNInfa
a
a
a
(a a ... a )
.

...

x
x
x
x x .... x
2

tag

n
2

a1
a2
an
(a1 a 2 ... an )2
Tn

...

x1
x2
xn
x1 x 2 ... xn
2

cMeBaH n 1 eK)an
cMeBaH n 2 eK)an

a
a
T1 1 1 0 0
x1
x1

Bit

a1
a2
( a1 a 2 ) 2
T2

x1
x2
x1 x 2
2

( x1 x 2 )(a1 x 2 a 2 x1 ) x1x 2 (a1 a 2 )2

x1x 2 ( x1 x 2 )
2

( a1 x 2 a 2 x1 ) 2

0
x1x 2 ( x1 x 2 )

Bit

]bmafavaBitdl;tYTI k KW Tk 0 Bit
eyIgnwgRsayfavaBitdl;tYTI k 1 KW Tk 1 0 Bit
eKman Tk 0 kar]bmaxagelI
eK)an ax ax ... ax (ax ax ......ax )
2

eroberogeday lwm pln

- Page 147 -

vismPaBeRCIserIs
eKTaj

ak
(a1 a 2 ... ak )2
a1
a2

...

xk
x1 x 2 ... xk
x1
x2
ak2 1
xk 1
2

EfmGgTaMgBIrnwg

eK)an

ak
ak2 1 (a1 a 2 ... ak )2 ak2 1
a1
a2

...

xk xk 1
x1 x 2 ... xk
xk 1
x1
x2
(a1 a 2 ... ak )2 ak2 1 (a1 a 2 ... ak 1 )2

x1 x 2 ... xk
xk 1
x1 x 2 ... xk 1
2

eday
eKTaj)an

ak
ak2 1 (a1 a 2 ... ak ak 1 )2
a1
a2

...

xk
xk
x1 x 2 ... xk xk 1
x1
x2
2

naM[ Tk 1 0 Bit .
dUcenH ax ax ... ax
2

eroberogeday lwm pln

n
n

(a1 a 2 ... an )2

x1 x 2 .... xn

- Page 148 -

vismPaBeRCIserIs
lMhat;TI88
eK[ a ; b ; c CabIcMnYnBitviCmanEdl abc 1 .
cUrbgajfa
1
1
1
3

?
3
3
3
a ( b c ) b (c a ) c (a b ) 2
eKman
T

1
1
1

a 3 ( b c ) b 3 ( c a ) c 3 (a b )
2

1
1
1
1 1 1




a
b
c
a b c
T
a(b c) b(c a ) c(a b ) a(b c) b(c a ) c(a b )
2

1 1 1 ab bc ca
2
(ab bc ca)

abc

a b c

eday
ehIy a(b c) b(c a) c(a b) 2(ab bc ca)
)
3
eKTaj T ab bc2 ca 3 (abc

2
2
dUcenH a3 (b1 c) b3 (c1 a) c3 (a1 b) 32 .
3

eroberogeday lwm pln

- Page 149 -

vismPaBeRCIserIs
lMhat;TI89
eK[ x ; y ; z 0 . cUrRsaybBaak;fa
x
y
z
1

x 2y 3z y 2z 3x z 2x 3y 2
x
y
z
T

x 2y 3z y 2z 3x z 2x 3y
x2
y2
z2
T 2
2
2
x 2xy 3xz y 2yz 3xy z 2xz 3yz

tag

( x y z )2
T 2
x y 2 z 2 5( xy yz zx )
1
( x y z )2
1
T 2

2 x y 2 z 2 5( xy yz zx ) 2
1 1 ( x y )2 ( y z )2 ( z x )2
T
0
2
2
2
2
2 2 x y z 5( xy yz zx )
1
T
2
x
y
z
1

x 2y 3z y 2z 3x z 2x 3y 2

eK)an

eKTaj)an
dUcenH

eroberogeday lwm pln

- Page 150 -

vismPaBeRCIserIs
lMhat;TI90
eK[sIVt a1 ; a2 ; ....; an epgpat;lkxN
a1 0 ; | a 2 || a1 1 |; ... nig | an || an 1 1 | .
cUrbgajfa a a n ... a 12 .
eyIgman | an || an 1 1 |
naM[ an 2 an2 1 2an 1 1
n
n1
n 1
2
2
eK)an (ak ) (ak 1 2ak 1 1) (ak 2 2ak 1)
1

k 1

an2 1
n

eKTaj a

k 1

k 1

k 1

k 1

2
2
(a k ) ( a k ) 2 ( a k ) n

k 1

an2 1

2
a1 a 2 ... an
1

n
2

k 1

dUcenH

eroberogeday lwm pln

n
2

- Page 151 -

vismPaBeRCIserIs
lMhat;TI91
eK[ a ; b ; c CabIcMnYnBitviCman .
cUrbgajfa

a
b
c
abc

)
1 1 1 2 (1 3
b
c
a
abc

a
b
c

T 1 1 1
b
c
a

a b c b a c
2
b c a a c b
a a a b b b c c c
1
a b c a b c a b c
3a
3b
3c
3
3
3
1
abc
abc
abc
3(a b c)
2(a b c) a b c
1

3
1
3
3
abc
abc
abc
2(a b c)
abc
3
1
2
(
1
)

3
3
abc
abc
b
c
abc
a

)
1 1 1 2 (1 3
b
c
a
abc

tag

dUcenH

eroberogeday lwm pln

- Page 152 -

vismPaBeRCIserIs
lMhat;TI92
eK[ a ; b ; c CaRbEvgRCugrbs;RtIekaNmYy .
cUrbgajfa
abc bca cab a b c

eday a ; b ; c CaRbEvgRCugrbs;RtIekaNenaHeK)an
a b c 0 ; b c a 0 ;c a b 0

tamvismPaB Cauchy Schwartz eK)an


a b c b c a 2 b (1)
a b c c a b 2 a ( 2)
b c a c a b 2 c ( 3)

bUkvismPaB (1) ; (2) nig (3) eKTTYl)an


2( a b c b c a c a b ) 2( a b c )

dUcenH
abc bca cab a b c

eroberogeday lwm pln

- Page 153 -

vismPaBeRCIserIs
lMhat;TI93
eK[RtIekaN ABC mYymanmMukgCamMuRsYc . cUrbgajfa
sin 2 A sin 2 B sin 2 C 2 3 sin A sin B sin C .
tamRTwsIbTsIunUs
a
b
c

2R R :kaMrgVg;carwkeRkARtIekaN
sin A sin B sin C
eKTaj

a 2R sin A

b 2R sin B
c 2R sin C

(I )

tamRTwsIbTkUsIunUs
a 2 b 2 c 2 2bc cos A ( II )

yk

(I )

CYskg (II ) eK)an

4R 2 sin 2 A 4R 2 sin 2 B 4R 2 sin 2 C 8R 2 sin B sin C cos A


sin 2 A sin 2 B sin 2 C 2 sin B sin C cos A
sin 2 B sin 2 C sin 2 A
cos A
2 sin B sin C
2
sin B sin 2 C sin 2 A
cot A
(1)
2 sin B sin C sin A
sin 2 C sin 2 A sin 2 B
cot B
( 2)
2 sin C sin A sin B
sin 2 A sin 2 B sin 2 C
cot C
( 3)
2 sin A sin B sin C

eKTaj

ehtuenH

dUcKaEdr
ehIynwg

eroberogeday lwm pln

- Page 154 -

vismPaBeRCIserIs
bUkTMnak;TMng (1) ; (2) nig (3) eKTTYl)an
sin 2 A sin 2 B sin 2 C
cot A cot B cot C
( 4)
2 sin A sin B sin C
ABC AB C

b
ma:geToteKman
eK)an tan( A B) tan( C)

tan A tan B
tan C
1 tan A tan B
tan A tan B tan C tan A tan B tan C

eKTaj
KuNGgTaMgBIrnwg cot A cot B cot C eK)an

cot A cot B cot B cot C cot C cot A 1


AM GM
x; y;z0

tamvismPaB
eKman

cMeBaHRKb;

x 2 y 2 2xy
2
2
y z 2yz
2
2
z x 2zx

eK)an 2(x y z ) 2(xy yz zx)


b
x y z xy yz zx
EfmGgTaMgBIrnwg 2xy 2yz 2zx eK)an
2

( x y z )2 3( xy yz zx )

edayyk x cot A ; y cot B ; z cot C


eK)an (cot A cot B cot C) 3
2

eroberogeday lwm pln

- Page 155 -

vismPaBeRCIserIs
naM[ cot A cot B cot C 3
tamTMnak;TMng (4) nig (5) eK)an

( 5)

sin 2 A sin 2 B sin 2 C


3
2 sin A sin B sin C
sin 2 A sin 2 B sin 2 C 2 3 sin A sin B sin C

dUcenH

eroberogeday lwm pln

- Page 156 -

vismPaBeRCIserIs
lMhat;TI94
eK[ A ; B ; C CamMukgrbs;RtIekaN ABC mYy .
cUrbgajfa cot A2 cot B2 cot C2 3 3
eKman A B C naM[ A2 B2 2 C2
eK)an tan A2 B2 tan 2 C2

A
B
tan
2
2 cot C
A
B
2
1 tan tan
2
2
tan

A
B
tan
2
2 1
A
B
C
1 tan tan
tan
2
2
2
C
A
B
A
B
tan (tan tan ) 1 tan tan
2
2
2
2
2
C
A
A
B
B
C
tan tan tan tan tan tan 1
2
2
2
2
2
2
A
B
C
cot cot cot
2
2
2
C
A
B
C
A
B
cot cot cot cot cot cot
2
2
2
2
2
2
A B C
0 ; ;
0 A;B;C
2 2 2 2
tan

eK)an

KuNGgTaMgBIrnwg
eday

eroberogeday lwm pln

enaH

- Page 157 -

vismPaBeRCIserIs
eK)an cot A2 0 ; cot B2 0 ; cot C2 0
tamvismPaB AM GM eK)an
A
B
C
A
B
C
cot cot 3 3 cot cot cot
2
2
2
2
2
2
C
A
B
C
A
B
cot cot cot 3 3 cot cot cot
2
2
2
2
2
2

cot

A
B
C
A
B
C

cot cot cot 27 cot cot cot


2
2
2
2
2
2

A
B
C
cot cot cot 3 3
2
2
2

dUcenH

eroberogeday lwm pln

- Page 158 -

vismPaBeRCIserIs
lMhat;TI95
eK[ A ; B ; C CamMuRsYckgrbs;RtIekaN ABC mYy .
cUrbgajfa (1 tan A)(1 tan B)(1 tan C) (1 3 ) .
eday A ; B ; C CamMuRsYcenaH tan A 0; tan B 0; tan C 0
tamvismPaB AM GM RKb; x 0 ; y 0 ; z 0 eKman
3

(1 x )(1 y )(1 z ) 1 ( x y z ) ( xy yz zx ) xyz


(1 x )(1 y )(1 z ) 1 3 3 xyz 3 3 x 2 y 2 z 2 xyz
(1 x )(1 y )(1 z ) (1 3 xyz )3

yk x tan A ; y tan B ; z tan C eK)an


(1 tan A )(1 tan B)(1 tan C) (1 tan A tan B tan C )3
tan A tan B
tan( A B ) tan( C)
tan C
1 tan A tan B
tan A tan B tan B tan A tan B tan C

eKman
eKTaj

x y z xyz

tamvismPaB AM GM eK)an
x y z 3 3 xyz
xyz 3 3 xyz

eKTaj xyz 3 3 b tan A tan B tan C 3 3


eK)an (1 tan A)(1 tan B)(1 tan B) (1 3 3 3 )3
dUcenH (1 tan A)(1 tan B)(1 tan C) (1 3 ) .
3

eroberogeday lwm pln

- Page 159 -

vismPaBeRCIserIs
lMhat;TI96
eK[ a ; b ; c CabIcMnYnBitviCman . cUrRsayfa
(a 2)(b 2)(c 2) 9(ab bc ca) .
2

a 2 tan A

A , B , C]0 ; [
b 2 tan B
2

c 2 tan C
2
2
2
a
2
2
(
1
tan
A
)

cos 2 A

2
2
2
b
2
2
(
1
tan
B
)

cos 2 B

2
2
2
c
2
2
(
1
tan
C
)

cos 2 C

8
(a 2 2)(b 2 2)(c 2 2)
cos 2 A cos 2 B cos 2 C
T ab bc ca

eRCIserIs

Edl

eK)an
naM[
tag

T 2(tan A tan B tan B tan C tan C tan A )


2(sin A sin B cos C sin B sin C cos A sin C sin A cos B )
T
cos A cos B cos C
2 [ cos A cos B cos C cos( A B C) ]

cos A cos B cos C

vismPaB (1) smmUlnwg

8
18 [cos A cos B cos C cos( A B C)]

cos A cos B cos C


cos 2 A cos 2 B cos 2 C

eroberogeday lwm pln

- Page 160 -

vismPaBeRCIserIs
eKTaj)an
cos A cos B cos C cos A cos B cos C cos( A B C)

tag A B3 C .
tamvismPaB AM GM nig Jensen eyIg)an

4
9

cos A cos B cos C


3
cos A cos B cos C
cos
3

4
cos 3 (cos 3 cos 3)
9
cos 3 4 cos 3 3 cos
4
cos 3 ( 3 cos 3 cos 3 )
9
4
cos4 (1 cos 2 )
27
AM GM

eKTaj
eday
eK)an

tamvismPaB

eK)an

cos 2 cos 2
2

1
cos

2
2
cos cos
2

.
.(1 cos 2 ) 2

3
2
2

4
cos4 (1 cos 2 )
27
(a 2 2)(b 2 2)(c 2 2) 9(ab bc ca)

naM[
dUcenH

eroberogeday lwm pln

Bit .

Bit.

- Page 161 -

vismPaBeRCIserIs
lMhat;TI97
RbsinebI xyx (1 x)(1 y )(1 z ) Edl 0 x ; y ; z 1
cUrbgajfa x(1 z ) y(1 x) z(1 y ) 43 .
eKman (x 12 ) x x 14 0 cMeBaHRKb; x
eKTaj x(1 x) 14 eday 0 x 1 enaHeKTaj)an
1
0 x(1 x ) . RsaydUcKaEdreK)an
4
1
1
0 y (1 y )
0 z(1 z )
ni
g
4
4
eyIg)an xyz(1 x)(1 y )(1 z ) 641
eday xyz (1 x)(1 y )(1 z ) enaH xyz 641 naM[ xyz 18 .
tag T x(1 z ) y(1 x) z(1 y )
2

( x y z ) ( xy yz xz)

eday xyz (1 x)(1 y )(1 z )


b (x y z ) (xy yz zx) 1 2xyz
eKTaj T 1 2xyz 1 2 18 43 Bit

dUcenH

x(1 z ) y (1 x ) z(1 y )

eroberogeday lwm pln

3
4

- Page 162 -

vismPaBeRCIserIs
lMhat;TI98
eK[ a ; b ; c CaRbEvgRCugrbs;RtIekaNmYyEdlman
brimaRtesI 2 .
cUrRsayfa 32 a b c 2abc 2 .
edaybrimaRtrbs;RtIekaNenHesI 2 enaHRCugTaMgbI a ; b ; c
rbs;RtIekaNsuTEttUcCag 1 .
eyIg)an S 12 bc sin A 12
tamrUbmnehrug S p(p a)(p b)(p c) eday p 1
enaH S (1 a)(1 b)(1 c) 12
eKTaj 0 (1 a)(1 b)(1 c) 14
b 0 1 (a b c) (ab bc ca) abc 14
b 0 1 2 (ab bc ca) abc 14
b 1 (ab bc ca) abc 54
b 2 2(ab bc ca) 2abc 52
eKman (a b c) a b c 2(ab bc ca)
2

eroberogeday lwm pln

- Page 163 -

vismPaBeRCIserIs
eKTaj
a 2 b 2 c 2 2abc (a b c)2 2abc 2(ab bc ca)
a 2 b 2 c 2 2abc 4 2(ab bc ca) 2abc
5
2 2(ab bc ca) 2abc
2
5
4 a 2 b 2 c 2 2abc 4 2
2
3
a 2 b 2 c 2 2abc 2
2

eday
eK)an
dUcenH

eroberogeday lwm pln

- Page 164 -

vismPaBeRCIserIs
lMhat;TI99
eK[ x ; y ; z CabIcMnYnBitviCmanEdl x y z 1 .
cUrRsaybBaak;fa 1x 1 y1 1 1z 1 8 .

1
(1 x )(1 y )(1 z )
1
1
1 1 1
xyz

x
y
z
1 x y z

xyz 1
1 y x z
1 z x y

eKman
eday

naM[

x )( x y )
eK)an 1x 1 y1 1 1z 1 (y z )(zxyz

tamvismPaB AM GM eKman
y z 2 yz ; z x 2 zx nig x y 2 xy
eK)an (y z )(z x)(x y ) 8xyz
x )( x y )
8
naM[ (y z )(zxyz

dUcenH

1
1
1 1 1 8

x
y
z

eroberogeday lwm pln

- Page 165 -

vismPaBeRCIserIs
lMhat;TI100
eK[ a ; b ; c CabIcMnYnBitviCmanEdl a b c
1
1
1
2(a 3 b 3 c 3 )
bgajfa a2 b2 c2 3
abc
2

1
1
1
a3 b 3 c3
T 2 2 2 3 2.
abc
a
b
c
a2 b 2 c2 1

tag
eday

enaHeKTaj

1
b 2 c2 1
a2 c2 1
a2 b2
1
; 2 1
; 2 1
2
2
2
a
a
b
b
c
c2

eFIVviFIbUksmPaBTaMgenHeK)an

1
1
1
1
1
1 2 1
2 1
2 1

3
a
b
c

2
2
2
a2 b 2 c2
c2
a2
b2
b
c
a

kenSam T Gacsresr

1
1
1 2 1
1 a2 b 2 c2
2 1
T a 2 2 b 2 2 c 2 2 2

c
a
b bc ca ab
b
c
a
2

1 1
1 1
1 1
a b 2 c2 0
b c
c a
a b
1
1
1
2(a 3 b 3 c 3 )

3
abc
a2 b 2 c2

dUcenH

eroberogeday lwm pln

- Page 166 -

vismPaBeRCIserIs
lMhat;TI101
eK[ a , b , c CabIcMnYnBitviCman . cUrRsaybBaak;fa
a2
2a 2 ab b 2

b2
2b 2 bc c2

c2
2c2 ca a 2

abc
2

ebI a b c enaHvismPaBxagelIkaCasmPaB .
x2
RKb; x 0
eyIgBinitGnuKmn_ f (x) 2

2x x 1
vismPaBxagelIsmmUl bf ( ba ) cf ( bc ) af ( ac ) a b2 c

.
edIm,IRsaybBaak;vismPaBenHeyIgRtUvkMNt;GnuKmn_lIenEGmYyEdl
f ( x ) x (*) ehIy nig CaBIrcMnYnBitEdlbMeBjlkxN (*)
BitcMeBaHRKb; x 0 .
edaysaEtvismPaBBitcMeBaH a b c enaHeyIgnwgkMNt;rk nig
EdleFIV[ExSekag (c) : y f (x) b:Hnwg (d) : y x Rtg; x 1
eBalKWRtUv[ f (1) nig f ' (1) .
eday f (1) 12 enaH 12
eKman f ' (x)

2x 2x 2 2x 1

eroberogeday lwm pln

4x 1

2 2x 2 2x 1
2x 2 2x 1

.x 2

- Page 167 -

vismPaBeRCIserIs
5
11
11
eK)an f ' (1) 4 4 16
enaH 16
ehIy 12 163
11x 3
x2
11x 3

ehtuenH f (x) 16 b 2
(**)
16
2x x 1
-ebI 0 x 113 enaHvismPaB (**) BitCaniceRBaHGgTI1CakenSamviCman
CanicRKb; x 0 ehIyGgTIBIrGviCman .
-ebI x 113 enaHvismPaB (**) Gacsresr
2
2
2
2
2

x
(
11
x

3
)
(
x

1
)
(
14
x
39x 9)

0
2

2
256
256( 2x x 1)
2x 2x 1
vaBitcMeBaHRKb; x 113 eRBaH 14x2 39x 9 0 .
tam (**) eKCMnYs x eday ba , bc , ac eK)an
a
b
c 11a 3b 11b 3c 11c 3a
bf ( ) cf ( ) af ( )
b
c
a
16
8a 8b 8c a b c

Bit
16
2
4

dUcenH

a2
2a 2 ab b 2

eroberogeday lwm pln

b2
2b 2 bc c2

c2
2c2 ca a 2

abc
2

- Page 168 -

vismPaBeRCIserIs
lMhat;TI102
eK[bIcMnYnBitviCman a , b , c . cUrRsaybBaak;fa
29a 3 b 3 29b 3 c3 29c3 a 3

4(a b c)
2
2
2
ab 6a
bc 6b
ca 6c
29x 3 1
eyIgBinitGnuKmn_ f (x)
RKb; x 0
2
x 6x
vismPaBxagelIsmmUl bf ( ba ) cf ( bc ) af ( ac ) 4(a b c)

edIm,IRsaybBaak;vismPaBenHeyIgRtUvkMNt;GnuKmn_lIenEGmYyEdl
f ( x ) x (*) ehIy nig CaBIrcMnYnBitEdlbMeBjlkxN (*)
BitcMeBaHRKb; x 0 .
edaysaEtvismPaBBitcMeBaH a b c enaHeyIgnwgkMNt;rk nig
EdleFIV[ExSekag (c) : y f (x) b:Hnwg (d) : y x Rtg; x 1
eBalKWRtUv[ f (1) nig f ' (1) .
1
eKman f (1) 29
4 enaH 4
1 6
87 x 2 ( x 6x 2 ) (1 12x )( 29x 3 1)
ehIy f ' (x)
( x 6x 2 )2
eK)an f ' (1) 87 7 213 28 87 7 52 5 enaH 5
7
ehIy 4 4 5 1 .

eroberogeday lwm pln

- Page 169 -

vismPaBeRCIserIs
29x 3 1
eK)anvismPaB
5x 1 (**)
2
x 5x
29x 3 1
( x 1)2 ( x 1)
eKman
(5x 1)
0 x 0
2

x
(
1
6
x
)
x 6x

naM[vismPaB (**) BitCanicRKb; x 0 .


edayCMnYs x ba , bc , ac kg (**) eK)an
a
b
c
bf ( ) cf ( ) af ( ) (5a b ) (5b c) (5c a )
a
b
c
4a 4b 4c
4(a b c)
29a 3 b 3 29b 3 c3 29c3 a 3
4(a b c)
dUcenH

2
2
2
ab 6a
bc 6b
ca 6c

eroberogeday lwm pln

- Page 170 -

vismPaBeRCIserIs
lMhat;TI103
eK[ a , b , c CabIcMnYnBitviCman . cUrRsaybBaak;vismPaB
a3
b3
c3
abc
.

2
2
2
2
2
2
3
a ab b
b bc c
c ca a
x3
eyIgBinitGnuKmn_ f (x) 2
RKb; x 0
x x1
vismPaBxagelIsmmUl bf ( ba ) cf ( bc ) af ( ac ) a b3 c .

edIm,IRsaybBaak;vismPaBenHeyIgRtUvkMNt;GnuKmn_lIenEGmYyEdl
f ( x ) x (*) ehIy nig CaBIrcMnYnBitEdlbMeBjlkxN (*)
BitcMeBaHRKb; x 0 .
edaysaEtvismPaBBitcMeBaH a b c enaHeyIgnwgkMNt;rk nig
EdleFIV[ExSekag (c) : y f (x) b:Hnwg (d) : y x Rtg; x 1
eBalKWRtUv[ f (1) nig f ' (1) .
eKman f (1) 13 enaH 1 b 1
3x 2 ( x 2 x 1) x 3 ( 2x 1)
ehIy f ' (x)
( x 2 x 1)2
eK)an f (1) 9 9 3 23 naM[ 23 ehIy 1 23 13 .
x3
2x 1
eK)anvismPaB 2
(**)

3
x x1

eroberogeday lwm pln

- Page 171 -

vismPaBeRCIserIs
eKman

x3
2x 1 ( x 1)2 ( x 1)

0
2
2
3
x x1
3( x x 1)

RKb; x 0

enaHmannyfavismPaB (**) BitCanicRKb; x 0 .


edayCMnYs x ba , bc , ac kg (**) eK)an

a
b
c 2a b 2b c 2c a
bf ( ) cf ( ) af ( )
b
c
a
3
abc

Bit
3

dUcenH

a3
b3
c3
abc
.

2
2
2
2
2
2
3
a ab b
b bc c
c ca a

eroberogeday lwm pln

- Page 172 -

vismPaBeRCIserIs
lMhat;TI104 Turkey 2007
eK[bIcMnYnBitviCman a , b , c Edl a b c 1 . cUrRsaybBaak;fa
1
1
1
1

ab 2c2 2c bc 2a 2 2a ca 2b 2 2b ab bc ca

CadMbUgeyIgnwgRsayfa

1
ab

ab 2c2 2c (ab bc ca)2

smmUl (ab bc ca)2 ab(ab 2c2 2c)


smmUl b2c2 c2a2 2abc(a b c) 2abc2 2abc
eday a b c 1 enaH b2c2 c2a2 2abc 2abc2 2abc
smmUl b2c2 c2a2 2abc2 c2 (a b)2 0 Bit
1
ab
(1)

ehtuenH
2
2
ab 2c 2c (ab bc ca)
1
bc

( 2)
2
2
bc 2a 2a (ab bc ca)
1
ca

( 3)
2
2
ca 2b 2b (ab bc ca)

bUkvismPaB (1) , (2) & (3) eK)an


1
1
1
1

ab 2c 2 2c bc 2a 2 2a ca 2b 2 2b ab bc ca

eroberogeday lwm pln

- Page 173 -

vismPaBeRCIserIs
lMhat;TI105
eK[RtIekaN ABC mYymanRCug BC a , CA b , AB c
ehIymMukg A , B , C CamMuRsYcbmMuEkg .
tag S CapRkLan ABC
cUrRsaybBaak;fa 14 14 14 9 2 .
a

16 S

eKman S2 p(p a)(p b)(p c) Edl p a b2 c


tag x b2 c2 a2 , y c2 a2 b2 , z a2 b2 c2
Edl x , y , z 0 nig minGacmanBIrsUnRBmKa .
eK)an x y 2c2 , y z 2a2 , z x 2b2
ehIy 16S2 (a b c)(b c a)(c a b)(a b c)
4b 2c 2 (b 2 c 2 a 2 )2
( x y )( z x ) x 2 xy yz zx
vismPaB 14 14 14 9 2 GacbEmgeTACa
a
b
c
16 S
4
4
4
9

( x y )2 ( y z )2 ( z x )2 xy yz zx
1
1
1
9
( xy yz zx ) [
]

4
( x y )2 ( y z )2 ( z x )2

tamlkNHGUm:UEsneKGaceRCIserIsyk xy yz zx 1 enaHivsmPaB
eroberogeday lwm pln

- Page 174 -

vismPaBeRCIserIs
xagelIsmmUl 4 (x y )2 (x z )2 9(x y )2 (y z )2 (x z )2
cyc

eday (x y )(x z ) x2 xy xz yz x2 1
ehIy (x y )(y z )(x z ) (x2 1)(y z )
x2 (y z ) y z
x( xy xz) y z
x(1 yz ) y z
x y z xyz

eK)an 4 (x2 1)2 9(x y z xyz)2


cyc

4[( x 2 1)2 ( y 2 1)2 ( z 2 1)2 ] 9( x y z xyz )2

4[(x4 y 4 z 4 ) 2(x2 y 2 z 2 ) 3] 9(x y z xyz)2 (*)

tag S x y z nig P xyz


Edl S a2 b2 c2 0 nig P 0 eRBaH x , y , z 0 .
eK)an S2 (x y z )2 x2 y 2 z 2 2
ehIy (S2 2)2 x4 y 4 z 4 2(x2y 2 y 2z 2 y 2z 2 )
eday xy yz xz 1 enaH (xy yz xz)2 1
b x2y 2 y 2z 2 x2z 2 2xyz(x y z ) 1
b x2y 2 y 2z 2 x2z 2 1 2S P
enaH (S2 2)2 x4 y 4 z 4 2(1 2S P)
eroberogeday lwm pln

- Page 175 -

vismPaBeRCIserIs
naM[ x4 y 4 z 4 S4 4S2 4SP 2
vismPaB (*) xagelIsmmUl
4(S 4 4S 2 4SP 2 2S 2 4 3) 9(S P )2
4(S 4 2S 2 4SP 1) 9(S P )2
4S 4 8S 2 16SP 4 9(S P )2
9S 2 (4S 2 1)(S 2 4) 16SP 9(S P )2 (**)

-cMeBaH S 2 eKman (4S2 1)(S2 4) 16SP 0


naM[vismPaB (**) Bit
eRBaH 9S2 (4S2 1)(S2 4) 16SP 9S2 9(S P)2 .
-cMeBaH 0 S 2 vismPaB (**) Gacsresr
9S 2 ( 4S 2 1)(S 2 4) 16SP 9S 2 18SP 9P 2
(4S 2 1)(S 2 4) 34SP 9P 2 0

tag T (4S2 1)(S2 4) 34SP 9P 2


eyIgnwgRsayfa T 0 .
eKman (x y z )(xy yz xz) 9xyz tam AM GM
eday xy yz zx 1 enaH S 9P
ehIy T (4S2 1)(S2 4) P(S 9P ) 33SP
eK)an T (4S2 1)(S2 4) 33SP
eroberogeday lwm pln

- Page 176 -

vismPaBeRCIserIs
tamvismPaB Schur eKman x2 (x y )(x z ) 0
cyc

eday x2 (x y )(x z ) x4 x2yz x3 (y z )


eK)an ( x4 ) xyz ( x) x3 (y z )
cyc

cyc

cyc

eday (x4 ) S4 4S2 4SP 2

, xyz x SP

cyx

cyc

nig x3 (y z ) x2 (xy xz) x2 (1 yz )


cyc

cyc

cyc
2

x2 xyz x S
cyc

2 SP

cyc

eK)an S4 4S2 5SP 2 S2 SP 2


6SP S 4 5S 2 4
6SP (4 S 2 )(S 2 1)
ehtuenH T (4S2 1)(S2 4) 112 6SP
11
T (4S 2 1)(S 2 4) (4 S 2 )(S 2 1)
2
3
T (4 S 2 )(S 2 3)
2
eday 0 S 2 enaH 4 S2 0

ehIy S2 (x y z )2 3(xy yz zx) 3 enaH S2 3 0


eroberogeday lwm pln

- Page 177 -

vismPaBeRCIserIs
eKTaj)an T 32 (4 S2 )(S2 3) 0 Bit .
srubmkeK)an (xy yz zx) [
dUcenH

1
1
1
9
4 4
4
a
b
c
16 S 2

1
1
1
9
]

2
2
2
4
(x y )
(y z )
( z x)

smal; ehtuGIV)anCakgkarRsaybBaak;vismPaB
( xy yz zx ) [

1
1
1
9

(*)
2
2
2
4
(x y )
(y z )
( z x)

eKGaceRCIserIsyk xy yz zx 1 ?
eRBaHfaebIeKyk xy yz zx t 2 0
ehIytag x t.u , y t.v , z t. w enaHvismPaB (*) smmUl

9
1
1
1
t

2
2
2
(tv tw )
(tw tu ) 4
(tu tv )
9
1
1
1
2
2
2

t 2
2
2
2
t (v w)
t (w u) 4
t (u v )
1
1
1
9

(**)
2
2
2
4
(u v )
(v w)
(w u)
2

mannyfavismPaB (**) dUcKanwgvismPaB (*) eBl xy yz zx 1 .


eroberogeday lwm pln

- Page 178 -

vismPaBeRCIserIs
lMhat;TI106>
cUrbgajfa

1
1
1
4

( x y )2 ( y z )2 ( z x )2 xy yz zx

cMeBaHRKb;cMnYnBitminGviCmanxusKa x , y , z .
eday x , y , z CacMnYnBitxusKa nig minGviCmanenaHeKGacsntyk
zyx0 .
tag A 1 2 1 2 1 2
(x y )

(y z )

(z x)

nig B xy yz zx .
yk y x p , z x p q Edl p 0 , q 0
eK)an A 12 12 1 2
p

(p q )

nig B x(x p) (x p)(x p q) x(x p q)


3x 2 2( 2p q )x p 2 pq

edaysarEt x 0 enaH B p2 pq p(p q)


1
1
1
eK)an A B 2 2
p(p q )
2
p

eroberogeday lwm pln

(p q )

- Page 179 -

vismPaBeRCIserIs

p q p( p q )
p

p
pq
q2

q
q p(p q )
1

pq
p
q2

q
q
p( p q )
)
2(
p pq
q2
q2
p(p q )
2

p(p q )
q2
q2
p( p q )
q2
eday p(p q) 2 2 p(p q) . p(p 2 q) 2
q
q
eKTaj)an A B 2 2 4 naM[ A B4
1
1
1
4

dUcenH
.
2
2
2
xy

yz

zx
(x y )
(y z )
(z x)
3x 2 2( 2p q )x 0
vismPaBenHkayCasmPaBluHRtaEt
p(p q ) q 2
eKTaj)an x 0 nig p(p q) q2 ehIy y p , z p q

enaH z y q tam p(p q) q2 eKTaj)an yz (z y )2


naM[ z 2 3yz y 2 0 eday z y enaH z 3 5 1
y

eroberogeday lwm pln

- Page 180 -

vismPaBeRCIserIs
lMhat;TI107
1
1

cUrbgajfa 1 1 1 1

1
1

a b c d
ac bd
cMeBaHRKb;cMnYnBitviCman a , b , c , d .

eKman
X

nig

1
1 1

a b
Y

1
1 1

c d
1

ab
cd
abc abd acd bcd

ab cd
(a b )(c d )

(a c)(b d )
abcd

1
1

ac bd
(a c)(b d ) abc abd acd bcd

YX
abcd
(a b )(c d )

eK)an
bnab;BItRmUvPaKrYm rYcsRmYleK)an

(ad bc )2
0 naM[ Y X
YX
(a b )(c d )(a b c d )
dUcenH 1 1 1 1 1 1 1 1 1 .

ac bd
a b c d

eroberogeday lwm pln

- Page 181 -

vismPaBeRCIserIs
lMhat;TI108 (USAMO 2003)
eK[ a , b , c CacMnYnBitviCman . cUrRsaybBaak;fa
( 2a b c)2
( 2b c a )2
( 2c a b )2
8
2
2
2
2
2
2
2a (b c)
2b (c a )
2c (a b )

rebobTI1
tag

( 2a b c)2
( 2b c a )2
( 2c a b )2
T 2
2
2
2
2
2a (b c)
2b (c a )
2c (a b )2

yk s a b c enaHkenSam T Gacsresr
(a s )2
(b s ) 2
(c s )2
T 2
2
2
2
2
2a (a s )
2b (b s )
2c (c s )2

eKman

(a s )2
a 2 2as s 2
2
2
2
2a (a s )
3a 2as s 2
1 3a 2 6as 3s 2
. 2
3 3a 2as s 2

1
8as 2s 2
(1 2
)
2
3
3a 2as s
(a s )2
1
8as 2s 2
1
8as 2s 2

2
2
2
2
3
3 ( 3a s )2 2s 2
2a (a s )
9a 6as 3s

eday (3a s)2 0 enaH (3a s)2 2s2 2s2


4a
4a
8as 2s 2
8as 2s 2

b
1
1
2
2
2
abc
s
( 3a s ) 2s

eroberogeday lwm pln

2s

- Page 182 -

vismPaBeRCIserIs
eKTaj

(a s )2
4
4a

(1)
2
2
3 abc
2a (a s )

RsaybMPWdUcKaxagelIEdreK)an
(b s )2
4
4b

( 2)
2
2
3 abc
2b (b s )
(c s )2
4
4c

( 3)
2
2
3 abc
2c (c s )

bUkvismPaB (1) , (2) nig (3) eK)an

4 4 4 4a 4b 4c

44 8
3 3 3
abc
( 2a b c)2
( 2b c a )2
( 2c a b )2
dUcenH 2
2
2
8
2
2
2
2a (b c)
2b (c a )
2c (a b )
T

rebobTI2
tag

( 2a b c)2
( 2b c a )2
( 2c a b )2
T 2
2
2
2
2
2a (b c)
2b (c a )
2c (a b )2

yk x a b , y b c , z c a
eK)an

x z 2a b c

x y 2b c a
z y 2c a b

kenSam T GacsresrCa
eroberogeday lwm pln

nig

2a x z y

2b x y z
2c z y x

- Page 183 -

vismPaBeRCIserIs
2( x z )2
2( z y )2
2( y x )2
T

2
2
2
2
( x z y ) 2y
( z y x ) 2x
( y x z )2 2z 2

tamvismPaB Cauchy Schwarz 2(u2 v 2 ) (u v )2


eK)an 2( x z y )2 2y 2 ( x z y y )2 (x z )2
2( x z y )2 4y 2 ( x z )2 2y 2
1
( x z y )2 2y 2 ( x z )2 y 2
2
4
2( x z )2
2( x z )2

eKTaj
( x z y )2 2y 2 1 ( x z )2 y 2
2y 2
1
2
( x z )2
2y 2
y2
2
2
2
2
eday (x z ) 2(x z ) naM;[
2
(x z)
x z2

2y 2
x2 y 2 z 2
1

2
(x z )
x2 z 2

eKTaj)an

2( x z )2
4( x 2 z 2 )
(1)
2
2
2
2
2
( x z y ) 2y
x y z

RsaybMPWdUcxagelIEdreK)an

eroberogeday lwm pln

- Page 184 -

vismPaBeRCIserIs
2( z y )2
4( z 2 y 2 )
2
( 2)
2
2
2
2
( z y x ) 2x
x y z
2( y x )2
4( y 2 x 2 )
2
( 3)
2
2
2
2
( y x z ) 2z
x y z

bUkvismPaB (1) , (2) , (3) eK)an


4( x 2 z 2 ) 4( z 2 y 2 ) 4( y 2 x 2 )
T
8
2
2
2
x y z
( 2a b c)2
( 2b c a )2
( 2c a b )2
dUcenH 2
2
2
8
2
2
2
2a (b c)
2b (c a )
2c (a b )

rebobTI3
tag

( 2a b c)2
( 2b c a )2
( 2c a b )2
T 2
2
2
2
2
2a (b c)
2b (c a )
2c (a b )2

eKmansmPaB (2u v )2 2(u v )2 3(2u2 v 2 )


edayyk u a nig v b c enaHeK)an

( 2a b c)2 2(a b c)2 3 2a 2 (b c)2

b (2a b c)2 3 2a2 (b c)2 2(a b c)2


EckGgTaMgBIrnwg 2a2 (b c)2 eK)an
2(a b c)2
( 2a b c)2
eyIg)an 2
3 2
2
2
2a (b c)

2a (b c)

kenSam T GacbMElgCa
eroberogeday lwm pln

- Page 185 -

vismPaBeRCIserIs
(a b c )2
(b a c ) 2
(c a b )2
2
2
T 92 2
2
2
2
2b (c a )
2c (a b )
2a (b c)
edIm,IRsayfa T 8 eyIgRtUvRsay[eXIjfa
(a b c )2
(b a c ) 2
(c a b )2
1
S 2

2a (b c)2 2b 2 (c a )2 2c2 (a b )2 2

eKman 2a2 (b c)2 2a2 b2 c2 2bc


eday 2bc b2 c2 enaH 2a2 (b c)2 2(a2 b2 c2 )
RsaydUcKaEdr 2b2 (c a)2 2(a2 b2 c2 )
ehIy 2c2 (a b)2 2(a2 b2 c2 )
(a b c )2 (b a c )2 (c a b ) 2
eKTaj S
2
2
2
2(a b c )

edaykenSam (a b c)2 (b a c)2 (c a b)2


esInwg 3(a2 b2 c2 ) 2(ab bc ca)
enaH S 32 ab2 bc2 ca2

a b c
a2 b 2 b 2 c2 c2 a2
eKman 2 2 2 ab bc ca
b a2 b2 c2 ab bc ca naM[ S 32 1 12 Bit .

eroberogeday lwm pln

- Page 186 -

vismPaBeRCIserIs

rebobTI4
( 2a b c)2
( 2b c a )2
( 2c a b )2
Rsayfa 2
2
2
8
2
2
2
2a (b c)
2b (c a )
2c (a b )
tag x b a c , y c b a , z a c b

Edl

1
1
1
a
b
c

1
x1 y 1 z1 abc abc abc

vismPaBsmmUl

( 2 x )2 ( 2 y )2 ( 2 z )2
8

2
2
2
2 x
2 y
2 z

( 2 u )2 8
1
1
eyIgnwgRsayfa

RKb; u 0
2
3
1
u
3

2u
( 2 u )2 8
1
1
(u 5)(u 2)2
0 Bit
eKman


2
2
3
1

u
3
2u
3( 2 u )(1 u )
( 2 u )2
1
1 8

(*)
2
1 u 3 3
2u

ehtuenH
tam (*) eK)an

( 2 x )2 ( 2 y )2 ( 2 z )2
1
1
1

1 8

2
2
2
1

x
1

y
1

z
2 x
2 y
2 z

dUcenH

( 2a b c)2
( 2b c a )2
( 2c a b )2
8
2
2
2
2
2
2
2a (b c)
2b (c a )
2c (a b )

eroberogeday lwm pln

- Page 187 -

vismPaBeRCIserIs
lMhat;TI109 ( Baltic Way 2010 )

0
x
x
CacM
n
Y
n
Bi
t
edaydw
g
fa

.
eKyk
2
cUrbgajfa cos2 (x) cot(x) sin2 (x) tan(x) 1 .
x
sin x
tan x
edayCMnYs cot x cos
ni
g
vi
s
mPaBsmmU
l
sin x
cos x
cos 3 x sin 3 x

1 ehIy sin x 0 , cos x 0 x (0, )


sin x
cos x
2
tamvismPaB AM GM eK)an
cos 3 x cos 3 x

sin 2 x 3 cos2 x
sin x
sin x
cos 3 x 3 cos 2 x sin 2 x
b sin x
(1)
2
sin 3 x sin 3 x
ehIy cos x cos x cos2 x 3 sin2 x
sin 3 x 3 sin 2 x cos 2 x
( 2)
b cos x
2
bUkvismPaB (1) nig (2) GgnigGgeK)an
cos 3 x sin 3 x

cos 2 x sin 2 x 1 Bit


sin x
cos x
dUcenH cos2 (x) cot(x) sin2 (x) tan(x) 1 .

eroberogeday lwm pln

- Page 188 -

vismPaBeRCIserIs
lMhat;TI110 (China National Olympiad 2005)
21
nigcMeBaH n 2 : 2an 3an1 n31
sIVt {an } kMNt;eday a1 16
2
eKyk m CacMnYnKt;mYyEdl m 2 . cUrbgajfacMeBaH n m
n ( m 1)

m2 1
3

2 m
eyIg)an an n 3 m 3
mn1 ?

1
n ( m 1)
m2 1
3 m

2 m
bgajfa an n 3 m 3
mn1

eKman 2an 3an 1 n31


2
naM[ 2n an 3.2n 1 an 1 43
n
n 1
2
2
b 3 an 3 an 1 43 1n


3
k 1
n 2 k
3 n 1

2
eK)an 3 ak 3 ak 1 4 k

k 2
k 2 3


n 1
1
1
n
2
2
3
1

3
an a1 2 .
1
3
4 3
3
1
3
1
m

eroberogeday lwm pln

- Page 189 -

vismPaBeRCIserIs
n

2 21 1
1
2
an . 1 n 1
3 16 8
3
3
n
n
3
3
3
3
eKTaj)an an 2 n 3 b an n 3 2


2
2
1
n ( m 1)
tag P an n3 3 m m 23 m

n
n ( m 1)
3 m
2 m
eK)an P 2 m 3

1)
m2 1
m 1

eKman m n 1 (m(m 1)(1m


n
)n
1
m1
m2 1
edIm,IRsay[eXIjfa P m n 1
n
eyIgRtUvRsay[eXIjfa (1 m 1) P m 1
tamvismPaB Bernoully eKman

n
n
1 n m
1

1
)
(1

1
m1
m1
m 1
1

eroberogeday lwm pln

- Page 190 -

vismPaBeRCIserIs
mn

naM[ 1 mn 1 1 1 11

1
(1 )m

m
m
cMeBaHRKb; m 2 tameTVFajtun eKman
1

1
m

enaH 1 m1

1 1
1 2
1
Cm 2 Cm
.... m Cm
m
m
m
m

1
1 2 5 1
9 3
1 C1m 2 Cm


m
2 2m 4 2
m

eKTaj)an 1 mn 1

2n
2m

eKTaj 1 mn 1 P 3

2n

2n
2 m

n

m 1 3

n ( m 1)

Et P 2 m 23 m

n
n
( m 1)

n
2
2
eK)an (1 m 1) P 3 m m 3 m

n
n
( m 1)

2 m
2 m
]bmafa 3 m 3
m 1 Bit

n
3 m

eroberogeday lwm pln

- Page 191 -

vismPaBeRCIserIs
n
2 m

edaytag u 3 enaH u(m um 1 ) m 1



smmUl mu um m 1 0
m(u 1) (um 1) 0
(u 1)[m (um 1 .... u 1)] 0
n
2 m

eday m n & m 2 enaH 0 u 3 1



naM[ (u 1)[m (um 1 .... u 1)] 0 Bit
dUcenH a 3 m 2 m 1 .
n ( m 1)
m

1
m

2n 3

eroberogeday lwm pln

mn1

- Page 192 -

vismPaBeRCIserIs
lMhat;TI111> (Croatia Team Selection Tests 2011)
eK[ a , b , c CacMnYnBitviCmanEdl a b c 3 .
a2
b2
c2
3
cUrRsaybBaak;vismPaB

2
2
2
2

ab
bc
ca
a2
a(a b 2 ) ab 2
ab 2
eKman
a
(1)

2
2
2
ab
ab
ab
b2
bc 2
c2
ca2
dUcKaEdr 2 b 2 (2) ; 2 c 2 (3)
bc
bc
ca
ca
a2
b2
c2
tag S 2 2 2 . bUkvismPaB (1), (2) & ( 3)
ab
bc
ca
ab 2
bc 2
ca2
eK)an S 3 ( 2 2 2 )
ab
bc
ca
edIm,IRsayfa S 32 enaHeyIgnwgRsayfa
ab 2
bc 2
ca2
3 3

.
3
S
3
2
2
2
2 2
ab
bc
ca
2ab 2
2
tamvismPaB AM GM eKman a b 2b a b a
ab 2
b a
a c
bc 2
c b ca2
eKTaj

ehI
y

,
2
2
2 c a2
a b2
b c2
ab 2
bc 2
ca2
1

(a c b a c b ) (*)
ehtuenH
2
2
2
2
ab
bc
ca

eroberogeday lwm pln

- Page 193 -

vismPaBeRCIserIs
tamvismPaB Cauchy Schwarz eKman
(a c b a c b )2 (a b c)(ab bc ca )

(a c b a c b )2 3(ab bc ca)

ehIy (ab bc ca)2 (a2 b2 c2 )(b2 c2 a2 )


(ab bc ca)2 (a 2 b 2 c2 )2
ab bc ca a 2 b 2 c2
ab bc ca (a b c)2 2(ab bc ca)
3(ab bc ca) (a b c)2 9

eKTaj (a c b a c b )2 3(ab bc ca) 9


naM[ a c b a c b 3 (**)
tamvismPaB (*) & (**) eKTaj)an
ab 2
bc 2
ca2
3

Bit
2
2
2
2
ab

dUcenH

bc
ca
a2
b2
c2
3

a b 2 b c2 c a2 2

eroberogeday lwm pln

- Page 194 -

vismPaBeRCIserIs
lMhat;TI112 (Greece National Olympiad 2007)
eK[ a , b , c CaRCugrbs;RtIekaNmYy . cUrRsayfa
( c a b ) 4 ( a b c )4 ( b c a ) 4

ab bc ca
a(a b c) b(b c a ) c(c a b )

tamvismPaB AM GM eKman
( c a b )4
a(a b c) 2(c a b )2
a( a b c )
( c a b )4
eKTaj a(a b c) 2(c a b)2 a(a b c)
( c a b )4
a 2 2b 2 2c2 5ac 5ab 4bc (1)
a( a b c )

b
RsaydUcKaEdreK)an

( a b c )4
2a 2 b 2 2c2 5ab 5bc 4ac ( 2)
b(b c a )
( b c a )4
2a 2 2b 2 c2 5bc 5ac 4ab ( 3)
c(c a b )

bUkvismPaB (1) ,

( 2) & ( 3)

eK)an

S 5(a 2 b 2 c2 ) 4(ab bc ca) ab bc ac

dUcenH

Bit

(c a b)4 (a b c)4 (b c a)4


S

ab bc ca
a(a b c) b(b c a) c(c a b)

eroberogeday lwm pln

- Page 195 -

vismPaBeRCIserIs
lMhat;TI113 (IMO Longlists 1980)
eKkMNt;cMnYn a0 , a1 , a2 , ...., an dUcxageRkam
1
ak 2
a 0 ; ak 1 ak
( n 1 , k 0 , 1 , 2 , ...., n 1 )
2
n
cUrbgajfa 1 n1 an 1 .
ak 2 a k (n a k )
eKman ak 1 ak n n
eK)an a 1 a (an n) a1 a 1 n
k 1
k k
k
k
b a 1 n a1 a 1
k
k
k 1
n 1
n 1 1
1
ehtuenH a n ( a a 1 ) a1 a1 2 a1 (*)
k 0 k
k 0 k
k 1
0
n
n
1
eKman a0 2 0 Bit . ]bmafa ak 0 Bit
ak 2
tam ak 1 ak n eKTaj)an ak 1 0 Bit
dUcenH ak 0 enaH ak n n b a 1 n n1 , n 1
k
n 1
n 1 1
1
eK)an a n n n1 n1 ... n1 nn 1
k 0 k
k 0

(n )

eroberogeday lwm pln

- Page 196 -

vismPaBeRCIserIs
tam (*) eKTaj)an 2 a1

naM[ an 1

(i )

ma:geToteday an 1 enaH ak 1 b ak n n 1
b a 1 n n 1 1 RKb; k 0 , 1 , 2 , ...., n 1 .
k

n 1

n 1

1
n

k 0
k 0 n 1 n 1
edayBiniteXIjfaRKb; n 1 eKman n n 1 nn 12 22 0
n 1
n 1
enaHeKTaj)an a 1 n n n 1 nn 12
k 0 k
tam (*) eKTaj)an 2 a1 nn 12
n
b a1 2 nn 12 n n 1 enaH an n n 1 1 n1 (ii )
n
1
(
i
)
&
(
ii
)
eKTaj)an
1

an 1 .
tam
n
dUcenH 1 n1 an 1 .

eK)an

ak n

eroberogeday lwm pln

- Page 197 -

vismPaBeRCIserIs
lMhat;TI114 ( Malaysia National Olympiad 2010 )
cMeBaHRKb; a , b , c FMCagmYy cUrbgajfa
log a bc logb ca logc ab 4 (logab c log bc a log ca b )

tag A loga bc logb ca logc ab


nig B 4 (logab c logbc a logca b)
ehIyyk x ln a , y ln b , z ln c
cMeBaH a , b , c 1 enaH x , y , z 0
p
tamrUbmnbreKal logq p ln
eK)an

ln q
A

yz zx xy
1 1
1 1
1 1

x( ) y ( ) z ( )
x
y
z
y z
z x
x y

B 4(

z
x
y
4x
4y
4z

xy yz zx yz zx xy

tamvismPaB AB GM RKb; u , v 0 : u v 2 uv
v
4
1 1
4

b (u v )2 4uv b uuv

b
u v u v u v
eKTaj x( y1 1z ) y4xz ; y( 1z 1x ) z4yx ; z( 1x y1 ) x4zy
bUkvismPaBenHeK)an A B Bit .
dUcenH
log a bc logb ca logc ab 4 (logab c log bc a log ca b )

eroberogeday lwm pln

- Page 198 -

vismPaBeRCIserIs
lMhat;TI115 ( Morocco National Olympiad 2011 )
eKtag , , CargVas;mMukgRtIekaN ABC mYyEdlmanbrimaRt 2p
nigkaMrgVg;carwkeRkA R .

R2
a / cUrRsaybBaak;fa cot cot cot 3 9 . 2 1

p
b / etIeBlNaeTIbeK)ansmPaB ?
2

dMeNaHRsay

R2
a / RsaybBaak;fa cot cot cot 3 9 . 2 1

p
2

eKman

1
1
1
2
2
2

cot

cot

cot

2
2
2
sin sin sin
1
1
1
27 R 2
eyIgnwgRsayfa 2 2 2 2
sin sin sin
p

tamvismPaB Cauchy Schwarz eKman


1
1
1
1 1
1
1 2

(
) (1)
2
2
2
sin sin sin 3 sin sin sin
2
(a 1 a 2 a 3 ) 2
a12 a 2 2 a 3
edayeRbI
b1 b 2 b 3
b1 b 2 b 3
9
eK)an 1 1 1
sin sin sin sin sin sin

eroberogeday lwm pln

- Page 199 -

vismPaBeRCIserIs
tamRTwsIbTsIunUsGnuvtn_kg ABC eK)an

a
b
c
abc

2R
sin sin sin sin sin sin
eKTaj sin sin sin a b c p
2R
R
9
9R
ehtuenH 1 1 1

( 2)
sin sin sin sin sin sin
p

tam (1) & (2) eKTaj)an


1
1
1
1 81R 2 27 R 2

2
2
2
2
sin sin sin 3
p
p2

Bit

R2

dUcenH cot cot cot 3 9 . 2 1 .


p

1
1
1

b / vismPaBenHkayCasmPaBluHRtaEt
sin sin sin
2

eKTaj)an naM[ ABC CaRtIekaNsmgS .

eroberogeday lwm pln

- Page 200 -

vismPaBeRCIserIs
lMhat;TI116 Japan 1997
eKyk a , b , c CabIcMnYnBitviCman . cUrbgajfa
(b c a )2
(c a b )2
(a b c )2 3

2
2
2
2
2
2
5
(b c ) a
(c a ) b
(a b ) c
(b c a )2
(c a b ) 2
(a b c ) 2

tag T
2
2
2
2
(b c ) a
(c a ) b
(a b )2 c 2
bc
ca
ab
yk x a , y b , z c
eK)an 1 1 x 1 1 y 1 1 z aa bb cc 1 ehIykenSam T eTACa
( x 1)2 ( y 1)2 ( z 1)2
T 2
2
2
.
x 1
y 1
z 1
ebI x y z 2 eK)an T 53 CakrNIEdlvismPaBkayCasmPaB .
( x 1)2
eyIgsntfamanBIrcMnYnBit a nig b EdlnaM[ 2 x a 1 b Bit
x 1
RKb; x 0 ehIyeday x 1 1 y 1 1 z 1 1 1 enaHedIm,I[ T 53
luHRtaEt ( x a 1 b) ( y a 1 b) ( z a 1 b) 53
naM[ a 3b 53 (*)

ma:geTotedaysmPaBekIteLIgcMeBaH x 2 enaH x 2 CabsDb


eroberogeday lwm pln

- Page 201 -

vismPaBeRCIserIs
( x 1)2
rbs;smIkar 2 x a 1 b
x 1
b (x 1)(x 1)2 (x2 1)(a b bx) 0

tag f (x) ( x 1)(x 1)2 (x2 1)(a b bx)


x 2 CabsDbn f ( x ) 0 enaH f ' ( 2) 0
eday f ' (x) (x 1)2 2(x2 1) 2x(a b bx) b(x2 1)
enaH f ' (2) 1 6 4(a 3b) 5b 7 4 53 5b 0
3
54
eKTaj)an b 23
ehI
y
tam
(*) eK)an a 3b
25
5
25

eK)an

( x 1)2 23
54

2
25
25( x 1)
x 1

smmUl

2( x 2)2 ( x 7 )
0 Bit
2
25( x 1)( x 1)

( x 1)2 23
ehtuenH 2 25 25(54
(1)
x
1
)

x 1
( y 1)2 23
54
( z 1)2 23
54
(
2
)
ni
g
( 3)

2
2
y 1 25 25( y 1)
z 1 25 25( z 1)

bUkvismPaB (1) , (2) & (3) eK)an


T

1
1
69 54 15 3
23
54 1
3 (

25
25 x 1 y 1 z 1
25
25 5

dUcenH

(b c a )2
(c a b )2
(a b c )2 3

2
2
2
2
2
2
5
(b c ) a
(c a ) b
(a b ) c

eroberogeday lwm pln

Bit

- Page 202 -

vismPaBeRCIserIs
lMhat;TI117> Iran 1996
eK[bIcMnYnBitminGviCman a , b , c nigminsUnRBmKaBIr .
cUrRsaybBaak;fa

1
1
1 9
(ab bc ca)

(*)
2
2
2
(b c )
(c a ) 4
(a b )
tag x a b c , y ab bc ca , z abc

eKmansmPaB
(a b )(b c)(c a ) (a b c)(ab bc ca) abc
xy z

nig (a b)2 (a c)2 (x2 y )2 4x(xy z )


cyc

( x 2 y )2 4x( xy z ) 9
vismPaB (*) xagelIsmmUl y

2
( xy z )

4
smmUl 4x4y 17x2y 2 4y 3 34xyz 9z 2 0

xy(x3 4xy 9z) y(x4 5x2y 4y2 6xz) z(xy 9z) 0 (**)

tamvismPaB Schur cMeBaHRKb;cMnYnBitminGviCman x , y , z eKman


x( x y )( x z ) 0 x3 4xy 9z 0

(1)

cyc

x2 ( x y )( x z ) 0 x4 5x2y 4y 2 6xz 0 ( 2)
cyc

eroberogeday lwm pln

- Page 203 -

vismPaBeRCIserIs
tamvismPaB AM GM eKman
(a b c)(ab bc ca ) 9abc xy 9z 0 ( 3)

tam (1), (2) & (3) eKTaj)an (**) Bit .

dUcenH (ab bc ca) 1 2 1


(a b )

eroberogeday lwm pln

1 9

2
2
(b c )
(c a ) 4

- Page 204 -

vismPaBeRCIserIs
CMBUkTI3

lMhat;Gnuvtn_
!> eK[ a , b , c , x , y , z CacMnYnBitviCmanEdl x y z 1 .
cUrbgajfa
ax by cz 2 (ab bc ca)( xy yz zx ) a b c

Ukraine,2001
@> eK[ a , b , c 0 . cUrbgajfa
a4 a 2b 2 b 4 b 4 b 2c2 c4 c4 c 2a 2 a4
a 2a 2 bc b 2b 2 ca c 2c2 ab

Gazeta Matematica
#> ebI a , b , c CacMnYnBitviCmanEdl abc 2 enaHbgajfa
a3 b 3 c3 a b c b c a c a b .
etIsmPaBekIteLIgenAeBlNa ?
JBMO 2002 Shortlist
$> eK[ a & b CaBIrcMnYnBitviCmanEdl a b 1 .
a2
b2
cUrRsaybBaak;fa a 1 b 1 13
Hungary 1996
eroberogeday lwm pln

- Page 205 -

vismPaBeRCIserIs
%> RKb;cMnYnBit x nig y cUrbgajfa

3( x y 1)2 1 3xy

Columbia,2001
^> eK[ x , y (0,1) . bgajfa xy y x 1 ?
&> RKb;cMnYnBitminGviCman a , b cUrRsaybBaak;fa

3
3
a b
a a b ab b a ab b a 2 b 2

2
4
3
2

APMC 1993
*>RKb;cMnYnBit x nig y mansBaadUcKa cUrbgajfa
2xy
x2 y 2
xy

xy
xy
2
2

(>RKb;cMnYnBitviCman a , b , c cUrRsaybBaak;fa
2(a 3 b 3 c 3 ) 9(a b c)2
33
2
2
2
abc
a b c

!0>RKb;cMnYnBit x, y , z 0 cUrbgajfa
3

|x y||y z||zx| x y z

3
3
x , y , z 0 Edl x y z 1 . cUrbgajfa

xyz

!!>eK[

x
y
z
3

1 x
1 y
1 z
2

eroberogeday lwm pln

- Page 206 -

vismPaBeRCIserIs
!@> eK[ a , b , c CabIcMnYnBitviCman . cUrbgajfa
(a 2b b 2c c2a)(ab 2 bc 2 ca2 ) abc 3 (a 3 abc)(b 3 abc)(c3 abc)

KMO Summer Program Test,2001


!#>eK[ a , b , c 0 . cUrbgajfa
a 2 ab b 2 b 2 bc c2 a 2 ac c2
!$> eK[cMnYnBit a , b , c 1 . cUrRsayfa
a 1 b 1 c 1 c(ab 1)

Hongkong 1998
!%>eK[ a , b , c 0 . cUrRsayfa
(a b )(b c)(c a )
ab bc ca

8
3
!^>eK[ a , b , c 0 Edl a b c 1 . cUrRsaybBaak;fa
3

1
1
1
1
6b 3 6c 3 6a
a
b
c
abc

IMO Short Lists 2004


!&> eK[bIcMnYnBitviCman a , b , c . cUrRsaybBaak;fa
a
b
c

2
bc
ca
ab

eroberogeday lwm pln

Macedonia,1995
- Page 207 -

vismPaBeRCIserIs
!*>eK[ a , b , c 0 Edl abc 1 . cUrRsaybBaak;fa
1
1
1
(a 1 )(b 1 )(c 1 ) 1 .
b
c
a

IMO 2000

!(>eK[ a , b , c 0 Edl abc 1 . cUrRsaybBaak;fa


ab
bc
ca

1
5
5
5
5
5
5
a b ab b c bc c a ca

IMO Short Lists 1996

@0>eK[ a , b , c , d 0 cUrRsayfa
a
b
c
d
2

b 2c 3d c 2a 3a d 2a 3b a 2b 3c 3

IMO Short Lists 1993


@!> eK[ a , b , c , d 0 Edl ab bc cd da 1cUrRsayfa
a3
b3
c3
d3
1

bcd cda dab abc 3

IMO Short Lists 1990

@@> eK[ a , b , c 0 . cUrRsaybBaak;fa


a 2 bc b 2 ca c2 ab

abc
bc
ca
ab

eroberogeday lwm pln

- Page 208 -

vismPaBeRCIserIs
@#> eK[ a , b , c 0 .
cUrRsaybBaak;fa
1
1
1
2(a 3 b 3 c3 )
2 2 3
2
abc
a
b
c
@$> eK[ a , b , c 0 Edl a2 b2 c2 3
1
1
1
3

1 ab 1 bc 1 ca 2

@%> eK[ a , b , c 0

. cUrbgajfa

Belarus 1999
Edl a2 b2 c2 1 . cUrbgajfa

1
1
1
9

1 ab 1 bc 1 ca 2
@^> eK[ a , b , c 0 Edl a4 b4 c4 3
1
1
1
1

4 ab 4 bc 4 ca

. cUrbgajfa

Moldova,2005
@&> eK[ a , b , c 0 Edl a2 b2 c2 1 . cUrbgajfa
a
b
c
3
2

a
a

b
b

c
c
b 2 1 c2 1 a2 1 4
Greece 2002

eroberogeday lwm pln

- Page 209 -

vismPaBeRCIserIs
@*> eK[bIcMnYnBitviCman a , b , c 0 . cUrbgajfa

1
1
1 9

(ab bc ca)

2
2
2
(b c )
(c a ) 4
(a b )

Iran 1996

@(> eK[bIcMnYnBitviCman a , b , c 0 . cUrbgajfa


1 3 2
1 1 1
(a b 2 c2 )( ) a b c a 2 b 2 c2
3 3
a b c

Albania 2002

#0> eK[bIcMnYnBitviCman a , b , c 0 . cUrbgajfa


a b c ab bc ca

b c a ca ab bc

Belarus 1997

#!>eK[ x, y , z 0 . cUrbgajfa
xyz
1
4
(1 3x )( x 8y )( y 9z )( z 6) 7

etIsmPaBekIteLIgenAeBlNa ?
#@>ebIcMnYnBit a , b , c 0 Edl a b c 1 enaHbgajfa
5(a 2 b 2 c2 ) 6(a 3 b 3 c3 ) 1

##> eK[ a , b , c 0 Edl abc 1 .


cUrbgajfa ba bc ac a b c .
eroberogeday lwm pln

- Page 210 -

vismPaBeRCIserIs
#$>cUrbgajfaebI n 3 ehIy x1 , x2 ,..., xn 0 manplKuNesI 1
enaH 1 x 1 x x 1 x 1 x x ... 1 x 1 x x 1
1

1 2

2 3

n 1

Russia,2001
#%>eK[ a, b, c 0 CamYy a b c 1 . cUrbgajfa
a2 b b 2 c c2 a

2
bc
ca ab
#^>eK[ a , b , c 0 edaydwgfa a4 b4 c4 2(a2b2 b2c2 c2a2 )

cUrbgajfa a2 b2 c2 2(ab bc ca) .


Kvant,1988
#&>
#*>eK[ x , y , z CacMnYnBitviCmanmanplbUkesI 3 . cUrbgajfa
x y z xy yz zx .
Russia 2002
#(>eK[ a , b , c CacMnYnBitviCman . cUrbgajfa
a3
b3
c3
3(ab bc ca)

a bc
b 2 bc c2 c2 ac a 2 a 2 ab b 2
a, b , c
axby cz 1

$0>eK[cMnYnBitviCman
nig
bgajfa (abc xyz)( ay1 bz1 cx1 ) 3 .
eroberogeday lwm pln

Russia 2002
- Page 211 -

vismPaBeRCIserIs
$!> eK[ a, b c CabIcMnYnBitviCman . cUrbgajfa
ab
bc
ca
1

(a b c )
a b 2c b c 2a c a 2b 4
$@> eK[bIcMnYnBit a, b, c (0 , 1) Edl ab bc ca 1 .

cUrbgajfa
a
b
c
3 1 a 2 1 b 2 1 c2

2
2
2
4 a
b
c
1 a 1 b
1 c
$#> eK[ x, y , z 1 nig x y z 1 . cUrbgajfa
1
1
1
27

1 x 2 1 y 2 1 z 2 10

$$> eK[ x, y , z CabIcMnYnBitviCmanEdl x2 y 2 z 2 2 .


cUrbgajfa x y z xyz 2
IMO Shortlist 1987
$%>eK[ a , b , c 0 man abc 1 . cUrbgajfa
(a b )(b c)(c a ) 4(a b c 1)

MOSP,2001
$^> cUrbgajfaRKb; a , b , c 0 eKman
(a2 b2 c2 )(a b c)(b c a)(c a b) abc(ab bc ca)

eroberogeday lwm pln

- Page 212 -

vismPaBeRCIserIs
$&> eK[ , x, y , z CacMnYnBitviCmanEdl xyz 1 nig 1 .
x
y
z
cUrbgajfa y z z x x y 32
$*> eK[ a , b , c CacMnYnBitviCmanEdl a b c 1 . cUrbgaj
b c
c a
a b
3 3

a( 3c ab ) b 3a bc ) c( 3b ca )
4

$(> eK[ x , y , z 0 Edl x y z xyz . cUrbgaj


( x 1)( y 1)( z 1) 6 3 10

%0> eK[cMnYnBitviCman a , b , c . cUrbgajfa


a 2 b 2 c2 2abc 3 (1 a )(1 b )(1 c)

%!> cUrbgajfacMeBaHRKb;cMnYnBitminGviCman a , b , c edaydwgfa


a 2 b 2 c2 abc 4 eyIgman 0 ab bc ca abc 2
USAMO,2001
%@> eK[ a , b , c CacMnYnBitviCman . cUrbgaj
a ab 3 abc 3 a b a b c
a.
.
3
2
3
%#> eK[ a , b , c CacMnYnBitviCman . cUrbgajfa
1
1
1
3

3.
a(1 b ) b(1 c) c(1 a )
abc (1 3 abc )

eroberogeday lwm pln

- Page 213 -

vismPaBeRCIserIs
%$> cUrbgajfaRKb;cMnYnBit a , b , c Edl a2 b2 c2 9 eKman
2(a b c) abc 10 .
Vietnam,2002
%%> cMeBaHRKb;cMnYnBit a , b , c bgajfa
4
( a b ) 4 ( b c )4 ( c a )4 ( a 4 b 4 c 4 )
7

Vietnam TST 1996


%^> eK[bIcMnYnBit x , y , z 0 Edl x y z 1 .
cUrbgajfa 0 xy yz zx 2xyz 277
IMO 1984
%&> eK[bIcMnYnBit x , y , z 0 Edl x y z 1 .
cUrRsaybBaak;fa x2y y 2z z 2x 274
Canada 1999
%*> eK[ a , b , c 0 Edl a b c abc .
cUrbgajfa a2 b2 c2 3 abc
BMO 2001
%(> eK[ x, y , z 0 Edl x y z xyz .
bgajfa xy yz zx 9(x y z ) .
Bearus 1996
eroberogeday lwm pln

- Page 214 -

vismPaBeRCIserIs
^0> eK[ x , y , z IR Edl x2 y 2 z 2 2 .
bgajfa x y z 2 xyz
Poland 1991
^!> eK[ a , b, c 0 Edl a2 b2 c2 1 .
1
cUrbgajfa a b c abc
4 3

Macedonia 1999

^@> eK[ x, y, z CacMnYnBitEdl 0 x, y , z 1 nig xy yz zx 1


cUrbgajfa x 2 y 2 z 2 3 2 3
1 x

1 y

1 z

^#> eK[ x, y, z CacMnYnBitEdl x y z xyz ( x, y , z 0 ) .


bgajfa x 2 y 2 z 2 3 2 3 .
1 x

1 y

1 z

^$> eK[ x, y, z CacMnYnBitEdl x y z xyz ( x, y , z 0 ) .


bgajfa xy yz zx 3 1 x2 1 y 2 1 z 2
^%> eK[ a , b , c CacMnYnBit . cUrbgajfa
(a 2 1)(b 2 1)(c 2 1) (ab bc ca 1)2

^^> eK[ x , y , z 0 Edl xy yz zx 1 .


cUrbgajfa x 1 y y 1 z z 1 x 52
eroberogeday lwm pln

- Page 215 -

vismPaBeRCIserIs
^&> eK[ a , b , c 0 Edl abc 1 .
a3
b3

cUrbgajfa
2
2
(a 1)

(b 1)

c3
3
2
(c 1)

^*>eK[ a , b , c 0 . cUrbgajfa
3

3
a b c



8
b c c a a b
^(> eK[ a , b , c 0 Edl a b c 3 . cUrbgajfa
1
1
1
3

3 7
3 7
3 7
a 7
b 7
c 7 2

&0> eK[ a , b , c , d 0 Edl a b c d 4 .


cUrbgajfa 1 2 2 1 2 1 4 2
1 a 1 b
1 c 1 d
&!> eK[ a , b , c 0 . cUrRsaybBaak;fa
a3
b3
c3
1

a 3 (b c ) 3 b 3 (c a ) 3 c 3 (a b ) 3 3

&@>eK[ a , b, c 0 Edl a2 b2 c2 1 . cUrRsaybBaak;fa


1
1
1
9

1 ab 1 bc 1 ca 2
&#> eK[ x , y , z [1 , 2 ] . cUrbgajfa
1 1 1
x
( x y z )( ) 6 (
)
x y z
cyc y z

eroberogeday lwm pln

- Page 216 -

vismPaBeRCIserIs
&$> eK[ a, b, c 0 Edl a b c 1 .
1
1
10
2
cUrbgajfa abc

a b 2 c2 ab bc ca
&%> eK[ a, b, c 0 nig ab bc ca 1 .
1 a 2b 2 1 b 2c2 1 c2a 2 5
cUrbgajfa

2
2
2
2
(a b )

(b c )

(c a )

&^> eK[ a , b , c 0 . cUrbgajfa


a3 b 3 c3
54abc

5
3
abc
(a b c )

&&> eK[ a , b , c 0 . cUrbgajfa


a4 b 4 c4
3abc
2
(a 2 b 2 c 2 )

ab bc ca a b c 3
&*> eK[ a , b , c 0 . cUrbgajfa
b 2 c2 a2 c2 a2 b 2 a2 b 2 c2
1

2
2
2
a 2bc
b 2ca
c 2ab
&(> eK[ a , b , c 0 . cUrbgajfa
1 1 1
bc
ca
ab
2
2
2
a b c a bc b ca c ab
*0> eK[ a , b, c 0 Edl abc 1 .

cUrbgajfa (1 a3 )(1 b3 )(1 c3 ) a b2 c

eroberogeday lwm pln

- Page 217 -

vismPaBeRCIserIs
*!> eK[ a , b , c 0 . cUrbgajfa
1 1 1
9

4
a b c abc

1
1
1

b
b

c
c

*@> eK[ a , b , c 0 . cUrbgajfa

a2 b 2 c2
8abc

2
ab bc ca (a b )(b c)(c a )

*#> eK[ a , b , c 0 Edl abc 1 . cUrbgajfa


1
1
1

3
2
2
2
a a1 b b1 c c1
*$> eK[ a , b , c 0 . cUrbgajfa
a 2 b 2 b 2 c2 c 2 a 2 3(a 2 b 2 c2 )

ab
bc
ca
abc
*%> eK[ a , b , c 0 . cUrbgajfa
ab
bc
ca
3(a b c)

a2 b 2 b 2 c2 c2 a2 a2 b 2 c2
*^> eK[ a , b , c 0 . cUrbgajfa
a
b
c
abc

ab bc ca
a 2 2bc
b 2 2ca
c2 2ab

*&> eK[ a , b , c 0 . cUrbgajfa

a
b
c
5

abc
ab
bc
ca 4

eroberogeday lwm pln

- Page 218 -

vismPaBeRCIserIs
**> eK[ a , b , c 0 Edl ab bc ca 1 . cUrbgajfa
a(1 a 2 ) b(1 b 2 ) c(1 c 2 ) 2 a b c

*(> eK[ a , b 0 . cUrbgajfa


(a b )2 (a b

1 1 2
) 8(1 2 )
a b

(0>eK[ a , b , c 0 . cUrbgajfa

1 1 1
1
1
1
(a b c )( ) 1 1 (a 2 b 2 c 2 )( 2 2 2 )
a b c
a
b
c

(!>eK[ a , b , c 0 Edl a b c 1 . cUrbgajfa


a2
b2
c2
3

b 2 1 c2 1 a2 1 2
(@> eK[ x, y , z 0 .

x y z
3

bgajfa x y z xyz
(#>eK[ a , b , c 0 . cUrbgajfa
x y z

9(a 2 bc )(b 2 ca )(c 2 ab ) 8(a 3 b 3 c 3 )2

($>eK[ x, y , z 0 Edl x y z 1. cUrbgajfa


2 (1 x 2 )2 (1 y 2 )2 (1 z 2 )2 (1 x )(1 y )(1 z )

(%>eK[ x, y , z 0 Edl xy yz zx 1 . cUrbgajfa


x(1 y 2 )(1 z 2 ) y (1 z 2 )(1 x 2 ) z(1 x 2 )(1 y 2 )

eroberogeday lwm pln

4 3
9

- Page 219 -

vismPaBeRCIserIs
(^>eK[ a , b , c 0 . cUrbgajfa
1
1
1
27

b(a b ) c(b c) a(c a ) 2(a b c)2

(&>eK[ a , b , c 0 Edl abc 1 . cUrbgajfa


1
1
1
3

(1 a )(1 b ) (1 b )(1 c) (1 c)(1 a ) 2

(*>eK[ a , b , c 0 Edl a2 b2 c2 1 . cUrbgajfa


a
b
c
2

1 bc 1 ca 1 ab
((>eK[ a , b , c 0 Edl a b c 1 . cUrbgajfa
1
1
1
27

1 ab 1 bc 1 ca 8
!00> eK[ a , b , c 0 . cUrbgajfa
1

ab
bc
ca
a
b
c

c
a
b
ca
ab
bc

!0!> eK[ a , b , c 0 Edl a b c 1 . cUrbgajfa

(b c )2
(c a )2
(a b )2
a
b
c
3
12
12
12
!0@> eK[ x, y , z 0 Edl xyz 1 . cUrRsayfa
x2y 2
y 2z 2
z 2x2
2 2
2 2
1
2 2
7
7
7
7
7
7
x y x y
y z y z
z x z x

eroberogeday lwm pln

- Page 220 -

vismPaBeRCIserIs
!0#> eK[ a , b , c CacMnYnBitviCmanEdl abc 1 .
cUrbgajfa a b c a b c
!0$> cUrbgajfacMeBaHRKb;cMnYnBitviCman a , b , c , d eKmanvismPaB
2

1 1 4 16
64

a b c d abcd
!0%> cMeBaHRKb;cMnYnBitminGviCman a , b , c 1 cUrbgajfa
a
b
c

(1 a )(1 b )(1 c) 1
bc1 ca1 ab1
(USAMO 1980 )

!0^> eK[ a , b , c CacMnYnBitviCmanEdl a b c 1 . cUrbgajfa


a 3 b 3 c 3 6abc

1
4

!0&> eK[ a , b , c CacMnYnBitviCmanEdl abc 1 . cUrbgajfa


2
2
2

1
2
2
2
2
2
2
(a 1) b 1 (b 1) c 1 (c 1) a 1
!0*> eK[ a0 , a1 , a2 , ..., an CacMnYnBitkgcenaH ( 0 , 2 ) edaydwgfa

tan(a0 ) tan(a1 ) ... tan(an ) n 1


4
4
4
cUrbgajfa tan(a0 ) tan(a1 )..... tan(an ) nn1
(USAMO 1998 )

eroberogeday lwm pln

- Page 221 -

vismPaBeRCIserIs
!0(> eK[ a , b , x , y , z CacMnYnBitviCman .
cUrbgajfa ay x bz az y bx ax z by a 3 b
!!0> eK[ n 2 CacMnYnKt; ehIy x1 , x2 , ...., xn CacMnYnBitviCman
edaydwgfa x x .... x 1 . cUrkMNt;tmtUcbMputn
2

2
n
5
x2

x1
xn

...
x 2 x 3 ... xn x 3 ... xn x1
x1 x 2 ... xn1

!!!> eK[ n CacMnYnKt;viCman . cUrkMNt;tmGb,brmanplbUk


2
3
n
x2
x3
xn

.....
.
S x1
2
3
n
(Poland 1995 )

!!@> cUrbgajfacMeBaHRKb;cMnYnBitviCman a , b , c , d eKman


a4b b 4c c4d d 4a abcd(a b c d )

!!#> eK[ a , b , c CacMnYnBitviCman . cUrbgajfa


2
3

2
3

2
3

2a
2b
2c



3
b c c a a b
(MOP 2002 )

!!$> cUrbgajfacMeBaHRKb;cMnYnBitviCman a , b , c eKman


a2
b2
c2
3

(a b )(a c) (b c)(b a ) (c a )(c b ) 4

eroberogeday lwm pln

- Page 222 -

vismPaBeRCIserIs
!!%> cUrbgajfacMeBaHRKb;cMnYnBitviCman a , b , c eKman
a 2b b 2c c 2a


3

a 2c b 2a c 2b
!!^> eK[ a , b , c , x , y , z CacMnYnBitviCmanEdl x y z 1 .
3

cUrbgajfa
ax by cz 2 (ab bc ca)( xy yz zx ) a b c

!!&> eK[ a , b , c CacMnYnBitviCmanEdl abc 1 .


cUrbgajfa 5 ba bc ac (1 a)(1 b)(1 c)
!!*> eK[ a , b , c CacMnYnBitviCman . cUrbgajfa
a3
b3
c3
1

(2a2 b2 )(2a2 c2 ) (2b2 c2 )(2b2 a2 ) (2c2 a2 )(2c2 b2 ) a b c

!!(> eK[ a , b , c CacMnYnBitviCman . cUrbgajfa


(a5 a 2 3)(b 5 b 2 3)(c5 c 2 3) (a b c)3
a2 b 2 c2 a b c
!@0> RKb; a , b , c 0 cUrRsaybBaak;fa b2 c2 a2 b c a
!@!> cUrRsayfaRKb; a , b , c 0 eKman
b 2 c2 a2 c2 a2 b 2 a 2 b 2 c2 3

a( b c )
b(c a )
c(a b )
2

!@@> eK[ a , b , c 0 Edl a b c 1 . cUrRsayfa


ab c bc a ca b 1 ab bc ca

eroberogeday lwm pln

- Page 223 -

vismPaBeRCIserIs
!@#> eK[ a , b , c 0 Edl abc 8 . cUrRsayfa
a2
(a 3 1)(b 3 1)

b2
(b 3 1)(c3 1)

c2
(c3 1)(a 3 1)

4
3

!@$>cUrRsayfaRKb;cMnYnBit 0 x1 , x2 , ....., xn 12 eKmanvismPaB


n

n 1

1 1
i 1 x i

x1 x 2 ... xn
!@%> bgajfaRKb; a , b , c 0 eK)an

a2
b2
c2
1

( 2a b )( 2a c) ( 2b c)( 2b a ) ( 2c a )( 2c b ) 3

!@^> bgajfaRKb; a , b , c 0 eK)an


3

4a 2
4b 2
4c2

4a 4b 4b 4c 4c 4a
ab bc ca
3

!@&> eKman a , b , c , x , y , z IR Edlepgpat;smPaB


(a b c)( x y z ) 3

nig (a b c )(x y z ) 4
cUrbgajfa ax by cz 0 .
!@*> eK[ a , b , c 1 Edl a21 1 b21 1 c2 1 1 1 .
2

cUrRsayfa a 1 1 b 1 1 c 1 1 1 .
eroberogeday lwm pln

- Page 224 -

vismPaBeRCIserIs
!@(> bgajfaRKb; a , b , c 0 eKman
a

a2 b2
b 2 c2
!#0> eK[ n 2 CacMnYnKt;viCman .

c
c2 a2

3 2
2

cMeBaHRKb;cMnYnBitviCman a1 , a2 ,..., an Edl a1 .a2 ....an 1


cUrRsaybBaak;fa
a1 1
a 1
a 1
2
... n
a1 a 2 ... an
2
2
2
!#!> eK[ a , b , c , x , y , z IR Edl xy yz zx 3 .
cUrbgajfa a(bycz ) b(czax) c(axby ) 3 .
!#@> eK[ a , b , c 0 Edl a b c 3 .
cUrbgajfa aba 1 bcb 1 cac 1 32 .
!##> eK[ a1 , a2 , ...., an 0 . cUrRsaybBaak;fa
2

a1 a1a2 3 a1a2a3 ... n a1a2 ...an n a1 a2 a1 a2 ... an


a1.
...
n
2
n
!#$> cUrRsayfaRKb; a , b , c 0 eKman
ab bc ca bc ca ab ca ab bc 3

2
2
2
2
2
2
2
b c
c a
a b

eroberogeday lwm pln

- Page 225 -

vismPaBeRCIserIs
!#%> eK[ a , b , c CacMnYnBitviCman . cUrbgajfa
a 2 bc b 2 ca c 2 ab b c a
2
2

2
b ca c ab a bc a b c
!#^> bgajfaRKb;cMnYnBitviCman x , y , z Edl x y z 1

eKman

x ( y z )2 y ( z x )2 z ( x y )2 3

!#&> eK[ a , b , c CacMnYnBitminGviCman nig KanBIrcMnYnNaesIsUnRBmKa.


ca bc ca ab ca ab bc 3

cUrRsayfa abc2 bc
2
2
2
2
2
2
a b
b c
a
!#*> cMeBaHRKb;cMnYnBitviCman a nig b cUrRsayfa
a b
2ab
1 2
2 2
2
b a
a b
!#(> eK[ a , b , c CargVas;RCugnRtIekaNmYy .
cUrRsayfa 3a cb c 3b ac a 3c ba b 1
!$0> eK[ a , b , c CabIcMnYnBitviCman .

cUrRsaybBaak;fa

a 2c
b 2a 2 c 2b
b2 .
c .
a 2 b 2 c2
3b
3c
3a
!$!> cMeBaHRKb;cMnYnBitviCman x , y , z cUrRsayfa
27
( x 2 yz )2
xy( xy z 2 )( zx y 2 )
8
a2 .

eroberogeday lwm pln

- Page 226 -

vismPaBeRCIserIs
!$@> eK[ a , b , c CacMnYnBitviCmanEdl abc 1 .
a 3 b 3 c3 3
a
b
c

cUrRsayfa
4
bc ca ab
!$#> eK[ a , b , c CacMnYnBitviCmanEdl a b c 2 .
cUrRsayfa b(aa b) c(bb c) a(ac c) 2
!$$> RKb;cMnYnBitviCman a , b , c cUrRsayfa
a 2 ac
b 2 ba
c 2 cb
abc

2b a c 2c a b 2a b c
2
!$%> eK[ a , b , c IR . cUrRsayfa
a 2 ab b 2 b 2 bc c 2 c 2 ca a 2 9

4
(a b ) 2
(b c ) 2
(c a )2

!$^> eK[cMnYnBitviCman a , b , c Edl a b c 1 .


cUrRsayfa
a
b
c
1

4b 3bc 4c 4c 3ca 4a 4a 3ab 4b 3


!$&> RKb;cMnYnBitviCman a , b , c Edl a b c 3 .
cUrRsayfa b ac 1 c ba 1 a bc 1 3

eroberogeday lwm pln

- Page 227 -

vismPaBeRCIserIs
!$*> RKb;cMnYnBitviCman a , b , c Edl a b c 3 .
cUrRsayfa
a
b
c
3

2b 3c 1 2c 3a 1 2a 3b 1 4
!$(> eK[ a , b , c 0 . cUrRsaybBaak;fa
a3 b3
b 3 c3
c3 a3
6(ab bc ca)

(a b c) (a b)(b c)(c a)
a2 b 2
b2 c2
c2 a2

!%0> ebI a , b , c CacMnYnBitviCmanenaHcUrRsayfa


a 2 (b c ) b 2 (c a ) c 2 (a b )
2
2
abc
b 2 c2
c a2
a b2
!%!> ebI a , b , c CacMnYnBitviCmanenaHcUrRsayfa
a b c 9(a 2 b 2 c 2 )

b c a
(a b c )2

!%@> ebI a , b , c CacMnYnBitviCmanEdl ab bc ca abc 4


cUrRsayfa
1
1
1
3

(a 1)2 (b c) (b 1)2 (c a ) (c 1)2 (a b ) 8

!%#> ebI a , b , c CacMnYnBitviCmanEdl abc 1 . cUrRsayfa


1
1
1
3

(a 1)2 (b c) (b 1)2 (c a ) (c 1)2 (a b ) 8

eroberogeday lwm pln

- Page 228 -

vismPaBeRCIserIs
!%$> eKman a , b , c , d CacMnYnBitminGviCman . cUrRsayfa
(a b c d ) 4 a(c d ) b(d a ) c(a b ) d(b c)
!%%> ebI a , b , c CacMnYnBitviCmanEdl abc 1 enaHcUrRsayfa
3

3 2
3 2
a2
b
c
3

bc
ca
ab
2
!%^> eK[ a , b , c CacMnYnBitviCman . cUrRsayfa
3

a2
b2
c2
2
2
1
2
2
2
2
a ab b b bc c c ca a
!%&> eK[ a , b , c CacMnYnBitviCman . cUrRsayfa
a b
c 1 1 1 1 ab
bc
ca

( 2 2 2

)
2
2
2
bc ca ab a b c 2 b c c a a b
!%*> ebI a , b , c CacMnYnBitminGviCman enaHcUrRsayfa
2(a 2 1)(b 2 1)(c 2 1) (a 1)(b 1)(c 1)(abc 1)

!%(> eK[ x , y , z 0 nig n IN . cUrRsayfa


( xn 3 xn 3)( y n 3 y n 3)( z n 3 z n 3) ( x y z )3

!^0> ebI a , b , c 1 cUrRsayfa


1 a2
1 b2
1 c2

2
1 b c2 1 c a2 1 a b 2

!^!> ebI a , b , c IR bgajfa

eroberogeday lwm pln

a4 b 4 c4 1 5 a5 b 5 c5 1

- Page 229 -

vismPaBeRCIserIs
!^@> ebI x , y , z CabIcMnYnBitviCmanenaHcUrRsayfa
xy
yz
zx
x
y
z

xy x2 y2 yz y2 z2 zx z2 x2 2x z 2y x 2z y

!^#> eK[ a , b , c CabIcMnYnBitminGviCman . cUrbgajfa


(a2 ab b2 )(b2 bc c2 )(c2 ca a2 ) (ab bc ca)3

!^$> eK[ a , b , c 0 Edl a b c 1 .


bgajfa ab cb bc ac ca ba 2 .
!^%> eK[GnuKmn_ f (x) x x Edl 0 , 0 1
k> RKb; x [ 0 , 1 ] bgajfa f (x) 0 .
x> RKb; u , v 0 Edl u v bgajfa u .v u v .
!^^> eK[ a , b , c , x , x , x , ..., x CacMnYnBitviCmanEdl
a b c 1 nig x .x .x .x .x 1 . cUrbgajfa
(ax bx c)(ax bx c)...(ax bx c) 1 .
!^&> rgVg;carwkkgnRtIekaN ABC b:HRCug BC , CA , AB erogKaRtg;
AB
BC
CA
3
A , B , C .Rsayfa

AB
BC
CA
2

!^*> eK[ 0 x 2 . cUrRsayfa

1
1

1
1
1
2

sin n x
cos n x

eroberogeday lwm pln

, n IN *

.
- Page 230 -

Вам также может понравиться